SX

Download as doc, pdf, or txt
Download as doc, pdf, or txt
You are on page 1of 126

SURGERY A 59-year-old African-American man comes to the office and complains of a vague upper abdominal discomfort.

The pain has been present most of the time for the past three weeks, but has never been severe enough to make him seek immediate medical attention. The discomfort is not associated with eating or performing any activities. He denies any other medical problems. His vital signs are unremarkable. Abdominal examination reveals the presence of multiple scars from previous surgical procedures around the epigastric area. A 3 x 5 cm oblong-shaped mass is palpable in the epigastric area. There is no tenderness on direct palpation of the mass and the surrounding epigastric area. The patient tells you that he previously had a similar mass in the same area "but it was taken out". Which of the following is the most likely explanation for the mass? B. C. D. A. Abdominal aortic aneurysm Ventral hernia Muscle growth Abdominal wall neuroma

This subscription is licensed to user: roopika only User ID: 123489 Explanation: The patient's clinical presentation is consistent with a diagnosis of desmoid tumor. Desmoid tumors are locally aggressive neoplasms arising from fibroplastic elements within the muscle or fascial planes. Since these are locally invasive and slowly infiltrate the surrounding tissues and structures, these only cause local complications. These usually present as painless or minimally painful, slow-growing masses over the extremities, shoulder girdle, and hip-buttock area, although these may occur at all body sites. The diagnosis is generally confirmed with a tissue biopsy. Surgical excision with a wide margin of resection is the treatment of choice for patients with an easily approachable and resectable mass. There is a high rate of local recurrence, even after adequate resection. (Choice A) Abdominal aortic aneurysms usually present as a painless pulsatile mass. This patient does not have any risk factors for the development of aortic aneurysm. (Choice B) Ventral or incisional hernia can develop at the site of previous surgical scars; however, this is generally reducible, and pain is an unusual symptom of ventral hernia, unless it is strangulated. Educational Objective: Desmoid tumors are slow growing and locally aggressive benign neoplasms with a high rate of local recurrence, even after surgical excision. 11% of people answered this question correctly;

A 76-year-old Caucasian female comes to the emergency department after sustaining a fracture in her right proximal femur in an accidental, non-syncopal fall. She is a heavy alcohol drinker. Her past medical history is insignificant. She is started on thiamine and folic acid. The next morning, she undergoes orthopedic surgery for internal fixation of her right femur. Her post-operative course is unremarkable, except for mild agitation, presumably secondary to alcohol withdrawal. She is started on low molecular weight heparin for prophylaxis of DVT after her surgery. On her third, post-operative day, she develops respiratory distress. Pulse oximetry drops to 85% on room air. She is extremely confused and agitated. Her temperature is 36.7C(98F), blood pressure is 110/70 mm Hg, pulse is 120/min, and respirations are 30/min. Auscultation of the heart and lungs is essentially normal. There is no JVD or peripheral edema. Chest x-ray is normal and EKG shows sinus tachycardia. Which of the following is the most appropriate next step in the management of this patient? B. C. D. E. A. V/Q scan of the lungs Echocardiography D-dimer testing Start IV furosemide CT scan of the head

This subscription is licensed to user: roopika only User ID: 123489 Explanation: Pulmonary embolism may still occur in patients who are receiving DVT prophylaxis due to their high risk. The sudden respiratory failure, tachycardia and tachypnea in the above patient may be due to pulmonary embolism; therefore, a V/Q scan should be obtained. (Choice C) D-dimer testing is not very useful in this setting. Although it has a very high negative predictive value, results are almost always elevated in hospitalized patients, especially when the patient has undergone recent surgery. This test might be useful in a healthy patient who comes to the emergency department with a similar presentation. In such a case, a negative D-dimer test result almost always ascertains that the patient does not have PE. (Choice B) There is nothing in the patient's history, physical exam, and test findings which suggest cardiac dysfunction or failure. Echocardiography is not indicated. (Choice D) Giving IV furosemide to the patient is inappropriate. IV furosemide is useful in the setting of acute pulmonary edema, which can be ruled out since this patient has clear lungs on exam and on chest-x ray. (Choice E) Hypoxia can cause confusion. She is clearly in respiratory distress and has no focal neurologic signs. CT scan of the head is not indicated. Educational Objective:

Pulmonary embolism can still occur in patients who are receiving heparin for DVT prophylaxis. 88% of people answered this question correctly; This subscription is licensed to us

A 36-year-old Caucasian woman comes to the emergency department and complains of severe left calf pain. While she was running on a racetrack, she heard a loud snap, after which she felt an "excruciating pain" in her left calf area. She was a marathon runner in the past, but she stopped running two years ago, after she had her first baby. She just started retraining a week ago for an upcoming race. The physical findings are highly suggestive of a complete Achilles tendon rupture. Which of the following is the best clinical sign that will support this diagnosis? A. compression B. C. D. E. Increased plantar flexion of the ankle on calf muscle

Decreased plantar flexion of the ankle on calf muscle compression Increased dorsiflexion of the ankle on calf muscle compression Decreased dorsiflexion of the ankle on calf muscle compression No foot movement on calf muscle compression.

This subscription is licensed to user: roopika only User ID: 123489 Explanation: Rupture of the Achilles tendon may occur after abrupt calf muscle contraction. This typically occurs in men over the age of 40 who do not perform a regular leg-conditioning program. The most common symptoms are severe pain in the calf and the inability to stand up on the toes. The patient may note an audible snap at the time of injury. A positive Thompson test is further evidence of an Achilles tendon rupture. This test is performed with the patient kneeling on a chair, or lying prone on an examination table with his feet hanging over the edge. When the examiner squeezes the calf muscle on the normal side, the foot responds with plantar flexion. On the affected side, there is no foot response. Treatment consists of immediate immobilization of the lower leg and surgical repair of the tendon as soon as possible. Educational Objective: A complete Achilles tendon rupture leads to severe pain in the calf and the inability to stand up on the toes. A positive Thompson test further supports the diagnosis. 41% of people answered this question correctly;

A 25-year-old football player is brought to the emergency room because of sudden onset of pain and swelling of his right knee. He injured his right knee while playing football, when an opponent player, in an attempt to stop him, hit his right knee from the lateral side. He is currently unable to bear weight on his right knee. Examination showed a moderately swollen and tender right knee. Tenderness is more prominent over the medial side of his knee. Further assessment could not be done because of the pain. A MRI scan is ordered. The preliminary reading by a radiology resident is reported as an anterior cruciate ligament (ACL) and medial collateral ligament (MCL) injury. What other associated injury should you possibly worry about in this patient? B. C. D. A. Lateral meniscus injury Posterior cruciate ligament injury Medial meniscus injury Lateral collateral ligament injury

This subscription is licensed to user: roopika only User ID: 123489 Explanation: This is a typical description of the most recognized knee injury complex in athletes known as O' Donohue's unhappy triad. It typically occurs after a lateral knee injury, resulting in an anterior cruciate ligament tear, a medial collateral ligament injury, and a medial meniscal injury. Since the medial meniscus is less mobile than the lateral meniscus, it is more vulnerable to damage. Educational Objective: O' Donohue's unhappy triad typically occurs after a lateral knee injury, resulting in an Anterior cruciate ligament tear, a medial (Tibial) collateral ligament injury, and a Medial meniscal injury. Remember 'MAT'. 82% of people answered this question correctly;

An asymptomatic 60-year-old Caucasian man comes to the physician for a routine health maintenance examination. He has no other medical problems. He is a software engineer. He does not drink alcohol. He has smoked one pack of cigarettes daily for 10 years. His family history is not significant. He has no medications. His vital signs are within normal limits. The physical examination shows no abnormalities. The digital rectal examination is normal. His serum PSA level is 6 ng/mL. Which of the following is the most appropriate next step in the management of this patient? B. C. D. A. Repeat PSA in six months Refer to urologist for biopsy Reassurance Radical prostatectomy

This subscription is licensed to user: roopika only User ID: 123489 Explanation: The normal range of serum PSA is 0 to 4 ng/mL. Patients with PSA levels greater than 4 ng/mL should be referred to a urologist for a biopsy. (Choices A and C) Only a biopsy can confirm or rule out a diagnosis of prostate cancer. Reassurance or repeating the measurement of PSA levels in six months is not the appropriate choice because more than 20% of patients with PSA levels greater than 20 ng/mL have cancer on biopsy. The cutoff value has been reduced to 4 ng/dL to increase the sensitivity for diagnosing early prostate cancer. (Choice D) Radical prostatectomy is a treatment option for early prostate cancer. Pretreatment tests include biopsy of the tumor and a CT scan. Educational Objective: Patients with PSA levels greater than 4 ng/ml should be referred to a urologist for biopsy. 36% of people answered this question correctly

The following vignette applies to the next 2 items A 25-year-old Hispanic college student comes to the emergency department with complaints of sudden onset of left foot pain for the past hour. The pain started suddenly when he was sitting at his desk at college and has progressed gradually. He denies any history of similar episodes in the past or recent trauma to his left leg. His temperature is 37.2C(99F), blood pressure is 120/80 mmHg, heart rate is 86/min, and respiratory rate is 14/min. The lungs are clear on auscultation. On cardiovascular examination, the heart rhythm is regular, and a diastolic murmur is heard over the cardiac apex. The abdomen is soft, nontender and without any palpable masses. The neurological examination is within normal limits. Examination of the extremities reveal adequate, bilateral, femoral pulses. On the left side, the popliteal and dorsalis pedis pulses are absent. The left foot also appears pale and cooler than the right foot. The sensations on the left foot are intact and he is able to make voluntary movements. There is no tenderness on palpation of the lower left leg or foot. The vascular surgeons are immediately called, and the patient is taken to the operating room for urgent embolectomy, where an embolus obstructing the entire lumen of the left popliteal artery is removed. Item 1 of 2 Which of the following is the most appropriate next step in the management of this patient?

B. C. D. E.

A. Doppler examination of bilateral lower extremities Angiographic examination of the left lower extremity Echocardiogram Histologic examination of the embolus Hypercoagulation workup

This subscription is licensed to user: roopika only User ID: 123489 Explanation: The patient has a classic presentation of acute arterial occlusion. Acute ischemia of the limb due to acute arterial occlusion usually presents with five Ps; pain, pallor, pulselessness, paresthesias and paralysis. Almost all the patients present with a gradually progressive pain, usually in the distal extremity. The skin of the lower extremities is cool and pale, and the pulses are greatly reduced to absent. Acute arterial occlusion is usually the result of (1) an embolus from a distal source, (2) acute thrombosis due to a previously diseased vessel, or (3) direct trauma to the involved artery. The onset of symptoms can help in differentiating the etiology of arterial occlusion. The sudden onset of symptoms in a previously asymptomatic patient is most likely due to an embolus, while a history of gradually progressive symptoms in a previously symptomatic patient is consistent with thrombosis. The sudden onset of dramatic symptoms in this patient is most consistent with an embolic source of acute arterial occlusion. Most of the emboli are from a cardiac source, with a few coming from the arterial aneurysms or atherosclerotic plaques. Some of the wellrecognized causes of cardiac emboli include atrial fibrillation, a prior history of myocardial infarction and ventricular dysfunction, endocarditis, valvular disease, atrial myxoma, and the presence of a prosthetic aortic valve. It is important to find the exact cause of cardiac emboli to prevent future recurrences. After an embolectomy, the surgical specimen should be sent for histopathologic examination to ascertain the exact source of the emboli. (Choices A and B) Doppler or angiographic examination of the lower extremities is used for the diagnosis of acute arterial occlusion. These are not helpful in locating the source of the emboli. (Choice D) An echocardiogram is an important part of the diagnostic workup since this may identify a source of emboli; however, approximately 20% of the emboli can arise from a peripheral arterial source. Histologic examination is therefore more important since this can differentiate a cardiac source from a peripheral one. (Choice E) Hypercoagulation workup is important to look for inherited thrombophilias causing venous thrombosis. Since this patient appears to have acute arterial occlusion from a cardiac embolic source, the hypercoagulation workup is not indicated at this point.

Educational Objective: Histopathologic examination of the embolectomy specimen is extremely useful in locating the origin of the embolus which caused the acute arterial occlusion. 11% of people answered this question correctly;

Item 2 of 2 Which of the following is the most likely cause of the patient's condition? B. C. D. E. A. Prolonged immobilization Left atrial myxoma Atherosclerosis of the abdominal aorta Thromboangiitis obliterans Left ventricular thrombus

This subscription is licensed to user: roopika only User ID: 123489 Explanation: Based on the clinical presentation, the most likely cause of acute embolic arterial occlusion in this 25-year-old previously asymptomatic male is left atrial myxoma. Atrial myxomas are the most common primary cardiac tumors. Most of these arise from the left atrium, and the remaining arise from the right atrium and left ventricle. The tumors are typically pedunculated, with a stalk arising from the atrial septum. These can be extremely friable, resulting in embolization of the part of the myxoma to the systemic circulation. Some large tumors may initially present with signs and symptoms of mitral valve obstruction (diastolic murmur or "tumor plop"), rapidly worsening heart failure in otherwise young healthy individuals, or new onset atrial fibrillation. The diagnosis is usually made by echocardiography (either transthoracic or transesophageal echocardiography). Once the diagnosis of atrial myxoma is made, it should be excised as soon as possible to reduce the risk of recurrent embolization. (Choice A) Prolonged immobilization, along with other underlying risk factors, can cause deep venous thrombosis. It does not cause acute arterial occlusion. (Choices C & D) Thromboangiitis obliterans or atheroemboli is an unlikely cause of acute arterial occlusion in a young and otherwise previously healthy patient. (Choice E) A left ventricular thrombus is usually seen in patients with a prior history of myocardial infarction or severely reduced LV function. There is no such history in this patient.

Educational Objective: Left atrial myxomas can present with signs and symptoms of mitral valve obstruction (diastolic murmur or "tumor plop"), rapidly worsening heart failure in otherwise young healthy individuals, or new onset atrial fibrillation. These can also present with systemic embolization, thereby causing acute arterial occlusion in otherwise healthy patients. 51% of people answered this question correctly;

A 35-year-old man with Klinefelter syndrome comes to the physician with his brother. He complains of a right-sided scrotal swelling that began four months ago, and has been gradually increasing in size ever since. He denies local pain, fever or any other symptoms. He has no significant past medical history, and does not use tobacco, alcohol, or illicit drugs. He is currently not taking any medications. His vital signs are within normal limits. Examination reveals a firm, diffusely enlarged, and mobile right testis. The left testis is firm and smaller. The transillumination test is negative. The rest of the physical examination shows features consistent with Klinefelter syndrome (i.e., sparse pubic hair, gynecomastia and long legs), but is otherwise unremarkable. Which of the following is the most appropriate immediate step in management? A. syndrome. B. C. D. E. Do nothing, it is consistent with his history of Klinefelter

Perform a testicular biopsy. Perform a testicular ultrasound. Perform an ultrasound-guided FNAC of the right testis. Perform a radical inguinal orchiectomy.

This subscription is licensed to user: roopika only User ID: 123489 Explanation: The malignancy with a reported higher incidence in patients with Klinefelter syndrome is breast cancer. Testicular cancer does not occur with an increased frequency in such patients; however, a working diagnosis of testicular cancer should be made in any patient with a history of painless scrotal swelling, unless proven otherwise. The differential diagnosis of testicular swelling includes entities such as epididymitis, hydrocele, testicular torsion, varicocele, hernia or hematoma. The first step in the evaluation of males with testicular swelling is a scrotal ultrasound, which helps in differentiating intra- and extra- testicular lesions. Cystic or fluid-filled lesions are unlikely to be cancerous. If the lesion appears suspicious on ultrasound, it is followed with CT scan of the abdomen and pelvis (to detect retroperitoneal lymph nodes metastasis) and measurement of serum tumor marker levels (alpha fetoprotein, beta hCG). If the test results are equivocal and the index of suspicion is high, a radical inguinal orchiectomy provides the most accurate histological diagnosis. Testicular cancer, if diagnosed and treated in the early stages, is completely curable.

(Choice A) Unilateral, painless swelling of the testis is not a feature consistent with Klinefelter syndrome, and requires a prompt investigation to rule out cancer. (Choices B and D) A testicular biopsy or FNAC is not advisable since it is thought that such procedures may result in dissemination of the cancer cells. (Choice E) A radical inguinal orchiectomy is the gold standard for confirming the presence of a testicular neoplasm; however, performing imaging studies and serological tests prior to radical orchiectomy is recommended to confirm the suspicion of testicular cancer. Educational Objective: A painless testicular mass is considered as cancer unless proven otherwise. If scrotal ultrasound and serological tests confirm the suspicion, a radical inguinal orchiectomy is done to provide a histological diagnosis. 42% of people answered this question correctly;

The following vignette applies to the next 3 items A 25-year-old Caucasian man comes to the emergency department because of persistent pain and limited range in motion of the right wrist. Five hours ago, while playing football, he fell to the ground and landed on his outstretched right hand. He went home after the incident because his hand did not look too swollen and the pain was tolerable. A few hours later, he noticed that he was unable to move his hand, and decided to come to the hospital. He has no other medical problems. He denies the use of tobacco, alcohol or drugs. Examination shows mild swelling in the dorsum of the right hand. The hand cannot be fully flexed, and can be extended only by passive motion due to pain. There is severe tenderness on palpation of the anatomical snuffbox. Item 1 of 3 Which of the following is the most likely diagnosis? B. C. D. E. A. Colles' fracture Chondral fracture Wrist sprain Scaphoid fracture Triangular cartilage injury

This subscription is licensed to user: roopika only User ID: 123489 Explanation:

A history of falling on an outstretched hand is characteristic of a scaphoid fracture. It is most commonly seen in young adults between the ages of 15 and 30 years. It results from a fall on the outstretched hand, resulting in severe hyperextension and slight radial deviation of the wrist. Accompanying symptoms are complete (or greater than 50%) loss in range of motion of the wrist joint, severe pain, and stiffness. Physical examination reveals tenderness on palpation of the scaphoid within the anatomic snuffbox. (Choice C) Wrist sprain is characterized by mild pain, stiffness, normal range of motion of the wrist joint, minimal tenderness on palpation of the lunate and scaphoid (navicular) bones, and mild swelling of the dorsum. (Choice B) Chondral fracture is characterized by moderate pain, stiffness, and approximately 20% loss in range of motion of the wrist joint. (Choice E) Triangular cartilage injury is characterized by tenderness of the region distal to the ulnar styloid, which increases in severity with forced ulnar deviation. (Choice A) Patients with Colles' fracture have tenderness in the region located 2 cm below the radioulnar joint. In almost half of the cases, the ulnar styloid separates from the rest of the bone, and lateral view of the wrist reveals a dinner fork deformity. Educational Objective: A history of falling on an outstretched hand, complete (or greater than 50%) loss in range of motion of the wrist joint, severe pain, and tenderness on palpation of the scaphoid within the anatomic snuffbox is diagnostic of scaphoid fracture until proven otherwise. 84% of people answered this question correctly; Item 2 of 3 An x-ray of the wrist reveals no evidence of dislocation or fracture. After receiving analgesic therapy, the patient reports that the pain in his wrist has improved. Which of the following is the most appropriate next step in the management of this patient? A. Prescribe oral analgesics, rest for two weeks, and discharge the patient home B. Order a computerized tomography (CT) scan of the right wrist C. Order a magnetic resonance imaging (MRI) study of the right wrist D. Place a cast brace with the hand on dorsal hyperextension E. Place a cast brace with the hand on dorsal flexion This subscription is licensed to user: roopika only User ID: 123489 Explanation:

Patients with a non-displaced scaphoid fracture can have normal radiographs for up to two weeks after a traumatic incident. Overlooking this lesion may lead to complications such as traumatic arthritis (which results from nonunion of the fracture) and avascular necrosis of the scaphoid bone. A high clinical suspicion for scaphoid fracture warrants the use of further diagnostic studies (i.e., CT scan of the hand, bone scan), even if the initial x-ray results are negative. (Choice A) Analgesic therapy and resting the affected hand are indicated in the management of wrist sprain. (Choice C) MRI studies are used in the diagnosis of triangular cartilage injury. Although it may also be used in this case, its higher cost makes it less appealing than a CT scan, which is enough to make the diagnosis. (Choice E) Cast placement in the management of a patient with uncomplicated Colles' fracture involves immobilization of the hand in the neutral position, or ideally, in the normal volar tilt position. (Choice D) Cast placement with the hand in a hyperextended position is not beneficial for both Colles' and scaphoid fractures. The uncomplicated, undisplaced scaphoid fractures are treated with immobilization in a long or short arm thumb spica cast with the wrist in slight radial deviation and in neutral flexion. Educational Objective: Although displaced fractures of the scaphoid can be identified in x-rays immediately after trauma, some scaphoid fractures can take one or two weeks before it becomes apparent in x-rays. A high clinical suspicion for scaphoid fracture therefore warrants the use of further diagnostic studies (i.e., CT scan of the hand, bone scan), even if the initial x-ray results are negative. The uncomplicated, undisplaced scaphoid fractures are treated with immobilization in a long or short arm thumb spica cast with the wrist in slight radial deviation and in neutral flexion. 12% of people answered this question correctly Item 3 of 3 The patient is concerned about the complications that he might develop. Which of the following is the most common complication of his injury? B. C. D. E. A. Avascular necrosis Nonunion Malunion Infection No complications

This subscription is licensed to user: roopika only User ID: 123489

Explanation: The most common complication of scaphoid fractures is nonunion. The other important complication is avascular necrosis. These two complications often result because the blood flows from the distal to proximal portion of the scaphoid bone, and this proximal portion is completely dependent on the distal blood supply. Proximal fractures of the scaphoid therefore require longer immobilization (up to 12 weeks) to ensure adequate healing. Educational Objective: The most common complication of scaphoid fracture is nonunion. 18% of people answered this question correctly A 37-year-old woman has had left breast discomfort for several months. The breast has also been painful before menses. Yesterday, she noted a lump. Her past medical history and family history are unremarkable for breast and gynecologic diseases. She had a baseline mammogram at the age of 36 that was normal. A smooth, soft, movable mass is palpated in the upper outer quadrant of the left breast. Some diffuse nodularity is present bilaterally. The vital signs and the rest of the exam are normal. Fine-needle aspiration (FNA) of the mass reveals a thin, greenish fluid. The mass disappeared completely after FNA. What should the patient's follow-up therapy include? B. C. D. E. A. Mammogram Oral contraceptives Observation Ultrasound Antibiotics

This subscription is licensed to user: roopika only User ID: 123489 Explanation: Cysts that disappear completely, contain non-bloody fluid, and possess a characteristic diffuse nodularity, are most likely due to fibrocystic disease. Careful observation for recurrence of the mass is the standard follow-up therapy. (Choice A) A mammogram would be indicated if the mass didn't disappear completely, or if its content was bloody. (Choice D) An ultrasound is indicated if the mass cannot be visualized using a mammogram because of dense, normal breast tissue (seen especially in young women). Ultrasound is also useful for the evaluation of cystic lesions, but this cyst has already been drained.

(Choice B) Oral contraceptives are sometimes given to control mastodynia; however, for this patient, the mastodynia is expected to resolve after FNA. (Choice E) The patient doesn't have signs of infection. Thick, not thin greenish fluid, could be pus. Antibiotics are not necessary. Educational Objective: A breast mass that produces non-bloody aspirate and disappears completely on aspiration, does not need any further evaluation other than observation. 62% of people answered this question correctly The following vignette applies to the next 2 items You are called to see a 32-year-old African American man with no past medical history who sustained a mid-shaft fracture of his left tibia during a motor vehicle accident. Open reduction and internal fixation were performed immediately after the patient arrived in the emergency department. The leg was stabilized in a plaster cast. Continuous morphine was provided for analgesic effect. At present, the patient complains of significant pain in his left calf. He appears very uncomfortable and is sweating and shifting restlessly in bed. He pleads for better analgesia. Pulses are present bilaterally in his lower extremities. Compartment syndrome seems likely. Item 1 of 2 Which of the following is an early sign of compartment syndrome in a limb? B. C. D. E. A. Erythema Homans' sign Loss of deep tendon reflexes Pain with passive muscle motion Tissue ulceration and necrosis

This subscription is licensed to user: roopika only User ID: 123489 Explanation: The events most commonly responsible for compartment syndrome include crush injuries, fractures, prolonged external compression, burns, and snake bites. Such events cause either a decrease in compartment size or an increase in compartment pressure. As the intracompartmental pressure rises, the capillary blood perfusion is reduced until it can no longer maintain tissue viability. Early physical signs of acute compartment syndrome include tightness, weakness, and pain with passive muscle motion (Choice D). One of the most suggestive signs of compartment syndrome is pain out of proportion to the injury. Hypesthesia and paresthesia may be documented as well.

Erythema (Choice A) is typically seen with venous congestion. Acute compartment syndrome is more commonly associated with cyanosis, pallor, or mottling of the extremity. Pain or increased resistance with dorsiflexion of the foot is known as Homans' sign (Choice B). It is an unreliable indication of deep venous thrombosis. The loss of deep tendon reflexes (Choice C) can be seen in association with acute compartment syndrome, but is typically a later finding. Tissue ulceration and necrosis (Choice E) would be a very late finding of compartment syndrome. Educational Objective: Early physical signs of acute compartment syndrome include tightness, weakness, and pain with passive muscle motion. One of the most suggestive signs of compartment syndrome is pain out of proportion to the injury. 85% of people answered this question correctly; Item 2 of 2 Which of the following is considered the most ominous sign of compartment syndrome? B. C. D. E. A. Paresthesia Loss of arterial pulse Mottling of the extremity Pallor of the extremity Excruciating pain that persists after cast removal

This subscription is licensed to user: roopika only User ID: 123489 Explanation: The most ominous sign of compartment syndrome is the loss of the arterial pulse, as that signifies a cessation of blood flow to the extremity (Choice B). Muscle tissue has impairment of function after 2-4 hours of ischemia and irreversible loss of function after 4-12 hours, while nerve tissue has impairment of function after 30 minutes of ischemia and irreversible loss of function after 12-24 hours. Treatment includes splitting of the cast and underlying padding, which can decrease the compartment pressure by 50-85%. Paresthesia (Choice A) is an early finding and can be associated with arterial pulses that are diminished and not absent. It is therefore not considered the most ominous finding of compartment syndrome.

Mottling (Choice C) and pallor (Choice D) of the extremity are physical findings occasionally associated with acute compartment syndrome. However, they can be seen in conjunction with arterial pulses that are diminished and not absent, and are therefore not considered the most ominous findings of compartment syndrome. Excruciating pain does sometimes persist after cast removal (Choice E) and is not always cause for increased concern. However, if any symptoms do not resolve within an hour after cast removal and the pressure measurement remains elevated, fasciotomy is indicated. Normal function will be regained in approximately two-thirds of patients if fasciotomy is performed within 12 hours of the onset of compartment syndrome. Educational Objective: The most ominous sign of compartment syndrome is the loss of the arterial pulse, as that signifies a cessation of blood flow to the extremity. Treatment includes splitting of the cast and underlying padding, which can decrease the compartment pressure by 50-85%. 60% of people answered this question correctly

A 62-year-old Hispanic woman comes to the office with complaints of a sense of heaviness and discomfort in her right groin. Her discomfort is worse with standing and coughing, and is relieved by lying flat. She also noticed a painful bulging mass at the same site two days ago, which resolved spontaneously after two hours. She has a past medical history of breast cancer and chronic bronchitis. She had a left mastectomy at age 48 and two cesarean sections at 28 and 33 years of age. She smokes a pack of cigarettes daily. On examination and Valsalva maneuver, you find a right-sided femoral hernia that is easily reducible. There is no evidence of incarceration or strangulation. Which of the following is the most appropriate next step? B. C. D. E. A. Observation alone Observation till the femoral hernia grows to 5 cm in size Refer to surgery for hernia repair Prescribe an abdominal binder Prescribe narcotic analgesics

This subscription is licensed to user: roopika only User ID: 123489 Explanation: A hernia is a small area of weakness or defect in the overlying muscular or fibrous wall through which abdominal contents pass. Groin hernias are either inguinal or femoral hernias. The most common presentation is the dull sensation of heaviness or discomfort that is more pronounced with prolonged standing or straining (increased intraabdominal pressure). These are easily reducible in most cases, and symptoms are relieved with lying flat. A dreaded complication is incarceration or strangulation. It occurs when a tight constriction ring forms around the herniated contents and the hernia then becomes

irreducible. Patients typically present with a constant pain that gets worse over time. If left untreated, it leads to necrosis and gangrene of the contents, and can be lifethreatening. A femoral hernia is much more predisposed to strangulation than an inguinal hernia. The effective treatment for all types of hernias is surgical repair. Risks of delaying the surgery are mainly those related to incarceration and strangulation. (Choices A and B) Observation alone is not appropriate in patients with a femoral hernia. The risk of strangulation is the greatest in the early stages and with small hernias. (Choice D) Abdominal binders are only useful for symptomatic relief in patients with large incisional or umbilical hernias. These are not used for patients with femoral hernias. The only nonsurgical therapy for groin hernias is the use of a truss for symptom relief, but this is generally discouraged. (Choice E) Narcotic analgesics may mask the clinical features of strangulation. These should not be prescribed for long-term management of femoral hernias. Educational Objective: The primary treatment for all types of hernias is surgical repair. 87% of people answered this question correctly; A 76-year-old Caucasian male comes to see you in the office. He complains that he has visible and palpable tortuous swelling on both of his lower legs. It is associated with mild leg heaviness and cramping, especially in the evening. His symptoms are worse with prolonged standing, and improve with leg elevation. He has a past medical history of hypertension, hyperlipidemia, benign prostatic hyperplasia, and degenerative joint disease of both knees. On examination of his lower legs, you notice a few scattered palpable veins consistent with a diagnosis of varicose veins. There is no edema, chronic skin hyperpigmentation, or leg ulcers. Peripheral pulses are full and equal on both sides. Which of the following is the most appropriate next step in the management of this patient? B. C. D. E. A. Leg elevation and compression stockings Injection sclerotherapy with saline Injection sclerotherapy with saline and local anesthetic Surgical ligation and stripping External laser therapy

This subscription is licensed to user: roopika only User ID: 123489 Explanation:

Varicose veins refer to visible, palpable, and tortuous superficial veins of the legs, usually on the calf or medial thigh. Most of the patients with varicose veins are asymptomatic. Some patients may complain of leg cramping, heaviness, fatigue, and swelling. The symptoms are generally worse in the evening with prolonged standing, and improve with leg elevation. Treatment of varicose veins is aimed at alleviating symptoms, although some patients may seek a physician's attention due to cosmetic reasons. Various treatment options for varicose veins include leg elevation, compression stockings, sclerotherapy, and surgical ligation. Most patients with symptomatic varicose veins should initially be treated with conservative measures such as leg elevation and compression stockings. Compression stockings, along with leg elevation and weight reduction, decreases the venous pressures in the lower extremities by direct compression, and improves the patient's symptoms. Compression stockings should not be used in patients with an underlying arterial insufficiency. (Choices B and C) Injection sclerotherapy with or without anesthetics is used in patients with symptomatic, small, varicose veins who have failed at least three to six months of conservative treatment. It involves the injection of a sclerosing agent into the affected vein. It causes endothelial damage and sclerosis of the involved vein, thus preventing further vein filling. (Choice D) Surgical ligation and stripping is used in patients with large symptomatic varicose veins with ulcers, bleeding, or recurrent thrombophlebitis of the veins. (Choice E) External laser treatment is used to treat particular veins and/or telangiectasias. It is not used for the treatment of varicose veins. Educational Objective: Most patients with symptomatic lower extremity varicose veins should be treated initially with conservative measures such as leg elevation, weight reduction, and compression stockings. 84% of people answered this question correctly

You are the emergency room physician on night shift, when you hear about a motor vehicle accident on a nearby highway. The patient, a 35-year-old male, was found unresponsive at the accident scene. He was emergently intubated at the scene by paramedics, and was transferred to the emergency department, where he is now under your care. In the emergency department, he is unresponsive to all verbal or tactile stimuli. There is a smell of alcohol coming from his mouth. His vital signs reveal a temperature of 36.7C (97F), heart rate of 110/min, and blood pressure of 95/55 mmHg. His oxygen saturation is 88% on high flow oxygen. On physical examination, there is reduced

expansion of the left side of the chest, and breath sounds are markedly decreased on the left side. The heart sounds are regular but distant, and without any murmurs. Item 1 of 2 Which of the following should be done immediately for this patient? B. C. D. E. A. Obtain a chest CT scan. Insert a chest tube on the right side. Initiate mechanical ventilation with higher tidal volumes. Check the endotracheal tube placement. Perform a needle decompression on the left side of chest.

This subscription is licensed to user: roopika only User ID: 123489 Explanation: It is important to always remember and follow the various steps involved in basic and advanced cardiovascular life support. Airway control is the first priority in any unconscious or obtunded patient. After the airway has been secured by endotracheal intubation, it is important to confirm its correct placement in the trachea by checking for lung expansion and auscultating for breath sounds on both sides. The patient in the above vignette has a reduced lung expansion and marked hypoventilation on the left side of the chest. This is consistent with a right main stem bronchus intubation, and can be easily managed by withdrawing the endotracheal tube back by a few centimeters. This would correct the hypoxia as well. The patient should be reexamined after withdrawing the endotracheal tube to check for lung expansion and airflow symmetry. (Choice A) Chest CT is not required at this time. It is important to perform the "ABC" (airway, breathing and circulation) check first, prior to obtaining any imaging for further workup of the hypoxia. (Choice B) There is no evidence of right-sided pathology. A chest tube insertion on the right side would be inappropriate. (Choice C) Mechanical ventilation with high tidal volumes can lead to high peak and plateau pressures on the right side. It can cause further barotrauma and development of pneumothorax on the right side. (Choice E) Tension pneumothorax can present with similar signs and symptoms of a patient with a blunt chest wall trauma. Patients typically have decreased or absent breath sounds on the affected side, with a mediastinal shift to the opposite side. They also have marked hypoxia and hemodynamic instability. Although a thoracic needle decompression may also be needed, checking the endotracheal tube placement is an important part of the initial assessment in ACLS protocol, and should be performed first.

Educational Objective: Remember and follow the "ABC's" of cardiopulmonary resuscitation. After the airway has been secured by endotracheal intubation, it is important to confirm its correct placement in the trachea by checking for lung expansion and auscultating for breath sounds on both sides. 55% of people answered this question correctly

Item 2 of 2 The patient's condition deteriorates after you perform the initial step. He continues to have markedly decreased breath sounds on the left side. His systolic blood pressure is about 70 mmHg, and his oxygen saturation is 80% on 100% oxygen. Which of the following is the most appropriate next step in the management of this patient? B. C. D. E. A. Obtain a chest CT scan. Aggressive intravenous hydration. Start him on a dopamine infusion. Check the endotracheal tube placement. Perform a needle decompression on the left side of the chest.

This subscription is licensed to user: roopika only User ID: 123489 Explanation: The patient in the above scenario continues to have a persistent and rapid decline in his condition, even after confirming the endotracheal tube placement. A marked hypoventilation, or absent breath sounds of the left side with hypoxia, and cardiovascular collapse support the diagnosis of tension pneumothorax in a patient involved in a motor vehicle accident. Tension pneumothorax causes marked compression and shifting of mediastinal structures, including the heart and great vessels, causing a reduced cardiac output and hypotension. The patient should have an emergent large-bore needle decompression, followed by a chest tube thoracostomy, to relieve the compression on the mediastinal structures. (Choice A) A chest CT scan can confirm the diagnosis of tension pneumothorax, but the delay in treatment will be fatal in this case. (Choice B) There is no evidence of hypovolemic shock in this patient. Although intravenous hydration may be required, it is not the best next step in management at this point. (Choice C) Dopamine infusion is not indicated in this setting. Needle decompression on the left side will rapidly reverse the hemodynamic collapse, and improve his blood pressure and hypoxia.

Educational Objective: Emergent large-bore needle decompression should be performed in a patient with suspected tension pneumothorax causing hemodynamic instability and cardiovascular collapse. 66% of people answered this question correctly;

The following vignette applies to the next 4 items A 24-year-old female is admitted to the hospital after a motor vehicle accident, where she sustained a right humerus fracture, multiple rib fractures and a contusion over her right lower leg. The next morning, a nurse calls to inform you that she is complaining of severe pain in her right lower leg. Her temperature is 37.2C(99F), heart rate is 96/min, respiratory rate is 18/min, and blood pressure is 140/82 mmHg. Physical examination reveals a tense swelling around the right calf region. The pain is worsened on palpation and passive movements of the foot. Neurological examination reveals motor weakness and hypoesthesia of the distal right leg. Item 1 of 4 Which of the following is the most likely diagnosis? B. C. D. E. A. Fat embolism Deep venous thrombosis Acute compartment syndrome Acute vascular occlusion Neural compression

This subscription is licensed to user: roopika only User ID: 123489 Explanation: Acute compartment syndrome refers to ischemic tissue damage secondary to elevated pressures in the enclosed compartments of the lower legs or forearm. When the tissue pressure in an enclosed compartment exceeds the perfusion pressure, the resulting diminished tissue perfusion and compromised blood flow to the muscles and nerves inevitably lead to ischemic tissue necrosis. Majority of the cases involving the lower extremities are due to a traumatic event, most commonly tibial fractures. Other causes include a crush injury or other long bone fractures in a motor vehicle accident, a tight cast or dressing after trauma, and drug overdose. Patients usually present with severe pain which is out of proportion to the extent of injury. The pain is typically worsened by passive movements of the involved muscles.

Sensory nerves are usually affected earlier than the motor nerves, and the neurologic deficit presents as decreased vibration sense, decreased two-point discrimination, numbness or hypoesthesia. Late features include extremity paralysis and absent distal pulsation (pulseless paralysis). (Choice A) Fat embolism is infrequently seen in patients with long bone or pelvic fractures. Patients usually present with a triad of hypoxemia, neurological abnormalities (i.e., confusion), and a petechial rash (involving the head, neck, anterior chest wall, or axilla). (Choice B) Deep venous thrombosis usually does not present acutely in a setting of motor vehicle accident in an otherwise healthy young patient. It is a rare cause of compartment syndrome and does not compromise blood circulation and neuromuscular function. (Choice D) Vascular occlusion secondary to a motor vehicle accident usually presents more suddenly and dramatically. Acute compartment syndrome usually has a lag period of a few hours before irreversible nerve injury and muscle necrosis occurs. (Choice E) Nerve compression may occur in a patient after a motor vehicle accident and possible bone fracture; however, it does not lead to blood flow compromise and muscle necrosis (pain with passive movements of the involved muscles). Educational Objective: Acute compartment syndrome usually occurs after a traumatic event and causes pain, paresis, hypoesthesia, and diminished to absent pulses in the involved limb. 87% of people answered this question correctly;

Item 2 of 4 Which of the following is the most common life-threatening complication of the above condition? B. C. D. E. A. Disseminated intravascular coagulation Rhabdomyolysis and renal failure Pulmonary embolism Gangrene of the limb Thrombocytopenia

This subscription is licensed to user: roopika only User ID: 123489 Explanation: Acute compartment syndrome results in markedly diminished to absent tissue perfusion within hours of the inciting event, causing tissue necrosis, muscle infarction, and

rhabdomyolysis, which releases myoglobin into the peripheral circulation. Myoglobin is directly toxic to the renal tubules, and subsequently causes acute tubular necrosis and acute renal failure. Acute renal failure and its complications (electrolyte disturbances) are one of the most common life-threatening complications of acute compartment syndrome. Laboratory studies typically reveal markedly elevated creatinine kinase levels and the presence of myoglobin in the urine (positive dipstick for blood in the absence of RBC's in the urine). (Choices A, C, D, E) Thrombocytopenia, disseminated intravascular coagulation, pulmonary embolism, and gangrene of the limb are not usually seen in patients with acute compartment syndrome. Educational Objective: Rhabdomyolysis and subsequent development of acute renal failure is one of the most common and severe life-threatening complications of acute compartment syndrome. 31% of people answered this question correctly

Item 3 of 4 Which of the following is the most appropriate next step in the management of this patient? B. C. D. E. A. Administer oxygen Start the patient on anticoagulation Order venous Doppler ultrasonography Check the tissue pressure Order a nerve conduction study

This subscription is licensed to user: roopika only User ID: 123489 Explanation: Compartment syndrome is characterized by an increase in the tissue pressure in the enclosed myofascial compartments of the extremities. When the elevated tissue or compartment pressure reaches its threshold level, the capillaries collapse, and this eventually leads to tissue and muscle necrosis. The exact value for the tissue pressure at which blood flows to the muscle and nerve tissue stops is controversial. The current general consensus for the threshold value is greater than 30 mmHg. It is therefore important to measure the tissue or compartment pressure early in the course of management, especially if the diagnosis is in question. (Choice A) Oxygen is usually used in the supportive treatment of patients with fat or cholesterol emboli.

(Choice B) Anticoagulation is usually required for patients with deep venous thrombosis. (Choice C) Venous Doppler ultrasonography is useful for the diagnosis of deep venous thrombosis. It is not helpful for the diagnosis of compartment syndrome. (Choice E) Nerve conduction studies are helpful in the diagnosis and localization of the site of nerve damage. Educational Objective: Direct measurement of the compartment or tissue pressure is the diagnostic procedure of choice for patients with suspected acute compartment syndrome. 81% of people answered this question correctly

Item 4 of 4 The appropriate step was taken for the patient. Which of the following is the best next step in management? B. C. D. E. A. Continue with oxygen therapy Obtain a hypercoagulable panel Consult a vascular surgeon Perform urgent fasciotomy Review the results of venous Doppler ultrasonography

This subscription is licensed to user: roopika only User ID: 123489 Explanation: Acute compartment syndrome is a surgical emergency. Any delay in treatment leads to irreversible muscle and nerve damage. A compartment pressure of 30 mmHg or greater warrants an emergent fasciotomy (also known as compartment release). Surgical decompression aims to relieve the pressure within the enclosed compartment and to restore the blood flow to muscles and other tissues within 6-10 hours of the initial symptoms. Some patients may develop a persistent sensory or motor deficit after an episode of acute compartment syndrome despite early fasciotomy. (Choice A) Oxygen therapy has no role in the management of patients with acute compartment syndrome. (Choice B) Obtaining a hypercoagulable panel may be useful in patients with idiopathic deep venous thrombosis. It is not indicated in patients with acute compartment syndrome.

(Choice C) A consultation with a vascular surgeon is necessary in patients with vascular injury or occlusion. (Choice E) Venous Doppler ultrasonography is useful to diagnose deep venous thrombosis. It has no role in the diagnosis or management of patients with acute compartment syndrome. Educational Objective: An emergent fasciotomy is the definitive treatment in patients with acute compartment syndrome. Surgical decompression aims to restore the capillary blood flow and tissue perfusion. 87% of people answered this question correctly

A relatively healthy 71-year-old Asian man comes to the office for a routine follow-up visit after a recent discharge from the hospital. During the last visit, he was diagnosed with benign prostatic hypertrophy with a slight elevation of PSA. He subsequently underwent TURP (transurethral resection of the prostate), and his prostatic specimen histopathology results came back as prostate adenocarcinoma in situ. He has a history of mild COPD and hypertension. He has smoked one pack of cigarettes daily for 38 years. Which of the following is the most appropriate next step in his care? A. exploration B. C. D. E. Radical, suprapubic prostatic resection with lymph node

Pelvic radiation and bilateral orchiectomy Pelvic radiation and estrogen therapy Chemotherapy Anti-androgen therapy

This subscription is licensed to user: roopika only User ID: 123489 Explanation: Suprapubic resection of the prostate is the currently accepted therapy for patients whose prostate cancer was diagnosed through TURP, needle biopsy, or cytology. This procedure is accompanied by lymph node resection, which can be preceded by sentinel lymph node identification through technetium radio labeling for a higher yield. (Choices B and C) Pelvic radiation can cause serious local lesions (such as strictures) and bowel or rectal damage. It is reserved for more advanced stages of the disease. (Choices D and E) Anti-androgen agents and chemotherapy have not increased survival when used as adjuvants in this type of patient. Hormonal therapy with bicalutamide is promising (and is being used already in Europe) but is not yet FDA-approved. These are usually reserved for advanced stages of the disease.

Educational Objective: Patients with an incidental finding of prostate cancer in situ through TURP or biopsy need further surgical resection of the gland through a suprapubic approach. Radiotherapy, estrogens, and chemotherapy are measures reserved for more advanced stages of the disease. 36% of people answered this question correctly;

You are working in the emergency department of a small community hospital, when the paramedics bring in a 24-year-old man after being involved in a motor vehicle accident. At the scene of the accident, the medics noted that he had sustained a traumatic amputation of his right index finger. They were able to retrieve the severed finger and brought it to the emergency room. You provided the patient with initial wound care and achieved hemostasis with a compression dressing. Your hospital does not have a facility for digit replantation and reconstructive surgery, and you decided to transfer the patient to a nearby tertiary care center. Five minutes later, the nurse approaches you and asks, "How do you want me to preserve his severed finger?" Item 1 of 2 Which of the following is the most appropriate response? A. Wrap it in sterile gauze, moisten it with sterile Ringer's lactate and place it in a sterile sealed plastic bag B. Wrap it in sterile gauze, moisten it with sterile water and place it in a sterile sealed plastic bag C. Wrap it in sterile gauze, moisten with sterile saline and place it in a sterile sealed plastic bag D. Wrap it in sterile gauze, moisten it with Ringer's lactate and antibiotic solution and place it in a sterile sealed plastic bag This subscription is licensed to user: roopika only User ID: 123489 Explanation: Recent technologic advances have led to successful replantations of various body parts including fingers, thumbs, ear, genitalia, and arms severed in various accidents. This has led to the preservation of the function and appearance of most body parts, as well as an improved quality of life for the patient. As a general rule, all amputated body parts should be retrieved and brought to the emergency room. Since the thumb or finger amputation can especially compromise the function of the hand, all attempts should be made to retrieve and replant the digit in the

hope to preserve function. For transportation purposes, the amputated part should be wrapped in saline-moistened sterile gauze and sealed in a sterile plastic bag. (Choices A and D) It is not recommended to keep the amputated thumb or body part in Ringer's lactate or an antibiotic solution. Educational objective: As a general rule, all amputated body parts should be retrieved and brought to the emergency room. For transportation purposes, the amputated part should be wrapped in saline-moistened sterile gauze and sealed in a sterile plastic bag. 65% of people answered this question correctly

Item 2 of 2 The appropriate step is taken. At your request, the nurse brings an insulated cooling container to transport the severed thumb. Before putting the plastic bag in the container, she asks you "What do you want to use to fill up the container?" Which of the following is the most appropriate response? B. C. D. E. A. Fill it with sterile saline solution Fill it with sterile water Fill it with ice only Fill it with saline moisten sterile gauze Leave it empty and just put the plastic bag in it

This subscription is licensed to user: roopika only User ID: 123489 Explanation: After adequate steps have been taken to preserve the severed finger, it should be sealed in a plastic bag and placed on ice in a container. The severed part should never be placed directly on ice, as this could lead to injury to the vessels and other tissues. It should also not be immersed in water as this may make the replantation technically more difficult. Educational Objective: The severed part should be sealed in a sterile plastic bag and placed on ice for transportation purposes. 83% of people answered this question correctly.

A 66-year-old Asian-American woman presents to the emergency department and complains of lower back pain, difficulty walking, urinary incontinence, fever of 39.3C (102.8F), and decreased appetite. She was recently diagnosed with a herniation of disc L4-5 and was given an epidural injection for radicular pain two days ago. Her past medical history is significant for diabetes mellitus, chronic renal failure, and hypertension. She is alert and oriented. Her temperature is 38.3C (100.9F), blood pressure is 136/88 mm Hg, pulse is 86/min, and respirations are 14/min. Physical findings include poor dentition, diffusely tender abdomen, decreased rectal sphincter tone, mild tenderness upon palpation of the lumbar region, and absent deep tendon reflexes in both lower extremities. Initial laboratory examination includes a WBC of 22,000/mm3 with 69% neutrophils and 15% bands, and an erythrocyte sedimentation rate of 104 mm/hr. The result of her urinalysis is normal. What is the most appropriate course of action in this woman's care? B. C. D. E. A. Perform lumbar puncture and start antibiotics Obtain MRI of spine Immediate neurosurgery consult Administer antibiotics Obtain bone scan

This subscription is licensed to user: roopika only User ID: 123489 Explanation: An epidural abscess should be suspected in a susceptible patient who presents with fever and back pain. Patients who are susceptible are the elderly, immunocompromised individuals (i.e., patients with AIDS, cancer, diabetes mellitus, chronic renal failure, or alcoholism), and patients who were recently involved in trauma, spinal surgery, or administration of epidural anesthesia. The diagnosis must be made immediately because delays in treatment increase the risk of permanent neurologic damage or death. A gadolinium-enhanced MRI is the preferred imaging modality to confirm the diagnosis because it adequately demonstrates inflammatory changes in the soft tissue. When MRI is not available, CT with myelography is an alternative. Once the diagnosis is made, antibiotic therapy should be guided by obtaining a culture specimen via CT-guided aspiration or open biopsy. The most common etiologic agent is Staphylococcus aureus. Spinal epidural abscesses is a surgical emergency; early surgical decompression and drainage, preferably within the first 24 hours, is the most important part of management to improve the ultimate prognosis. (Choice A) Lumbar puncture is not required to make the diagnosis and is not recommended. However, blood cultures should be drawn and antibiotics should be started. (Choice C) A neurosurgery consult should be obtained once more information is collected that suggests a diagnosis.

(Choice D) Even though antibiotics are important, making the diagnosis and urgent surgical drainage is more important. Educational Objective: Epidural abscesses are best diagnosed with gadolinium-enhanced MRI of the spine. Spinal epidural abscess is a surgical emergency. Early surgical decompression and drainage, preferably within the first 24 hours is the most important part of management to improve the ultimate prognosis. 39% of people answered this question correctly

The following vignette applies to the next 2 items A 75-year-old Caucasian man comes to your office complaining of periodic leg pain. He says, 'I get this dull, crampy pain when I walk even one block.' The pain improves with rest. He has no pain at night. His past medical history is significant for myocardial infarction experienced four years ago and carotid endarterectomy performed two years ago. Currently, he takes aspirin, clopidogrel, propanolol, simvastatin, and enalapril. He does not smoke anymore, but he used to smoke heavily for 40 years. He consumes 1-2 bottles of beer on weekends. His family history is significant for stroke in his father and breast cancer in his mother. His blood pressure is 135/90 mmHg and heart rate is 65/min. His left foot is shiny and devoid of hair. The left-sided posterior tibial and dorsalis pedis pulses are diminished. There is increased pallor upon elevation of the left foot. Item 1 of 2 What is the best initial investigation in this patient? B. C. D. E. A. Ankle-brachial pressure index Doppler ultrasonography MRA of left leg Contrast angiogram Exercise treadmill testing

This subscription is licensed to user: roopika only User ID: 123489 Explanation: Ankle-brachial pressure index (ABI) is a simple and relatively inexpensive method to confirm the clinical suspicion of peripheral vascular disease and to judge its severity. It is calculated by measuring the systolic blood pressure using a Doppler probe at the brachial, posterior tibial and dorsalis pedis arteries, and then dividing the lower extremity pressures by the brachial pressure. The normal ABI is 1.0 to 1.3 because the blood

pressure is normally higher in the ankle than in the arm. An ABI below 0.9 is highly accurate in detecting clinically significant peripheral vascular disease; an ABI below 0.4 usually implies severe ischemia. (Choice E) If the ABI is normal at rest, but the symptoms strongly suggest intermittent claudication, the ABIs should be obtained before and after exercise on a treadmill. (Choice B) Although Doppler ultrasonography is highly accurate in detecting peripheral artery disease, ABI is simpler and less expensive, making it the initial screening test of choice. (Choice C) MRA is usually performed if revascularization is being considered; it is too expensive to serve as an initial screening test. (Choice D) Invasive procedures have a limited role in the evaluation of patients with peripheral vascular disease because accurate non-invasive alternatives are available. Educational Objective: ABI is the best initial screening test to detect peripheral vascular disease. An ABI below 0.9 is highly accurate in detecting clinically significant peripheral vascular disease. 70% of people answered this question correctly

Item 2 of 2 After discussing the prognosis and available treatment options, the patient decides to have conservative therapy. He requests for therapy that will allow him to carry out his daily activities. Which of the following interventions will most likely improve this patient's condition? B. C. D. E. A. Add pentoxifylline Increase the dose of clopidogrel Add cilostazol Stop propranolol Switch to metoprolol

This subscription is licensed to user: roopika only User ID: 123489 Explanation: Cilostazol is currently the most effective conservative therapy available for peripheral vascular disease. Cilostazol is a phosphodiesterase inhibitor that inhibits platelet aggregation and acts as a direct arterial vasodilator. It has been shown to improve maximal walking distance in patients with moderate to severe intermittent claudication.

Although it has anti-aggregant properties, it can be taken in combination with aspirin and clopidogrel. (Choice A) Cilostazol is clearly more effective than pentoxifylline. (Choice B) There is no evidence that high-dose clopidogrel improves symptoms in patients with intermittent claudication. (Choices D and E) Beta-adrenergic blockers have been considered relatively contraindicated in peripheral arterial disease because of the perceived risk that these drugs could worsen intermittent claudication. At the same, many studies conducted to assess this risk have failed to show any adverse effect associated with decreased peripheral circulation. Although selective beta-blockers are theoretically preferred over non-selective agents such as propranolol, stopping propranolol therapy or switching to metoprolol should not be expected to produce significant improvement of this patient's intermittent claudication symptoms. Educational Objective: Cilostazol is currently the most effective conservative therapy available for peripheral vascular disease. 29% of people answered this question correctly.. A 47-year-old Caucasian man is brought to the emergency department of a small rural clinic. He was hunting in the woods with a group of friends 45 minutes ago, when he was accidentally shot by another hunter. He complains of abdominal pain, shortness of breath and palpitations. The symptoms are progressively getting worse. He does not use tobacco, alcohol, or illicit drugs. His temperature is 36.1 C (97 F), blood pressure is 90/60 mm Hg, pulse is 118/min and respirations are 26/min. His pulse oximetry reading is 94% at room air. The abdomen is mildly distended. Bowel sounds are absent. Rigidity and rebound tenderness are present. There is a gunshot wound in the left upper quadrant, with no exit orifice. A bedside chest x-ray reveals clear lung fields. Item 1 of 2 Which of the following is the most appropriate course of action? B. C. D. E. A. Perform a Diagnostic Peritoneal lavage (DPL) Perform a Focused abdominal sonogram (FAST) Order an Abdominal CT Scan with oral contrast Call the nearest surgical reference center to prepare for laparotomy Perform laparoscopy

This subscription is licensed to user: roopika only User ID: 123489 Explanation:

Approximately 85% of abdominal wall gunshot wounds (GSWs) penetrate the abdominal cavity, and approximately 95% will require surgery. This patient was injured with a long arm bullet (probably a rifle). The force of the impact is enough to assume penetration through the abdominal wall, unless there is evidence of only a superficial or tangential laceration during the physical examination. Furthermore, the presence of peritoneal signs in the patient is an absolute indication for an urgent laparotomy. Every effort must be made to transfer him immediately to the operating room. Other absolute indications for an urgent laparotomy in patients with acute abdominal trauma are: major vascular injury, hollow viscus perforation, hemodynamic compromise, hemo- or pneumoperitoneum, diaphragmatic lesions, and spinal cord injury. (Choices A, B, and C) There is no need to perform further ancillary procedures to investigate the extent of intra abdominal injuries in this patient because he already has an absolute indication for an urgent laparotomy. (Choice E) Laparoscopy is used to assess tangential abdominal GSWs. It is not recommended in the management of patients with abdominal GSWs with absolute indications for an urgent laparotomy. Educational Objective: Most abdominal gunshot wounds will require laparotomy for further exploration and immediate management of intra abdominal injuries, unless the wounds are only superficial or tangential to the skin. The absolute indications for an urgent laparotomy are: presence of peritoneal signs, spinal cord damage, intra abdominal vessels or bladder damage, and hollow viscus perforation. If the patient is hemodynamically unstable or has lost a great amount of blood, the procedure must be done immediately. 71% of people answered this question correctly

Item 2 of 2 The patient is reevaluated 20 minutes later. His abdominal symptoms are stable. His shortness of breath has improved after nebulizations, but he continues to have palpitations. He has received one and a half liters of normal saline solution. His blood pressure remains at 90/60 mm Hg, pulse is 100/min and respirations are 22/min. His pulse oximetry reading is 97% at room air. Examination reveals clear lung fields. Abdominal findings have not changed. Which of the following is the most appropriate immediate step in the management of this patient? A. Cross type at least 2 units of packed red blood cells B. Perform an abdominal CT scan with oral contrast C. Transfer the patient to the nearest surgical facility, which is 20 minutes apart by helicopter

D. E.

Observe and reevaluate clinically every two to four hours Explore the abdominal wound under local anesthesia

This subscription is licensed to user: roopika only User ID: 123489 Explanation: Even though the patient remains stable, he still needs an urgent laparotomy. If the procedure cannot be done because of lack of resources or an experienced surgeon, the patient must be transferred immediately to another facility. Most rural areas accomplish immediate patient transfers through air transportation. (Choice A) Although the patient may need a blood transfusion as indicated by the failure of his blood pressure to increase despite adequate intravenous hydration, the transfusion should not delay his transfer. Furthermore, most surgical centers consider blood transfusion in trauma patients when the systolic blood pressure is less or equal to 70 mmHg, because transfusing at higher levels of blood pressure has been shown to increase the rate of bleeding. For these reasons, the transfusion may be given to the patient in the operating room. (Choices B and D) There is no need to periodically observe the patient or perform further ancillary tests/procedures. He requires an urgent laparotomy. (Choice E) Exploration of the abdominal wound under local anesthesia is done when a superficial injury is suspected, and when the case of penetrating abdominal trauma is due to stabbing. Educational Objective: Early transportation can be life-saving for patients with penetrating abdominal trauma secondary to gunshots. Transfer to a regional trauma center is required if there is no surgeon or operating room available, if the patient has multiple injuries, or if he needs procedures that are not available (e.g., hemodialysis, organ transplantation, neurosurgery, etc.). Air transport (life flight) is recommended for patients who are potentially unstable, or who are in rural areas or places where ground transportation may take more than 20 minutes. 52% of people answered this question correctly

An 18-year-old Caucasian male comes for a follow-up visit two weeks after receiving conservative treatment for his fractured right clavicle. He has started lower body strengthening exercises and pendulum exercises of his right arm. He denies any pain during these activities. He is a long-distance runner. His past medical history is insignificant. He denies smoking, alcohol consumption, and any recreational drug use. His mother has accompanied him, and she expresses much concern about her son 'losing

his athletic shape.' She asks when her son will be able to return to his usual physical activities. Which of the following is the best response? B. C. D. E. A. 4-6 weeks 6 months 10-12 months About 2 years Never

This subscription is licensed to user: roopika only User ID: 123489 Explanation: Clavicle fractures are common injuries; these represent up to 10% of all fractures in adults and 35-45% of all fractures that occur in the shoulder girdle area. The criteria for return to sport activities are always controversial and essentially equate to a judgment call by the physician. Some of the most problematic factors associated with this decision are the parental demands and expectations. If there are no associated injuries, the athlete is allowed to continue lower body strengthening exercises. As soon as the pain resolves, the patient performs gentle pendulum exercises for shoulder motion and active range of motion of the elbow and hand. For noncontact athletes, a gradual return to usual activities may be allowed once the patient has: (1) painless, full, active range of motion, (2) nearnormal strength, and (3) evidence of bridging callus. These usually occur approximately 4 to 6 weeks after the injury in younger patients. (Choice B) For contact athletes, more time is required because the risk of re-fracture is greater. (Choice C) The normal process of bone remodeling takes approximately 10 to 12 months, but it is not acceptable for most athletes to be inactive for such a long period of time. Educational Objective: For the noncontact athlete with a clavicle fracture, a gradual return to usual activities may be allowed in approximately 4 to 6 weeks the after injury. 77% of people answered this question correctly

A 63-year-old African-American male presents for the evaluation of urinary urgency, hesitancy, nocturia, and a weak urinary stream. These symptoms have progressively worsened over the past year, despite being on doxazosin. He denies any fever, abdominal pain, hematuria, malaise, or weight loss. He is a non-smoker and non-alcoholic. Rectal examination reveals a smooth, firm enlargement of the prostate without any induration or asymmetry. Neurological examination is normal. His PSA level is elevated, and you explain the option of transurethral resection of the prostate. He is concerned about the

complications of the procedure. Which of the following is the most likely expected complication of this procedure? B. C. D. E. A. Retrograde ejaculation Urinary tract infection Urethral stricture Urinary incontinence Erectile dysfunction

This subscription is licensed to user: roopika only User ID: 123489 Explanation: Retrograde ejaculation is the most frequent complication of transurethral resection of the prostate (TURP), which is an invasive procedure used in the management of benign prostatic hyperplasia (BPH). (Choices B and C) Urinary tract infections and urethral strictures are also complications of TURP, but are less common. (Choices D and E) Urinary incontinence and erectile dysfunction are two common complications of radical prostatectomy, which is used for the treatment of early prostate cancer. These complications occur due to damages to the urethral sphincter and penile nerves during surgery. These complications do not occur with TURP. Educational Objective: Retrograde ejaculation is the most frequent complication of transurethral resection of the prostate, and all patients should be made aware of this. 60% of people answered this question correctly

You are visiting a 76-year-old Caucasian male in the nursing home. He has been complaining of pain in his anal region for the last three days, and he had an episode of a small amount of rectal bleeding. His other medical problems include constipation, benign prostatic hypertrophy, and chronic obstructive pulmonary disease. His temperature is 37.2 C (99 F), blood pressure is 110/60 mmHg, pulse is 100/min., and respirations are 20/min. Examination of the anal area reveals a thick protruding rectal mucosal mass with bluish discoloration and friability. He appears to be in moderate distress. Which of the following is the most appropriate next step in his management? B. C. D. E. A. Observation alone Observation with narcotic analgesics Send the patient to the emergency department Give antibiotics and schedule an outpatient follow up visit in 1 week Obtain surgical consultation in the nursing home

This subscription is licensed to user: roopika only User ID: 123489 Explanation: The patient in the above scenario has developed a complete rectal prolapse with strangulation and gangrene. He should be transferred to an emergency department as soon as possible to prevent further complications. Rectal prolapse occurs when a mucosal or full-thickness layer of rectal tissue slides through the anal orifice. It is usually associated with conditions causing increased intraabdominal pressure such as chronic straining with constipation, benign prostatic hypertrophy, and COPD. It usually presents with difficulty with defecation, diarrhea/fecal incontinence, pain in the anal area, rectal bleeding, and a protruding rectal mass. Untreated complete rectal prolapse can lead to strangulation and gangrene of the rectal mucosa. Uncomplicated prolapsed rectal mucosa can be reduced by gentle digital pressure. Adequate local anesthesia and sedation help reduce the prolapsed mucosa. Topical application of granulated sucrose helps reduce the mucosal edema and makes manual reduction easier. Irreducible prolapsed mucosa and gangrene of the mucosa warrant a prompt surgical consultation. The prognosis is generally good with prompt and appropriate care. (Choices A and B) Observation alone or with pain medications is not appropriate in this setting. The patient has developed gangrene of the mucosa that can be fatal unless treated emergently. (Choices D and E) The patient needs prompt treatment and possible emergent surgical intervention in the form of a rectosigmoidectomy. Outpatient medical management and surgical consultation at home are not recommended at this point. Educational Objective: Complete rectal prolapse should be promptly evaluated for signs of incarceration and gangrene. Surgical intervention is required for irreducible prolapse and gangrene of rectal mucosa. 49% of people answered this question correctly

An 84-year-old Caucasian woman is brought to the emergency department because of severe pain in her left hip and leg. She fell from the bed while sleeping. She has been bedridden for the past two years due to morbid obesity and advanced Alzheimer's dementia. Her other medical problems include hypertension, hypercholesterolemia, diabetes mellitus type 2, and coronary artery disease. She lives at home with her granddaughter. Her medications include aspirin, hydrochlorothiazide, enalapril,

donepezil, glimepiride and atorvastatin. Physical examination shows an obese, alert, mildly dehydrated elderly woman. Her temperature is 36.1 C (97 F), blood pressure is 170/85 mmHg, pulse is 100 /min and respirations are 18/min. The heart sounds are normal, and the lungs are clear. The abdomen is soft, non-tender and non-distended. Bowel sounds are normal. There is no rebound tenderness or rigidity. An ecchymotic area is present on the lateral aspect of the left thigh. Her left leg is rotated outwards, and looks shorter than the right one. Passive movement of the left lower extremity is extremely painful. An x-ray of the hip reveals a trochanteric fracture of the left femur. Her granddaughter is concerned about the fracture, and says that she is willing to sign consent forms so that her grandmother can undergo hip surgery to alleviate her pain. Which of the following is the most appropriate statement about the patient's condition? A. If surgery is not done, the pain will persist B. Surgery will not offer benefits in her case and can be dangerous C. If surgery is not performed, she may become more disabled and need nursing home placement D. Because of her advanced age and dementia, surgery will be an unnecessary risk E. Instead of surgery, external traction will be preferred This subscription is licensed to user: roopika only User ID: 123489 Explanation: The patient is not a good candidate for surgery. The type of surgery, the patient's age, multiple comorbidities, and severe disability places her at an extremely high risk for perioperative complications. The ideal candidates who may benefit from a surgical intervention are elderly patients who were ambulatory prior to their hip fracture. Since the patient is already bedridden, performing the surgical procedure will make no difference. The possibility of a successful rehabilitation and recovery is difficult to predict or evaluate before surgery because this heavily relies on the surgical outcome and the patient's response to the procedure. (Choice C) Although the decision to send the patient to a nursing home may be affected by her disability or need for surgical intervention, one of the main considerations is her current available social support. Since this patient's granddaughter seems to provide ample social support, there is no need to send her to a nursing home. (Choice D) Advanced age and dementia should not preclude surgical intervention if the procedure offers the opportunity to improve or at least partially restore function and the quality of life of the patient. (Choices A and E) Several studies have evaluated and discussed the role of conservative therapy in patients who are unable to undergo hip surgery. These have shown that hip pain due to a fracture is not an indication for surgery because conservative measures can

effectively control pain symptoms. On the other hand, systematic reviews have concluded that other forms of conservative therapy such as external traction may cause more pain. Educational Objective: Even in cases where the risk is high, only the absence of benefits may prompt the physician to refuse a procedure; therefore, a physician must always look at the entire clinical picture and prognosis before suggesting or refusing a surgical intervention. The patient's age, presence of comorbidities, functional status, quality of life, and personal values must all be considered to determine if the patient will benefit from hip surgery. If the patient is already bedridden, the benefit of surgery is minimal. If the patient has other serious medical problems, the risks may outweigh the benefits. Pain alone is not an indication for surgery, as conservative therapy can also effectively control this symptom. 24% of people answered this question correctly

A 47-year-old Caucasian man is playing tennis, when a thunderstorm occurs. He gets struck by lightning, and is thrown several feet away by the force of the bolt of lightning. Emergency medical services arrive shortly, find him in cardiac arrest, and begin cardiopulmonary resuscitation (CPR). The patient fails to respond to the CPR. Which of the following is the most appropriate intervention? B. C. D. E. A. Epinephrine Lidocaine Defibrillation Amiodarone Atropine

This subscription is licensed to user: roopika only User ID: 123489 Explanation: The majority of patients who develop cardiac arrest after an acute electrical injury are in asystole. Although a spontaneous return to sinus rhythm has been observed in certain cases, this is not a general rule; therefore, epinephrine is recommended in such cases to revert the asystole. At the same time, cardiopulmonary resuscitation must be performed for a prolonged period of time because these patients do not respond immediately to resuscitation measures. (Choices B and D) Lidocaine and amiodarone are not effective in the management of a patient with cardiac arrest after an acute electrical injury. (Choice C) Defibrillation will not be useful because the patient is probably in asystole. If the patient is not ventilated adequately, he may develop ventricular fibrillation after an initial response to a cardiac arrest.

(Choice E) Atropine can be used in the management of asystole secondary to a lightening injury if epinephrine is not effective. Educational Objective: Cardiac arrest due to lightening injuries usually presents as asystole; therefore, epinephrine is the first-line drug in such cases. 31% of people answered this question correctly An 88-year-old female is admitted in the hospital after undergoing open reduction and internal fixation of a hip fracture. She has no known recent cerebrovascular accident, arteriovenous malformation, hereditary telangiectasia, gastrointestinal ulcer, bleeding disorder, or chemotherapy. There is no evidence of any recent or intraoperative MI She has a history of hypertension and diabetes. Her temperature is 36.7C(98F), blood pressure is 160/86 mm Hg, pulse is 80/min, and respirations are 16/min. What is (are) the most appropriate measure(s) to prevent postoperative pulmonary embolism? A. Calf-length elastic stockings and early ambulation B. Subcutaneous low molecular weight heparin or adjusted-dose oral warfarin (goal INR 2.5) C. Intermittent pneumatic compression devices to both lower extremities D. Placement of an inferior vena cava filter E. Full dose IV unfractionated heparin to increase PTT to 1.5 to 2 times control This subscription is licensed to user: roopika only User ID: 123489 Explanation: According to the 2000 American College of Clinical Pharmacy (ACCP) consensus guidelines, patients undergoing surgery can be classified into four levels of risk for postoperative deep venous thrombosis (DVT), depending upon age, type of surgery, and presence of clinical risk factors for DVT. (See listing of risk factors below.) Low risk -- uncomplicated minor surgery in patients younger than 40 years with no clinical risk factors Moderate risk -- any surgery in patients aged 40-60 years with no additional risk factors; major surgery in patients younger than 40 years with no additional risk factors; minor surgery in patients with risk factors High risk -- major surgery in patients older than 60 years without additional risk factors or patients aged 40-60 years with additional risk factors; patients with MI; medical patients with risk factors

Highest risk -- major surgery in patients older than 40 years with prior venous thromboembolism, malignant disease, or hypercoagulable state; patients with elective major lower extremity orthopedic surgery, hip fracture, stroke, multiple trauma, or spinal cord injury Clinical risk factors for deep venous thrombosis include the following: Age (>40 y) Prolonged immobility (>4 h) or paralysis Prior DVT or PE Obesity Hypercoagulable states (e.g. factor V Leiden mutation, protein S deficiency, protein C deficiency, antithrombin III deficiency, antiphospholipid antibodies or lupus anticoagulant, plasma hyperhomocystinemia) Major surgery or fractures (especially abdominal, pelvic, lower extremities) Malignancy Varicose veins Heart failure Myocardial infarction (MI) This patient is therefore at the highest risk level for development of postoperative DVT. The modalities recommended to prevent such DVT, assuming prophylaxis is only begun postoperatively, are, in approximate (decreasing) order of effectiveness: Full-dose therapeutic IV heparin Either Adjusted-dose oral warfarin (goal INR 2.5) (or) Low dose heparin (unfractionated) 5000 U administered SC q8h to q12h (or) Adjusted (low dose) heparin (unfractionated)- 3500 U administered SC q8h adjust 500 U to maintain 1.5-2 times the reference range aPTT (or) Low molecular weight heparin SC [for example Enoxaparin (Lovenox) 3000 U (30 mg) SC q12h] Calf-length elastic stockings (ES) with early ambulation (or) Intermittent pneumatic compression devices (IPC) In general, highest or high-risk patients without contraindications require anticoagulation via modalities 1 or 2. Full dose therapeutic IV heparin is generally reserved for patients with MI. Warfarin is recommended mainly as a possible alternative to low dose or low molecular weight heparin for highest risk orthopedic patients undergoing lower extremity procedures. Modality 3 is adequate for low risk patients and some moderate risk patients, although most moderate risk patients should receive low dose or low molecular weight heparin. ES and/or IPC may be used adjunctively in higher risk patients. Since this patient is at highest risk and does not have an MI, the recommended prophylaxis would be modality 2. Modality 3 might be used adjunctively with, but not in place of, modality 2.

(Choice A and C) Elastic stockings and/or intermittent pneumatic compression devices could be used in addition to, but not in place of, anticoagulation in this patient, who is at the highest level of risk for postoperative deep venous thrombosis. Furthermore, early ambulation may not be feasible in an elderly patient after hip surgery. (Choice D) This describes a measure which may be recommended only if other forms of anticoagulant-based prophylaxis are not possible in high-risk orthopedic patients (i.e. active bleeding or one of the contraindications to anticoagulation: recent cerebrovascular accident, arteriovenous malformation, hereditary telangiectasia, gastrointestinal ulcer, bleeding disorder, or chemotherapy with bone marrow suppression). (Choice E) Full anticoagulation with the usual dose of unfractionated heparin is usually reserved for prophylactic perioperative anticoagulation in patients with a recent preoperative or intraoperative myocardial infarction. Educational Objective: Elderly individuals undergoing elective or emergent hip surgery are in the group of patients at highest risk for the development of postoperative deep venous thrombosis and pulmonary embolism. In the absence of contraindications, they should receive perioperative prophylactic anticoagulation. 51% of people answered this question correctly

A 58-year-old obese woman presents with a six-month history of intermittent, postprandial, colicky right upper quadrant abdominal pain. Each bout of pain lasts about 15 minutes. She takes propanolol and hydrochlorothiazide for primary hypertension. She has a 10 pack-year smoking history (quit 20 years ago), and drinks alcohol socially. Both her parents died of heart attacks. Her older brother is in good health. Her blood pressure is 130/90 mmHg, pulse is 62/min, temperature is 36.7C (98F) and respirations are 14/min. Palpation of the abdomen reveals pain on the right upper quadrant. Murphy's sign is present. Abdominal ultrasound shows several gallstones and biliary sludge. She undergoes an uncomplicated laparoscopic cholecystectomy, and is discharged the following day. On her first follow-up visit, she complains of diarrhea, with 4-5 watery bowel movements per day. What is the most appropriate next step in the management of this patient? B. C. D. E. A. Cholestyramine Dietary changes Antibiotics Subcutaneous octreotide Pancreatic enzyme supplements

This subscription is licensed to user: roopika only User ID: 123489 Explanation:

This patient has bile salt-induced diarrhea, which is a complication that occurs in 5-10% of cholecystectomies. Gallbladder surgery alters the gut dynamics. The increased bile acid flux to the colon induces a shift to the secondary bile acids, which are more likely to cause diarrhea. Cholestyramine is a bile-acid binding resin that sequesters the excess bile acids and is often effective against this form of diarrhea; therefore, it is the drug of choice for this patient's condition. (Choice B) Dietary changes are the first line of treatment for dumping syndrome. More frequent feedings containing less simple sugars provoke less gastrointestinal stimulation, thereby alleviating dumping symptoms. (Choice C) Antibiotics are indicated in bacterial overgrowth syndrome. (Choice D) The somatostatin analogue octreotide is useful in diarrhea caused by neuroendocrine tumors (e.g., somatostatinomas, VIPomas, etc.) and in some forms of AIDS-related diarrhea. (Choice E) Pancreatic enzyme supplements are useful in steatorrhea due to exocrine pancreatic insufficiency. Educational Objective: Bile salt-induced diarrhea is usually seen with short bowel syndrome. This complication also occurs in 5-10% of cholecystectomies. The treatment of choice for bile-salt induced diarrhea is bile-acid binding resins. 25% of people answered this question correctly;

A 19-year-old Caucasian male is brought to the emergency department (ED) after a major motor vehicle accident. He was an unrestrained driver. He confessed to consuming a moderate amount of alcohol several hours ago. He did not lose his consciousness. His blood pressure is 150/95 mmHg, and heart rate is 110/min. His respiration is shallow and rapid. He is able to move his extremities and obey commands. His pupils are equal and reactive to light. Inspection reveals anterior chest bruises and peripheral cyanosis. Breaths sounds are heard over the lower lung lobes. Neck vein distention is not present. Which of the following is the most likely diagnosis in this patient? B. C. D. E. A. Tension pneumothorax Massive hemothorax Cardiac tamponade Flail chest Cardiac contusion

This subscription is licensed to user: roopika only User ID: 123489 Explanation: The patient has tachypnea, shallow breathing, tachycardia, anterior chest bruises and peripheral cyanosis. The clinical scenario described is most consistent with flail chest. Flail chest is usually the result of double rib fractures in more than one site, and is present in 10-20% of trauma admissions. The main pathophysiologic feature of flail chest is the increased work of breathing due to muscular spasm and pain. Hypoxia develops frequently due to associated pulmonary contusions. (Choices A and B) Pneumothorax and massive hemothorax are typically associated with hypotension and a unilateral decrease in breath sounds. (Choice C) Cardiac tamponade is characterized by hypotension and neck vein distension. (Choice E) Cardiac contusion, if massive, may lead to hemodynamic instability; however, significant respiration changes (rapid shallow breathing) should not be present. Educational Objective: Flail chest is usually the result of double rib fractures in more than one site, and is present in 10-20% of trauma admissions. 39% of people answered this question correctly

A 24-year-old African American male presents to an urgent care clinic and complains of severe right knee pain and some difficulty in walking and pivoting, though he remains able to ambulate. Earlier that afternoon, he was playing football with friends and was tackled from the right side. Physical examination produces pain upon externally rotating the right tibia with the right knee bent at a 90-degree angle. Pain and moderate laxity is demonstrated with valgus stress testing of the right knee. A right knee joint effusion is present. Plain film of this knee is normal. What is your diagnosis? B. C. D. E. A. Anterior cruciate ligament injury Posterior cruciate ligament injury Medial collateral ligament injury Lateral collateral ligament injury Prepatellar bursitis

This subscription is licensed to user: roopika only User ID: 123489 Explanation: Medial collateral ligament injury is associated with tenderness and pain along the medial joint line. This common condition is most frequently caused by an injury involving

valgus (abductor) stress to the partially flexed knee with the foot fixed. Such an injury can occur while skiing or during contact sports, when another person falls across the knee from the lateral to medial direction. (Choice A) Anterior cruciate ligament injury is associated with pain and laxity upon performance of the anterior drawer or Lachman maneuver. This condition most frequently occurs after noncontact deceleration, a cutting movement, or hyperextension. It may be accompanied by a "popping" sound. (Choice B) Posterior cruciate ligament injury is associated with little pain or alteration in range of motion, but the posterior drawer test is positive. This uncommon condition occurs from a posteriorly directed force on a flexed knee (e.g., the dashboard being struck by the anterior of the flexed knee in a motor vehicle accident) or from hyperextension. (Choice D) Lateral collateral ligament injury is associated with tenderness and pain along the lateral joint line. This uncommon condition is most frequently caused by an injury involving dramatic varus (adductor) stress. (Choice E) Prepatellar bursitis is an inflammation of the largest knee bursa. This condition is most frequently caused by trauma from a fall or secondary to prolonged friction and pressure from repetitive kneeling ("housemaid's knee"). Educational Objective: Medial collateral ligament injuries commonly occur during skiing or contact sports, when a force is applied to the knee from the lateral to medial direction. 77% of people answered this question correctly;

A 7-year-old Caucasian girl is brought to the emergency department because of abdominal pain and bloody stools. Her past medical history is not significant. Her temperature is 37.2 C (99 F), and pulse is 108/min. Examination shows a soft, moderately tender, and slightly distended abdomen. Bowel sounds are increased, and tenderness is worse in the right lower quadrant. There is no rebound tenderness or rigidity. Rectal examination reveals bright red blood in the stools. A technetium-99 nuclear scan shows an increased uptake in the right lower quadrant. Which of the following is the most likely diagnosis? B. C. D. E. A. Intussusception Meckel's diverticulum Appendicitis Inflammatory bowel disease Intestinal lymphangiectasia

This subscription is licensed to user: roopika only User ID: 123489 Explanation:

Meckel's diverticulum usually affects children younger than 2-years of age but can also be found in older children and young adults. A technetium-99 nuclear scan will identify the diverticulum, which is usually located in the right lower quadrant, near the ileo-cecal valve. Technetium-99 will concentrate in the parietal cells of the diverticulum and stomach. This characteristic gives a high specificity to this test, which is also known as the "Meckel's scan". (Choice A) Approximately 80% of intussusception cases present before the age of two years. Also, technetium-99 nuclear scan will not be positive. (Choice E) Intestinal lymphangiectasia is an unusual condition. (Choice C) Appendicitis is generally not associated with intestinal bleeding or increased uptake in the nuclear scan. (Choice D) Inflammatory Bowel Disease (IBD) is more common in older children, and gives negative results for the Technetium uptake scan. Educational Objective: Acute abdominal pain with rectal bleeding in children between five and ten years of age can be due to Meckel's diverticulum bleeding, inflammatory bowel disease or appendicitis. A technetium-99 nuclear scan is highly specific for Meckel's diverticulum. 75% of people answered this question correctly

A 43-year-old Chinese-American man comes to the office. He is complaining of diarrhea, cramping abdominal pain, postprandial nausea and vomiting for the past five days. He also has dizziness, generalized sweating, and shortness of breath. He had a partial gastrectomy for a bleeding gastric ulcer one month ago. He has smoked one pack of cigarettes a day for twenty years, and quit two months ago. He does not use alcohol or drugs. The patient is a food handler. His medications are omeprazole and antacids. Examination reveals a mildly distended abdomen with increased bowel sounds. The epigastrium is mildly tender, and the postsurgical scar is healing well. Which of the following will be the most appropriate therapeutic intervention? B. C. D. E. A. Start metoclopramide. Start a high protein diet. Start a low-fiber diet. Start a high carbohydrate diet. Start alprazolam.

This subscription is licensed to user: roopika only User ID: 123489 Explanation:

The patient has dumping syndrome, a complication of gastrectomy. Liquid and food passage through the stomach into the jejunum is faster. This leads to abdominal pain, diarrhea, nausea and vomiting, as well as some neurovegetative symptoms such as dizziness, generalized sweating, and dyspnea. A change in the diet usually relieves the symptoms. A high protein diet, and fractionated, smaller, but more frequent food portions are advised. (Choice D) The diet should be low (not rich) in carbohydrates. (Choice A) Metoclopramide stimulates the contraction of the lower esophageal sphincter and increases gastric emptying. It may worsen the symptoms. (Choice C) A low-fiber diet may improve the diarrhea, but will not control the other features of dumping syndrome. (Choice E) Alprazolam is helpful for neurovegetative (not digestive) symptoms. Educational Objective: Dumping syndrome is a common complication of gastrectomy. Treatment is aimed at decreasing the speed of the passage of fluids and food into the small gut. A high-protein and low-carbohydrate diet is advised, as well as smaller but more frequent meals throughout the day. 29% of people answered this question correctly;

A 34-year-old diabetic woman undergoes emergency laparoscopic cholecystectomy for acute cholecystitis. On the third post-operative day, she has a fever of 38.0 C (100.3 F). The site of the intravenous cannula in the cephalic vein is erythematous and tender. A cord is palpated at this site. The cannula is removed, and treatment with nafcillin is started. On the sixth post-operative day, the patient's fever persists. Erythema and tenderness of the same site have increased. Auscultation of the lungs reveals clear breath sounds; the abdomen is quiet. Two blood cultures are positive for Staphylococcus epidermidis , which is susceptible to naficillin. Urine cultures are negative. The abdominal ultrasound is negative for fluid collections. What is the most appropriate next step in the management of this patient? B. C. D. E. F. A. Increase the dose of naficillin to her maximal Chest x-ray Abdominal CT scan with contrast Needle aspiration of the affected area of the cephalic vein Add intravenous heparin to the current treatment Repeat blood cultures

This subscription is licensed to user: roopika only User ID: 123489 Explanation: This patient is persistently febrile, despite using susceptible antibiotics. This patient may have a suppurative superficial thrombophlebitis (collection of pus in the vein). The cephalic vein has to be explored and drained of any pus found. (Choice A) Antibiotic therapy alone would not cure this condition unless the involved vein is drained of pus. (Choice E) In addition to antibiotics, heparin is used for treatment of deep thrombophlebitis, most notably for phlebitis of the pelvic veins as an obstetric complication. Because of the minimal risk of pulmonary thromboembolism due to superficial thrombophlebitis, heparin is rarely used for this condition. (Choices B, C and F) Post-operative fever can be due to pneumonia, lung abscess, or intraabdominal abscess. These possibilities must be checked if it is proven that the site of the infection is not the cephalic vein. Educational Objective: Superficial suppurative thrombophlebitis requires drainage if the antibiotic therapy alone is ineffective. 23% of people answered this question correctly A 37-year-old Caucasian construction worker arrives at the emergency department by ambulance twenty minutes after being hit by a car while crossing the street. At the scene of the accident, he was alert, awake, and in considerable distress. On initial assessment in the emergency department, his temperature was 36.7C (98F), blood pressure was 104/60 mm Hg, pulse was 108/min, and respirations were 31/min. His abdomen was found to be tender and he complained of severe diffuse abdominal pain. Chest radiograph showed bilateral pulmonary contusions. Radiographs of the pelvis and cervical, thoracic, and lumbar areas of the spine revealed no fractures. An abdominal CT showed a splenic laceration and free fluid in the abdominal cavity. His medical history is significant for a history of addiction to opiates (OxyContin). After completing a drug rehabilitation program eight months ago, he completely stopped using opiates. He smokes one pack of cigarettes per day and does not drink alcohol. He takes no medications. He continues to be in great pain and asks for relief. Given the circumstances, which of the following is the most appropriate therapy? B. C. D. E. A. Acupuncture and cold packs Hypnotherapy Trans-cutaneous electrical neural stimulation Ketorolac IV Morphine sulfate IV

This subscription is licensed to user: roopika only User ID: 123489 Explanation: Patients in recovery from addiction to narcotics are often hesitant to accept treatment with morphine or other potentially addictive medications, even during times of severe pain. However, there are some situations that necessitate the administration of narcotics for adequate pain control, regardless of addiction history. Severe traumatic injury is one such situation. In this case, therefore, the patient should be given morphine sulfate (Choice E) as the other options will provide inadequate pain relief. Acupuncture, cold packs, hypnotherapy, trans-cutaneous electrical neural stimulation, and NSAIDs (Choices A, B, C, and D) are alternative treatment options that may provide relief of mild or moderate pain. They are not appropriate therapies for severe traumatic injuries. Educational Objective: An individual who suffers severe traumatic injury should be given narcotics for pain relief, regardless of addiction history. 59% of people answered this question correctly

A 32-year-old white female with no significant past medical history comes to the office because she has noticed a small lump in her left breast. She is married, has no children, and works as an attorney in a very busy local bureau. She does not use tobacco, alcohol, or drugs. There is no history of breast cancer in the family. Her last menstrual period was five days ago. On physical examination, you find a 1.5 cm round, smooth, soft, mobile, mildly tender mass in the left breast. No axillary nodes, skin lesions, or nipple discharge are found. Which of the following is the most appropriate next step in her management? B. C. D. E. A. Fine needle aspiration of the lesion Breast ultrasound Mammogram Reevaluation three weeks from now Order serum BRCA1 and 2

This subscription is licensed to user: roopika only User ID: 123489 Explanation: Female patients younger than 35 years have a decreased risk of breast cancer, especially if there is no positive family history. The patient is a busy woman who needs a fast answer to her problem. At the same time, the characteristics of her breast mass - smooth,

soft, mobile, and round - usually correspond to a breast cyst, which may easily be assessed through fine needle aspiration (FNA). (Choice C) A mammogram is not usually recommended in patients less than 35 years of age because the breast tissue at this age is too dense to allow good imaging. (Choice B) If the mass has no cystic characteristics, or if the patient refuses FNA, a breast ultrasound is indicated. (Choice D) Benign lesions are supposed to decrease in size 3 to 10 days after menstruation, not three weeks later. Since the patient came five days after her menstrual period, reevaluation of the lesion will not be useful. (Choice E) BRCA tests are not advisable. Educational Objective: Even though breast cancer is rare in women younger than 35 years, every breast lesion must be taken seriously. Patients can be reevaluated 3 to 10 days after the menstrual period to look for regression. If the lesion appears cystic, FNA should be done unless the patient declines. If the mass looks solid, is too small, or cannot be felt, ultrasound is the next step to determine if biopsy is needed. 37% of people answered this question correctly A 56-year-old Caucasian female comes to the emergency department with complaints of a sudden onset of severe pain in the left side of her abdomen. It is accompanied by nausea and two episodes of vomiting. The pain waxes and wanes in intensity, and she has been unable to find a comfortable position. An initial CT scan of the abdomen reveals a 5 mm stone present in the left upper ureter. She is admitted to the hospital and is treated with intravenous hydration and pain control. After 30 hours of conservative management, she continues to have severe pain. Her kidney function is within normal limits. Which of the following is the most appropriate next step in the management of this patient? B. C. D. E. A. Continue observation and treatment. Start her on intravenous antibiotics. Refer her for extracorporeal shockwave lithotripsy. Refer her for flexible ureteroscopic removal of the stone. Refer her for percutaneous ureterolithotomy.

This subscription is licensed to user: roopika only User ID: 123489 Explanation: Renal or ureteral stones are a common presenting problem in the emergency department. Most of the patients present with a sudden onset of pain and hematuria. The pain is usually colicky in nature (waxing and waning) and can vary from a mild ache to severe

discomfort requiring narcotic analgesics. The location of the pain can sometimes give clues to the site of the stone. Upper ureteral or renal stones usually cause pain in the flank, whereas a lower or distal ureteral stone causes pain which radiates to the ipsilateral groin area. Initial management is usually conservative, and includes intravenous hydration and pain control. Stones that are less than 5 mm in size usually pass spontaneously. On the other hand, stones greater than 8 to 10 mm are unlikely to pass spontaneously and require removal. Stone removal is also indicated in patients with persistent pain, acute renal failure, or signs of urosepsis. There are three common techniques that have been employed to facilitate stone removal or passage from the ureters. These include extracorporeal shockwave lithotripsy (ESWL), flexible ureteroscopy, and percutaneous ureterolithotomy. The choice of procedure usually depends on the location of the ureteral stone. Shockwave lithotripsy is the treatment of choice for small symptomatic proximal ureteral calculi (less than 5 - 10 mm in size). For large (more than 10 mm) proximal ureteral stones, flexible ureteroscopy combined with laser lithotripsy is the preferred initial therapy. (Choice A) Constant or persistent pain after conservative treatment is an indication for stone removal. (Choice B) There is no indication for using intravenous antibiotics in this patient with an uncomplicated ureteral stone. (Choice D) Ureteroscopic removable of calculi is performed in patients with large and proximal (more than 10 mm) ureteral calculi and/or distal ureteral stones. It is also useful in the management of ureteral calculi after failed therapy with shockwave lithotripsy. (Choice E) A percutaneous approach for stone removal is only used when shockwave lithotripsy and ureteroscopic removal fail to remove the ureteral stones. Educational Objective: Stones that are less than 5 mm in size usually pass spontaneously. Shockwave lithotripsy is the preferred initial therapy for small symptomatic proximal ureteral calculi (less than 5 to 10 mm). Stone removal is indicated in patients with stones greater than 8 to 10 mm (since these are unlikely to pass spontaneously), persistent pain, acute renal failure, or signs of urosepsis. 54% of people answered this question correctly A 24-year-old man is brought to the emergency department (ED) by the police after he was arrested for fighting in the streets. During the fight, he was struck with a knife on his head, and the knife was firmly implanted in his skull. The estimated length of the knife is 14 cm, of which 3 cm has penetrated his skull. His vital signs are within normal limits. An initial physical examination reveals that he is minimally responsive to verbal or tactile

stimuli. There is no other evidence of injury on his body. While in the ED, which of the following is the most important next step in the management of this patient? B. C. D. A. Remove the knife from his skull Irrigate the wound with normal saline Obtain a stat PT, PTT, blood group, and crossmatch Obtain a CT scan of the head to assess the damage

This subscription is licensed to user: roopika only User ID: 123489 Explanation: The patient has presented to the emergency department with a penetrating injury to his skull. Maintenance of an adequate airway and hemodynamic stability are the first steps in acute management. (This is true for all patients with a penetrating injury or trauma to any body part.) Even if he has normal hemodynamic parameters at this point, he is still at high risk for subsequent deterioration secondary to acute blood loss within the next few hours, especially during the process of removal of the knife. This is further compounded by the lack of availability of any pertinent medical history due to his mental status. In conclusion, all such patients should receive immediate laboratory studies to identify any reversible coagulopathy that may increase the risk of bleeding. This also includes blood typing and crossmatching, in case there is an urgent need for a blood transfusion during the course of his treatment and hospitalization. (Choice A) Even though immediate removal of the knife seems appropriate, it should not be attempted without obtaining any prior information. Removal of the knife may lead to an increase in the bleeding due to opening up of the dural venous sinuses. This should be attempted in the operating room. (Choice B) Local wound management is not an emergency, and can be done at a later stage. (Choice D) It is important to ensure the patient's hemodynamic stability before proceeding with any further radiographic imaging. Educational Objective: Maintenance of adequate hemodynamic stability is the most important step in the management of a patient with a penetrating injury. Always attempt to identify and correct any reversible cause of coagulopathy, and anticipate the need for blood transfusions in such patients. 42% of people answered this question correctly; A 24-year-old Caucasian male was involved in a major motor vehicle accident. The ambulance service that arrived several minutes after the accident found him dead. He was

found in the driver's seat, and the steering wheel in front of him was broken. Which of the following is the most likely cause of his death? B. C. D. E. A. Cardiac contusion Tension pneumothorax Aortic rupture Stomach rupture Cervical spine injury

This subscription is licensed to user: roopika only User ID: 123489 Explanation: Aortic injury is the most common cause of sudden death due to steering wheel injuries. Patients typically die at the scene before an ambulance service arrives. Rapid deceleration produces a shearing force along the aortic arch where the aorta is firmly attached. It is usually observed in the area of the ligamentum arteriosum, the aortic root, and the diaphragmatic hiatus. The remaining choices on the list are either less common causes of sudden death after a motor vehicle accident, or conditions that do not produce sudden death. (Choice B) Tension pneumothorax is a life-threatening condition that may result in cardiovascular collapse; however, the resulting death is not usually instantaneous. (Choice D) Abdominal injury may result in profound shock if rapid internal bleeding occurs. (Choice A) Cardiac contusion produces sudden death when the injury involves the cardiac chambers or vessels. (Choice E) Cervical spine injury leads to quadriparesis and sometimes respiratory paralysis. Educational Objective: The most common cause of sudden death after a steering wheel injury from motor vehicle accident is aortic injury. 71% of people answered this question correctly

A 22-year-old African-American man comes to the emergency department because he sustained an animal bite in the local zoo. He claims that he was playing with the animal with a few other friends, when the animal suddenly attacked him. He has a small 1 x 1 cm laceration on the dorsum of his right hand; there is no loss or limitation of function. His last tetanus injection was approximately two years ago. Which of the following is the

most important information to obtain before proceeding with the management of this patient? B. C. D. room E. The presence of any endemic diseases in the zoo This subscription is licensed to user: roopika only User ID: 123489 Explanation: An animal bite is an extremely common problem encountered in the emergency department. Most of the bites involve an animal known to the victim, and are unprovoked. Sometimes, the bites can be from stray and wild animals as well. It is important to obtain an accurate and complete history from the patient for appropriate management. All the above queries are important in the initial history; however, the most important piece of information is the exact location of the animal in the zoo. The bite in the zoo should not be presumed to be from the animals kept under observation. The involved animal could just have been within the surroundings, and not kept under observation; therefore, the animal may not have been vaccinated. This can change the whole management strategy, since a more aggressive approach will be warranted in such a situation. Educational Objective: The animal involved in the bite should be traced and ideally kept under observation to look for the development of symptoms in the animal. 14% of people answered this question correctly TEST 2 A 6-year-old Caucasian boy is brought to the emergency department by his family. He was playing with their family dog in the backyard of their home, when the dog suddenly attacked the boy and bit his left hand. His father immediately washed the wound with copious amounts of water and brought him to the emergency department. On physical examination, there is a 1 cm x 1 cm laceration seen over the dorsum of his left hand. There is minimal amount of serous drainage from the wound. X-ray of the left hand does not reveal the presence of any foreign body in the wound. What should be done for wound closure in this patient? A. Close the wound now with nylon sutures. A. The exact location of the animal in the zoo The kind of animal involved in the incident The duration of the bite The time lapse between the bite and his presentation to the emergency

B. C. D. E.

Leave the wound open to drain and observe closely. Close the wound with Vicryl after ensuring adequate saline irrigation. Do not close the wound with sutures, but put a nylon tape over the wound. Close the wound with a Dacron patch.

This subscription is licensed to user: roopika only User ID: 123489 Explanation: Animal bites, especially dog and cat bites, are common presenting problems in the emergency department. They are frequently caused by animals known to the victim. Animal bites can lead to a wide spectrum of injuries, ranging from minor scratches or abrasions to deep puncture wounds or lacerations. The extremities, especially the upper extremities, are common sites of injury in older children or adult patients. All animal bites, regardless of the site, should be thoroughly cleaned and irrigated with normal saline, and all devitalized tissues should be debrided. A plain radiograph should be obtained if a patient is suspected of having a foreign body or has a bite occurring close to a bone. Most open lacerations can be closed primarily within a few hours of injury. This is especially true for injuries of the face where infections are less common due to a good vascular supply; however, bites involving the hands should not be sutured or closed primarily due to a high risk of subsequent wound infection. These should be left open to drain and examined frequently for signs of infection. Other situations where primary closure is not recommended include puncture wounds, cat and human bites (high risk of infection), and patients presenting late after the bite. (Choices A, C, D, and E) Primary wound closure with nylon, Vicryl, or covering the wound with tape is not recommended in patients with bite wounds over the hands. These wounds should be left open to drain and examined closely for signs of infection. Educational Objective: Dog bite injuries of the hand and puncture wounds anywhere on the body should not be closed primarily due to a high risk of development of wound infection. 61% of people answered this question correctly

A 22-year-old white male who underwent a laparotomy two weeks ago for a perforated appendix, develops a swinging fever, dry cough, and pain in his right shoulder-tip. His temperature is 38.9 C (102 F), blood pressure is 120/76 mmHg, pulse is 110/min, and respirations are 20/min. Examination shows tenderness over the 8th to 11th rib. Abdominal and rectal examinations are within normal limits. The WBC count is 14,000/micro-L, with 6% bands. Which of the following is the best diagnostic test for this patient?

B. C. D.

A. Abdominal ultrasound Chest x-ray Abdominal x-ray Liver biopsy

This subscription is licensed to user: roopika only User ID: 123489 Explanation: The above patient is most likely suffering from a right subphrenic abscess, which typically develops 14-21 days after abdominal surgery. Clues to this diagnosis are a history of abdominal surgery, swinging fever, and leukocytosis. Cough and shoulder-tip pain may be the presenting symptoms. Of all the mentioned choices, abdominal ultrasound is the best diagnostic test. It has an accuracy of greater than 90%, and has good specificity when correlated with clinical findings. (Choice C) Abdominal x-ray findings (e.g., ileus, air-fluid levels, gas outside the lumen, or mottled soft tissue) may suggest the need for further evaluations, but are not diagnostic of a subphrenic abscess. (Choice B) Chest x-ray may show nonspecific findings (e.g., pleural effusion, atelectasis, basal infiltrates, or elevated hemidiaphragm) which are not diagnostic of a subphrenic abscess. (Choice D) Liver biopsy is not the appropriate choice. Educational Objective: Abdominal ultrasound is the best diagnostic test for a suspected subphrenic or other abdominal abscess. 60% of people answered this question correctly

A 32-year-old male was admitted to the orthopedic department after being involved in a motor vehicle accident. His initial injuries included a transverse fracture of the right femur, tibia, fibula, and a comminuted fracture of the left femur shaft. The secondary survey revealed no other fracture or injuries. Thirty-six hours after admission, the patient complains of shortness of breath. His blood pressure is 110/80 mmHg, heart rate is 80/min, respirations are 34/min, and temperature is 37.9 C (100.1 F). His oxygen saturation on 3L per nasal cannula is 88%. Physical examination reveals a petechial rash on his head, neck, anterior thorax, and axilla. He appears confused. The portable chest xray result reveals patchy consolidation. Emergency V/Q scan reveals a mottled pattern of sub-segmental perfusion defects with normal ventilation. Which of the following would have prevented this complication in this patient?

A. fracture B. C. D. E.

Early immobilization and operative correction of the

Low molecular weight heparin Breathing exercises and chest physiotherapy Sequential compressing devices for lower limbs Prophylactic antibiotics

This subscription is licensed to user: roopika only User ID: 123489 Explanation: Fat embolism is a clinical diagnosis. It typically manifests 24-72 hours after severe trauma, and is usually characterized by a triad of respiratory insufficiency, neurological impairment, and a petechial rash. Other common findings include fever, tachycardia, and altered mental status. The classic petechial rash is most commonly found on the trunk; this is typically a late manifestation, and is described in fewer than 50% of cases. Petechiae result from the occlusion of dermal capillaries by fat globules, thereby leading to extravasation of erythrocytes. No abnormalities of platelet function have been documented. Early immobilization and operative fixation of fractures reduces the chances of fat embolism. Supportive care is the mainstay of therapy for clinically apparent fat embolism. (Choice B) There is no evidence that prophylactic heparin reduces the risk of fat embolism. The use of corticosteroid prophylaxis is controversial, largely because it is difficult to definitively prove its efficacy in a condition with a low incidence, unclear risk factors, low mortality, and a good outcome with conservative management. Educational Objective: Always consider the risk of fat embolism in patients with multiple complicated fractures. Fat embolism is a clinical diagnosis that is usually characterized by a triad of respiratory insufficiency, neurological impairment, and a petechial rash. Early immobilization and operative fixation of fractures reduce the chances of fat embolism. 48% of people answered this question correctly;

A 38-year-old man comes to the office and complains of right testicular pain. For the past week, he has had dysuria, which is improving. The patient has no other medical problems. He is currently not taking any medications. He denies the use of tobacco, alcohol or illicit drugs. He occasionally has unprotected sex with prostitutes, and had gonococcal urethritis 6 years ago. Examination reveals tenderness on palpation of the right testicle, with an apparent increase in size, and a soft, small (2 x 2 cm), but very tender mass adjacent to it. There is no urethral discharge. Which of the following will be the most appropriate next step in the management of this patient?

B. C. D. E.

A. Immediate referral to the urologist Order a testicular sonogram. Start oral ciprofloxacin. Start ibuprofen. Perform transillumination of the testicles.

This subscription is licensed to user: roopika only User ID: 123489 Explanation: This patient has symptoms of epididymitis. The small, tender mass is the inflamed or infected epididymis. Transillumination of the testicle is an easily performed office procedure, and will be very useful in this case since epididymitis is usually associated with a unilateral hydrocele. (Choice B) A sonogram should be performed if transillumination fails to show a hydrocele. (Choice A) After the abovementioned procedures are done, the patient can then be referred to the urologist. (Choice D) Ibuprofen will alleviate the pain, but is only an adjuvant therapy. (Choice C) Treatment of epididymitis is done with ceftriaxone and doxycycline because Chlamydia trachomatis and Neisseria gonorrhoeae are the usual pathogens involved. Ciprofloxacin is not indicated. In any case, antibiotic therapy must not be started until transillumination is done, and the possibility of a malignancy is ruled out. Educational Objective: Epididymitis is an infection characterized by testicular pain and tenderness, hydrocele, and a tender, swollen epididymis. When there is an apparent increase in testicular size, transillumination of the testicle can be done to determine the presence of a hydrocele or malignancy. 30% of people answered this question correctly;

A 16-year-old boy presents to the emergency room half an hour after he received a kick on his scrotum while playing soccer. The patient complains of extreme pain. There is no hematuria or other urinary signs. On examination, you note swelling and bruising of the scrotum, most prominent on the right side. No open wound or skin loss is noted. Transillumination test is negative. Abdominal, penile, and thigh examinations are without anomalies. You suspect testicular hematoma. Which of the following is the most appropriate next step in the management of this patient?

B. C. D.

A. Ultrasound Needle aspiration Surgical exploration Watchful waiting

This subscription is licensed to user: roopika only User ID: 123489 Explanation: The utility of ultrasound (Choice A) in scrotal trauma is controversial. Some studies report excellent sensitivity, while others do not recommend it because of its unreliability. In any case, a sonogram is a useful adjunct to physical examination, but normal results should not preclude surgical exploration when the testis is grossly abnormal. Several studies have demonstrated a significant increase in orchiectomy rates when surgical repair was delayed. Surgical exploration and repair should therefore be performed without delay if there is evidence of significant trauma on physical examination. Educational Objective: Surgical exploration should be performed immediately in scrotal trauma cases wherein there is evidence of significant trauma on physical examination. 16% of people answered this question correctly

A 75-year-old female is brought to the emergency room after she fell in her bathroom. She is unable to walk or bear weight on her right leg. Her past medical history is significant for hypertension, coronary artery disease, myocardial infarction, congestive heart failure and osteoporosis. On examination, you note that her right leg is externally rotated, and all the movements involving the right hip joint are very painful. The x-ray of the right hip shows a fracture in the neck of the femur. She is scheduled for right hip replacement. Which of the following is the most appropriate course of action to prevent deep venous thrombosis in this patient? B. C. D. E. A. Aspirin alone Warfarin alone Pressure stockings Low molecular weight heparin Inferior vena cava filter

This subscription is licensed to user: roopika only User ID: 123489 Explanation: Approximately 250,000 hip fractures occur every year in the United States. Development of venous thromboembolism in these patients is one of the major causes of postoperative morbidity and mortality. 4-7% of DVT patients suffer from fatal pulmonary embolism.

Because of this, thromboembolic prophylaxis has been recommended for these patients. Studies comparing the prophylactic effects of aspirin, coumadin and unfractionated or low molecular weight heparin (LMWH) have shown that the LMWH group has superior results when compared with the other two groups. LMWH is therefore considered to be the therapy of choice, and should be given unless contraindications for its use are present. (Choice B) Coumadin is considered a second line prophylactic agent. It can be used as first line if any contraindication to the use of LMWH exists. Administration of coumadin should be started on the day of admission, and the target INR of 2 to 3 is achieved for adequate prophylaxis. (Choice C) Pressure stockings and intermittent pneumatic compression are used as adjuvants to other therapies. They are not sufficient to prevent DVT as sole agents because they are only effective in preventing DVT formation in calf veins, and have no prophylactic effect on pelvic vein thrombosis. (Choice A) Aspirin is used as a third line agent after heparin and coumadin. It is recommended as a sole prophylactic agent against thromboembolism only if both heparin and coumadin (warfarin) are contraindicated. (Choice E) Inferior vena cava filter placement is indicated in patients who have contraindications to anti-coagulation. It is also used in patients who develop DVT despite ongoing anticoagulation treatment. Educational Objective: LMWH is considered to be the prophylactic therapy of choice for preventing deep vein thrombosis in patients at high risk. 81% of people answered this question correctly;

A 22-year-old Caucasian man comes to the physician because of a painless mass in his right testes. He has no other medical problems. He denies the use of tobacco or alcohol. His family history is not significant. He is currently taking no medications. His vital signs are within normal limits. Examination shows a small, firm and painless mass in his right testes. The transillumination test is negative. There is no inguinal lymphadenopathy. Which of the following is the most appropriate next step in the management of this patient? B. C. D. A. Treat with antibiotics and reevaluate in 2 weeks. Do a CT scan of abdomen and pelvis. Reassure him and advise him a follow-up visit after 6 weeks. Obtain urology consult.

This subscription is licensed to user: roopika only User ID: 123489 Explanation:

A painless testicular mass should always raise the suspicion of malignancy, and the patient should be evaluated with scrotal ultrasound and measurement of serum tumor markers. Thereafter, he should be referred to urology for a radical inguinal orchiectomy, which is the procedure of choice. This procedure will also permit histological analysis for the diagnosis of the tumor. A scrotal biopsy should not be attempted. Scrotal ultrasound and serum markers are not sufficient to replace radical inguinal orchiectomy for the diagnosis of testicular cancer. (Choice C) A painless testicular mass is very likely to be malignant. Any unnecessary delays in the diagnosis and management could result in fatal outcomes. (Choice B) A CT scan of the abdomen and pelvis is performed to detect any metastasis of the tumor, but this does not provide a histological diagnosis of the tumor; therefore, this is not the first step. (Choice A) Antibiotics are used for the treatment of epididymo-orchitis, an extremely painful condition which is very unlikely in the above patient. Educational Objective: A painless testicular mass should always raise the suspicion of malignancy, and the patient should be sent for urology consult for a radical inguinal orchiectomy. 47% of people answered this question correctly;

A 7-month-old Caucasian boy is brought to your office for a routine check-up. The child was born at term and the prenatal course was uncomplicated. Evaluation reveals a leftsided undescended testis that can be palpated in the suprapubic area. Physical examination is normal otherwise. Which of the following is the best statement concerning surgery (orchiopexy) in this patient? A. years of age B. C. D. E. The testis will most probably descend spontaneously by 2

Orchiopexy may increase the risk of testicular torsion Orchiopexy does not affect the risk of infertility The risk of malignancy remains increased after orchiopexy Orchiopexy should ideally performed between 2 and 4 years of age

This subscription is licensed to user: roopika only User ID: 123489 Explanation: Cryptorchidism (undescended testis) is a common problem. It affects 4% of newborn infants, but the majority of cases resolve spontaneously during the first several months of

life. Although the risk of malignant transformation may decrease a little after the surgery, it remains higher than that of the general population. At the same time, the surgery makes it easier to examine the testis. (Choice A) Spontaneous descent is rare after six months of age. (Choices B and C) The complications of undescended testis include decreased fertility, increased risk of testicular torsion, inguinal hernia, and malignant transformation. Early orchiopexy helps to prevent testicular torsion and infertility. (Choice E) Orchiopexy can be performed as early as six months of age, but definitely before two years of age. Educational Objective: Early orchiopexy helps to prevent testicular torsion and infertility. Although the risk of malignant transformation may decrease a little after the surgery, it remains higher than that of the general population. 48% of people answered this question correctly

A 48-year-old Hispanic woman with no past medical history is brought to the emergency department by an ambulance after being rescued from a burning apartment building. The woman was trapped in a smoke-filled back bedroom of the apartment. On physical exam, her vital signs are stable, and while there is some singing of her facial hair, there is no visible evidence of burns. An arterial blood gas is obtained, and the values are as follows: pH: 7.39 PaCO2: 42 mm Hg PaO2: 91 mm Hg HCO3-: 24 mEq/L What potential development is of most concern in this woman over the next 24 hours? B. C. D. E. A. Night terrors Acute stress disorder Sepsis Supraglottic edema Cardiac arrhythmias

This subscription is licensed to user: roopika only User ID: 123489 Explanation: The majority of burn center deaths are due to inhalation injuries. Supraglottic damage is one of the most alarming types of inhalation injuries and stems from inhaling hot air,

steam, or smoke. The ensuing inflammatory response results in edema of surrounding soft tissues and significant narrowing of the airway. Fiberoptic laryngoscopy or bronchoscopy may be necessary to properly assess the extent of airway involvement. In patients such as this woman, the clinician must maintain a high index of suspicion for airway injury and a correspondingly low threshold for intubation. (Choice A) Night terrors are episodes of sleep disturbance that are typically found in children aged 3-12 years, and are not of concern in this woman. (Choice B) Acute stress disorder is a psychiatric condition that occurs when an individual feels intense fear, horror, or helplessness during a traumatic event that involves actual or threatened death, or serious injury. Subsequently, the individual exhibits dissociative symptoms, but continues to be functionally impaired by re-experiencing the traumatic event. Unlike the more chronic posttraumatic stress disorder, acute stress disorder lasts a maximum of 4 weeks after the traumatic event. While this woman is certainly at risk for developing either acute stress disorder or posttraumatic stress disorder, neither condition would put her in immediate danger within the next 24 hours. (Choice C) Sepsis would be of grave concern if this woman had experienced substantial burns to her body that disrupted the skin's protective barrier. Since no such burns are evident in her physical examination, the potential consequences of inhalation injury are much more ominous. (Choice E) The development of cardiac arrhythmias is indeed potentially lethal in any individual deprived of oxygen for a lengthy period; however, since this woman has normal vital signs and normal arterial blood gas values, it appears unlikely that her tissues were subjected to hypoxic stress great enough to induce life-threatening arrhythmias in the next 24 hours. Educational Objective: Suspect airway injury and edema in any individual who has been exposed to smoke or superheated air. Remember the ABCs (airway, breathing, circulation) and intubate early if the airway is in danger. 74% of people answered this question correctly;

A 62-year-old Caucasian female comes to the emergency department with a complaint of severe upper abdominal pain. She has had episodes of nausea and upper abdominal pain over the past three months, but the pain was tolerable and only lasted for 1 to 2 hours. Last night, she had a similar episode one hour after dinner, but the pain progressed in severity, and was accompanited by three episodes of vomiting. The pain is now maximally located in the right upper quadrant and radiates to her back. She denies any diarrhea or constipation. She is a moderately obese female who is in marked distress due to the pain. Her temperature is 38.3C (101F), blood pressure is 118/87 mmHg, heart rate is 112/min and respirations are 20/min. Her lung and cardiovascular examination is

unremarkable. Her abdominal examination reveals the presence of hypoactive bowel sounds. There is marked tenderness on deep palpation of the right upper quadrant. She has voluntary and involuntary abdominal guarding. Murphy's sign is positive. The initial blood work reveals the presence of marked leukocytosis with a left shift. The results of the liver function tests are pending. Item 1 of 2 What is the next best step in the management of this patient? B. C. D. E. A. Plain x-ray of the abdomen Order a right upper quadrant ultrasound Obtain a CT scan of the abdomen with IV contrast Wait for the liver function tests Consult GI for an endoscopic retrograde cholangiopancreatography

This subscription is licensed to user: roopika only User ID: 123489 Explanation: The clinical presentation of this patient is consistent with acute cholecystitis. Acute cholecystitis is defined as gallbladder inflammation characterized by steady right upper quadrant pain, fever and leukocytosis. It is generally caused by gallstone disease; however, acalculus cholecystitis can also occur especially in elderly and critically ill patients. Patients with acute cholecystitis typically complain of a steady and severe upper abdominal or right upper quadrant pain. The pain may radiate to the back or right shoulder, and is generally associated with fever, nausea, vomiting and anorexia. The patient may give a history of intolerance to fatty foods or history of prior episodes of intermittent pain with fatty food ingestion. The physical examination usually reveals an ill-looking patient with significant right upper quadrant tenderness, voluntary and involuntary abdominal guarding and a positive Murphy's sign (discomfort and inspiratory arrest on palpation of the gallbladder fossa during deep inspiration). Although patients have a typical presentation, no clinical or laboratory findings is sufficiently accurate to confirm or refute the diagnosis of acute cholecystitis. Confirmation of the diagnosis is generally made using the physical findings, laboratory studies and radiographic imaging studies. Patients with a typical clinical presentation should undergo imaging studies to confirm the diagnosis. Ultrasonography is the initial best test to establish the diagnosis. The ultrasonographic findings which support the diagnosis include the presence of stones in the gall bladder or cystic duct, gallbladder wall thickening, pericholecystic fluid collection, and positive sonographic Murphy's sign. (Choices A) Plain abdominal x-rays have no role in the diagnosis of acute cholecystitis.

(Choice C) Abdominal CT scan is usually not necessary for the diagnosis of acute cholecystitis. It can easily distinguish the gallbladder wall edema or thickening as the cause of upper abdominal pain; however, right upper quadrant ultrasound remains as the initial test of choice for the diagnosis of acute cholecystitis. (Choice D) Liver function tests are usually normal or slightly elevated in patients with acute cholecystitis; therefore, these will not help to rule in or rule out the diagnosis. (Choice E) ERCP should be done in patients with suspected common bile duct stones. It has no role in the management of patients with acute cholecystitis. Educational Objective: Ultrasonography is the initial test of choice to establish the diagnosis of acute cholecystitis. HIDA scan is useful in excluding cystic duct obstruction in patients with clinical features suggestive of acute cholecystitis. 85% of people answered this question correctly;

Item 2 of 2 The appropriate test is ordered and the results are pending at this point. In the meantime, what is the most appropriate next step in her management? B. C. D. E. A. Emergent exploratory laparotomy Emergent T tube placement and biliary tree decompression Begin intravenous antibiotics Emergent laparoscopic cholecystectomy Perform an endoscopic retrograde cholangiopancreatography

This subscription is licensed to user: roopika only User ID: 123489 Explanation: In patients with uncomplicated acute cholecystitis, the symptoms generally subside in 7 to 10 days; however, if left untreated, there is a high risk of complications. The most common complication in patients with acute cholecystitis include gall bladder gangrene and perforation, development of cholecystoenteric fistula and emphysematous cholecystitis. All patients with acute cholecystitis should therefore be admitted to the hospital and managed with supportive care in the first 24 - 48 hours. This includes adequate analgesia, NPO (nil per os; "nothing by mouth"), placement of nasogastric tube if necessary, appropriate intravenous hydration, and correction of any associated electrolyte abnormalities. All patients with acute cholecystitis should be treated with intravenous antibiotics. Empiric antibiotic therapy is usually directed against the most commonly recovered organisms (Escherichia coli, enterococcus, Klebsiella, and Enterobacter). A commonly used empiric antibiotic regimen is intravenous ampicillin +

gentamicin (for coverage against gram-negative bacilli and synergistic action with ampicillin against enterococcus). Since anaerobic organisms are rarely recovered from patients with acute cholecystitis, their empiric treatment is not necessary. (Choices A and D) All low-risk patients should receive 24 - 48 hours of supportive therapy prior to cholecystectomy. Emergent laparotomy or laparoscopic cholecystectomy is only considered for patients with gallbladder gangrene or perforation, and for patients who fail to respond or have progressive symptoms while on supportive therapy. (Choice B) T tube placement (percutaneous cholecystostomy) is generally considered only for high risk surgical patients (advanced pulmonary or cardiac disease) who continue to have severe symptoms and fail to improve with conservative therapy. (Choice E) ERCP has no role in the management of patients with acute cholecystitis. Educational Objective: All hospitalized patients with acute cholecystitis should be treated empirically with intravenous antibiotics to reduce the risk of secondary infection. 84% of people answered this question correctly A healthy 27-year-old Caucasian man is brought to the emergency department after being involved in a motor vehicle accident. He complains of right knee pain and swelling. He was sitting in the front seat without a seat belt. His knees hit the front dashboard. His vital signs are stable. On examination, there is mild swelling and stiffness. Which of the following was most likely injured in this patient's knee joint? B. C. D. E. A. Anterior cruciate ligament Posterior cruciate ligament Medial meniscus Medial collateral ligament Lateral collateral ligament

This subscription is licensed to user: roopika only User ID: 123489 Explanation: This patient has a classic 'dashboard' injury, wherein a posteriorly directed force is placed on the anterior aspect of the proximal tibia, with the knee in a flexed position. This results in disruption of the posterior cruciate ligament (PCL). A similar injury can be seen in an athlete who falls on a flexed knee with the foot in plantar flexion. Unlike patients with ACL injury, these patients do not complain of a typical 'popping' sound, and may not have an unstable knee. (Choice A) ACL injuries are common in athletic activities. An isolated injury may occur after hyperextension of the knee. Up to 33% to 90% of patients with an ACL tear

experience a 'popping' sensation. The rapid onset of a knee effusion and an unstable knee are common. Lachman's test is a very sensitive physical test for acute anterior cruciate ligament tear. It is done with the patient's knee flexed at 20 degrees; one hand pulls the proximal tibia, while the other hand stabilizes the femur. (Choices C and D) Meniscal injuries principally occur after a twisting injury to the knee with one foot fixed to the ground (e.g. with sudden turning while running). The medial meniscus is more commonly injured, as compared to the lateral meniscus. A bucket handle tear is the most common type of medial meniscus tear. Patients generally complain of pain and swelling of the knee, and a popping sensation at the time of injury. In medial meniscus injuries, tenderness is generally felt along the medial side of the knee. Effusion with meniscus injuries takes about 24 hours to form, and is often not significantly bloody, unlike ACL or osteochondral injuries. Locking of the knee joint on extension is generally seen. McMurray's sign is snapping felt with tibial torsion and the knee flexed at 90 degrees. (Choices E and F) Medial and lateral collateral ligaments are injured when the line of force strikes from the side of the joint. An abduction injury with a torsional component causes a medial collateral ligament tear in most of the instances. On examination, the knee joint is swollen due to the effusion, with tenderness over the medial aspect of the knee. As the medial collateral ligament resists valgus angulations at the knee, injury to this ligament leads to increased angulation of the affected knee on valgus stress. A valgus stress test is generally performed with the knee flexed at 20-30 degrees. Educational Objective: The classic dashboard injury results from a posteriorly directed force on the anterior aspect of the proximal tibia with the knee in a flexed position. This results in disruption of the posterior cruciate ligament (PCL). 50% of people answered this question correctly

The following vignette applies to the next 4 items A 26-year-old woman comes to see you in the office for an annual physical examination. She informs you that she is very dissatisfied with her general physical appearance. She is planning to get breast implant surgery; however, she is worried because she read about an association of silicone breast implants with the development of disease conditions on the Internet. She wants to know your opinion about this matter. Item 1 of 4 Which of the following is the most appropriate response to address her concerns?

A.

It can significantly increase the chances of developing

rheumatoid arthritis. B. Women with silicone implants have a higher incidence of systemic lupus erythematosus than the general population. C. It will cause a slight increase in the lifetime risk of breast cancer. D. They have been associated with an early onset of dementia. E. Capsular contracture and pain can be a frustrating problem in some patients. This subscription is licensed to user: roopika only User ID: 123489 Explanation: Silicone is a long-chain polymer derived from silicon. A silicone implant consists of an outer shell made of silicone, and is inflated with a saline solution. They are most often used by women to "improve" their physical appearance (breast augmentation). They are also used for breast reconstruction in women who have had a unilateral or bilateral mastectomy for various reasons, including breast cancer. There have been a lot of questions and concerns about the safety of silicone breast implants. Silicone breast implants have been a source of a lot of media attention due to the alleged claims by some women regarding their association with systemic disorders. Some women have claimed that silicone implants have and can cause connective tissue and autoimmune disorders (i.e. rheumatoid arthritis and systemic lupus erythematosus), as well as increase the risk of cancer and neurological diseases; however, no evidence currently exists which can support such statements. On the contrary, women who have received breast implants have had postoperative complications in the form of capsular contracture, causing pain and distortion of the shape, implant deflation, and rupture, requiring surgical removal of the implant. These local complications are the most common cause of morbidity associated with implants. (Choices A and B) Contrary to the claims by some women, the general consensus is that silicone breast implants do not cause any major connective tissue or autoimmune disease. There is no evidence of an increase in the incidence of rheumatoid arthritis, lupus, Raynaud's disease, or scleroderma in these women. (Choice C) There is no evidence that silicone breast implants cause or increase the incidence of primary or recurrent breast cancer. (Choice D) There is no reported evidence of early or late onset dementia associated with silicone implants. Educational Objective: Silicone breast implants do not cause any major rheumatologic, autoimmune, or neurological disorder. Some patients can have postoperative capsular contracture, causing pain and disfigurement.

78% of people answered this question correctly. Item 2 of 4 The patient proceeds with breast implant surgery. She comes to see you a few weeks later for a follow-up visit. She is planning to get pregnant, and is concerned about the adverse effects of silicone on the baby. Which of the following is the most appropriate response? A. There is no evidence of harmful effects of silicone implants to the developing fetus. B. You should not get pregnant for at least 2 years after the breast implants. C. Silicone breast implants have been implicated in the pathogenesis of mental retardation in some studies. D. There is a weak correlation between silicone breast implants and the development of autism. E. The risk of birth defects is greatest in the first 5 years. This subscription is licensed to user: roopika only User ID: 123489 Explanation: Although there is a lot of concern about the harmful effects of silicon from breast implants crossing the placenta and entering the fetal circulation, there is no evidence that having silicone breast implants causes any harmful effects, such as birth defects or any other neurological abnormalities, on the developing fetus. (Choices B, C, D and E) There is no evidence of a higher incidence of birth defects or disorders in children born to women with silicone implants. Postponing the pregnancy is not necessary. Educational Objective: Silicone breast implants do not cause any disease or defects in the developing fetus. 90% of people answered this question correctly

Item 3 of 4 During the same visit, she also expressed her concerns about breast-feeding the baby. She is concerned that some of the silicone will be admixed with the breast milk and cause adverse effects. She asks you if she should avoid breast-feeding the baby. Which of the following is the most appropriate response? B. A. Silicone in the breast milk can cause diarrhea in the baby. She should avoid breast-feeding the first baby.

C. The breast milk should be tested for the amount of silicone before starting breast-feeding. D. She should preferably use infant milk formula, if possible. E. There is no risk in breast-feeding. This subscription is licensed to user: roopika only User ID: 123489 Explanation: Breast milk is the best source of nutrition for the babies. All women, even those with breast implants, should be encouraged to breast feed their babies. There is no risk in breast-feeding with a silicone breast implant. (Choice C) There is no need to check silicone levels in the breast milk. There is actually no difference in the silicon content of breast milk in women with silicone implants from those without implants. Futhermore, there is no evidence that the presence of silicone in the milk is harmful to infants. (Choice D) Silicon content is actually lower in breast milk than the levels found in cow's milk or infant's formula. (Choice A) There is no association between the presence of silicone in breast milk and diarrhea. Educational Objective: Breast milk is safe and the best source of nutrition for infants. There is no evidence of any harmful effects in babies who are breastfed by mothers with silicone breast implants. 82% of people answered this question correctly

Item 4 of 4 After several years, your patient returns and requests for a mammographic screening. She is now 35 years old, and her sister was recently diagnosed with breast cancer. Which of the following is the most appropriate statement regarding silicone breast implants and mammographic screening for breast cancer? A. The presence of silicone breast implants never interferes with the mammographic screening. B. Women with implants should continue to get regular mammograms as per the guidelines. C. Women with breast implants should have more frequent mammograms as implants may obscure the cancerous tissue.

D. Regular exposure of silicon implants to radiation from mammography leads to early implant leaks or rupture. This subscription is licensed to user: roopika only User ID: 123489 Explanation: Regular mammograms starting at age 40 years, or earlier if there are risk factors, have consistently been shown to reduce morbidity and mortality from breast cancer. It plays an important role in detecting the lesions in early stages, thereby improving the chances of a complete cure. It is recommended that all women, including those with breast implants, obtain regular screening mammograms as per the guidelines. (Choice A) The presence of capsule contraction can interfere with obtaining good quality imaging. The capsule around the implant can also develop small calcifications which may appear similar to the micro-calcifications seen in cancerous tissue. This can rarely lead to a false positive result, requiring further investigation. (Choice C) There have been some concerns that the silicone implant may obscure a cancerous lesion during routine mammographic screening. This has never been proven in clinical studies; thus, it does not warrant more frequent screening efforts. (Choice D) There is no evidence that radiation exposure affects the stability of breast implants. Educational Objective: Women with breast implants should continue to have screening mammograms at regular intervals. 75% of people answered this question correctly; An 18-year-old high school sophomore is brought to the emergency room after she was involved in a motor vehicle accident. Upon her arrival to the emergency room, she is minimally responsive, and there is evidence of bleeding from the right side of her head. Her vital signs are as follows: temperature 37.6C (99.6F), blood pressure 182/98 mmHg, heart rate 52 per minute and respiratory rate 6 per minute. Her oxygen saturation is 96% on two liters of oxygen. Her breath smells of alcohol. Physical examination reveals a right pupillary size of 7 mm with minimal response to light. The left pupil is 3 mm with normal pupillary light reflex. The right eye is deviated outwards and downwards. There is bilateral papilledema on funduscopic examination. While in the ER, the patient had an episode of generalized body extension, with adduction and pronation of her upper extremities. There is no evidence of any thoracic or abdominal injuries. Which of the following is the most appropriate next step in the management of this patient? B. A. Intravenous thiamine CT scan of the brain

C. D. E.

Intravenous mannitol Hyperventilation to achieve a PaCO2 of 25-30 mmHg Endotracheal intubation

This subscription is licensed to user: roopika only User ID: 123489 Explanation: The above patient has the typical clinical features of elevated intracranial pressure (ICP), most likely secondary to an intracranial hemorrhage from head trauma sustained in the motor vehicle accident. Other causes of elevated intracranial pressure include an intracranial hemorrhage from ruptured aneurysms, bleeding AV malformations, CNS infections, CNS neoplasm, and hydrocephalus. Patients with intracranial hypertension have been classically described to have bradycardia, hypertension and respiratory depression (Cushing's triad). In the early stages, patients complain of headaches, vomiting, blurred vision, and have papilledema on funduscopic examination. Further elevation in the intracranial pressure leads to transtentorial herniation of brain tissue, causing altered levels of consciousness (stupor progressing to coma), dilation of the ipsilateral pupil, third cranial nerve palsy, hemiparesis, decerebrate posturing, and eventually, respiratory arrest. The patient in the above vignette has signs of marked respiratory depression and intracranial hypertension. The most important next step in the management is to rapidly intubate the patient to protect and maintain her airway, in case of a respiratory arrest. (Choice A) Intravenous thiamine is useful in alcoholic patients with severe malnutrition to prevent the development of Wernicke's encephalopathy. It has no role in the lowering of an elevated intracranial pressure. (Choice B) CT scan of the brain should be done urgently to look for an intracranial hemorrhage or any other potentially reversible causes of elevated intracranial pressure; however, the patient's airway should be secured before sending her for neuroimaging. (Choice C) Intravenous mannitol is an osmotic diuretic which reduces brain volume by drawing water out of the cells. It can be used to acutely lower elevated intracranial pressures in emergent situations; however, securing and maintaining a patent airway should still remain the first priority in the management of such patients. (Choice D) Lowering the PaCO2 by hyperventilating the patient causes cerebral vasoconstriction, which decreases the cerebral blood flow, and reduces intracranial pressure. Hyperventilation is therefore used to decrease the PaCO2 to 25-30 mmHg and to rapidly lower intracranial pressure in emergent situations; however, it is contraindicated in patients with traumatic brain injury and an acute stroke. This is due to the fact that a significant decrease in cerebral blood flow caused by vasoconstriction can lead to worsening of neurological injury in these patients.

Educational Objective: Securing and maintaining a patent airway is the first priority in the management of trauma patients with symptomatic intracranial hypertension. 76% of people answered this question correctly A 24-year-old Caucasian male presents to your office with a 24-hour history of mild scrotal pain. His past medical history is insignificant. He is sexually active, having a stable relationship with one partner over a period of several months. He is not taking any medications and denies drug abuse. His temperature is 37.2 C (99 F), blood pressure is 120/70 mmHg, pulse is 80/min, and respirations are 14/min. Physical examination reveals mild, right-sided testicular swelling and tenderness. The cremasteric reflex is intact, but the test of testicular elevation is inconclusive. The urinalysis shows no abnormalities. Which of the following is the most likely diagnosis in this patient? B. C. D. E. A. Testicular torsion Testicular neoplasia Orchitis Epididymitis Varicocele

This subscription is licensed to user: roopika only User ID: 123489 Explanation: The typical clinical presentation of epididymitis is mild-to-moderate scrotal pain accompanied by swelling and tenderness. No irritative voiding symptoms are usually present, and the urinalysis is normal. More dramatic presentation characteristics for acute bacterial epididymitis with severe scrotal pain, fever, and voiding symptoms are less common. The most common cause of epididymitis is Chlamydia trachomatis. Prehn's sign (decrease in pain on testicular elevation) is usually but not universally present. (Choice A) Testicular torsion is characterized by the sudden development of moderate-tosevere pain. High-riding testis can usually be palpated on the affected side. The cremasteric reflex is usually absent. In equivocal cases, color Doppler ultrasonography is helpful. (Choice C) Orchitis that is caused by bacteria or which complicates mumps infection presents with sudden onset of fever, severe scrotal pain, and swelling. (Choice B) A testicular tumor typically manifests as a painless scrotal mass discovered by the patient or physician on physical examination. (Choice E) Varicocele presents as dull scrotal pain relieved by recumbency. A soft scrotal mass can be palpated ('bag of worms') that disappears in the recumbent position. Varicocele is left-sided or bilateral in a vast majority of cases.

Educational Objective: The typical clinical presentation of epididymitis is mild-to-moderate scrotal pain accompanied by swelling and tenderness. Prehn's sign is usually positive, and the cremasteric reflex is intact. 40% of people answered this question correctly; A 70-year-old Caucasian male presents to the emergency department due to right-sided arm and leg weakness that occurred several hours ago. The episode lasted 15 minutes and resolved spontaneously. He had a similar episode of right arm weakness two days ago. He was diagnosed with hypercholesterolemia on routine check-up three months ago, and has been treated with pravastatin. He does not smoke or consume alcohol. His blood pressure is 140/90 mmHg and his heart rate is 70/min. Doppler studies shows 89% stenosis in his left carotid artery and 40% stenosis in his right carotid artery. He undergoes left carotid endarterectomy. The surgery goes well and he recovers from the anesthesia without any complications. In the recovery room, upon protrusion of the tongue, the tongue deviates to the left. Which of the following structures is most likely damaged? B. C. D. E. A. Left vagus nerve Left hypoglossal nerve Left recurrent laryngeal nerve Right facial nerve Ansa hypoglossus nerve

This subscription is licensed to user: roopika only User ID: 123489 Explanation: A number of nerve injuries can occur during carotid endarterectomy. Inadvertent retraction or transection of the hypoglossal nerve causes tongue deviation to the site of injury, as observed in this patient. (Choice C) Recurrent laryngeal nerve injury leads to unilateral vocal cord paralysis, with a resultant change in voice quality. This nerve lies distal to the area of carotid dissection . (Choice D) The facial nerve can be damaged after it exits from the stylomastoid foramen and courses along the inferior portion of the ear. Its marginal mandibular branch (which supplies the orbicularis oris muscle) is most commonly injured, with a resultant asymmetric smile. (Choice A) The vagus nerve lies posterolaterally in the carotid sheath and is also at risk for injury during the procedure.

(Choice E) The ansa hypoglossus nerve innervates the strap muscles of the neck; unlike the other nerves, this nerve may be sacrificed with impunity. Educational Objective: A number of nerve injuries can occur during carotid endarterectomy. Inadvertent retraction or transection of the hypoglossal nerve causes tongue deviation to the site of injury. *Extremely important question for USMLE step-3 84% of people answered this question correctly

A 62-year-old Caucasian man is admitted to the hospital for coronary artery bypass grafting and aortic valve replacement. He has a past history of myocardial infarction, hypertension, hyperlipidemia, and chronic renal insufficiency. He is a retired financial advisor with a 50-pack year smoking history. He underwent the surgery without any complications, and was successfully extubated the same day. His post-operative medications include aspirin, metoprolol, lisinopril, atorvastatin, and morphine via patientcontrolled analgesia (PCA) pump. On the next day, the nurse calls you with concerns about his mental status. He has been getting progressively drowsy, lethargic, and extremely difficult to arouse. His temperature is 36.1 C (97 F), blood pressure is 90/60 mmHg, respirations are erratic with a rate of 2-4/min, heart rate is 58/min, and oxygen saturation is 92% on 2L/min of oxygen. His pupils are equal and pinpoint. What is the most appropriate next step in the management of this patient? A. necessary B. C. D. E. Order 2 mg of intravenous naloxone and repeat as

Raise the head of the bed to 60 degrees to prevent aspiration Obtain a stat arterial blood gas analysis to rule out CO2 narcosis Check and secure his airway Administer intravenous doxapram (respiratory stimulant)

This subscription is licensed to user: roopika only User ID: 123489 Explanation: Morphine overdose is a common scenario in hospitalized patients, especially for postoperative patients receiving high doses of morphine for acute pain control. Opiate naive patients with renal insufficiency are at a much higher risk. The usual manifestations of overdose are lethargy, drowsiness, miosis, bradycardia, hypotension, respiratory depression, and apnea. The first and initial priority in the management of overdose is always to check for and maintain a patent airway. It is important to ensure adequate respiratory exchange at all times before instituting specific therapy.

(Choice A) Intravenous naloxone is the specific antidote for reversing the effects of morphine overdose. It is usually given as a 2 mg IV dose and repeated as necessary up to a maximum dose of 10 mg. However, adequate control of the airway is still the first priority in all patients. (Choice B) The airway should be secured invasively in this setting to prevent aspiration. Raising the head end of the bed alone will not prevent aspiration. (Choice C) Obtaining laboratory studies will lead to an inappropriate delay in the treatment and adversely affect the outcomes. For instance, identifying CO2 retention will not change the initial management in this setting, and should be deferred. (Choice D) The use of respiratory stimulants has not been shown to improve patient outcomes, and is not recommended. Educational Objective: Remember the ABCs of basic life support. Specific therapy with naloxone should be given after ensuring adequate gas exchange. 58% of people answered this question correctly..

A 30-year-old male with seizure-like activities is admitted to the intensive care unit. He is hypercalcemic, and his serum parathyroid hormone levels are also elevated. After the initial resuscitation, the patient is stabilized and is taken to the OR for parathyroidectomy. A single parathyroid adenoma is removed. Eight hours after the surgery, the nurse reports that he has developed facial asymmetry. Physical examination reveals facial asymmetry. He had a history of upper respiratory infection two weeks ago. What is the likely cause of his new symptoms? B. C. D. A. Bell's palsy Hypercalcemia Hungry bone syndrome Complication of the anesthetic agent

This subscription is licensed to user: roopika only User ID: 123489 Explanation: This patient had hypercalcemia secondary to primary hyperparathyroidism, and subsequently underwent removal of the parathyroid adenoma. Following parathyroidectomy, serum calcium levels can fall (see below) and produce symptoms of hypocalcemia such as perioral numbness, muscle cramps, carpopedal spasm, positive Chvostek sign (ipsilateral contraction of facial muscles on tapping the angle of the jaw) and positive Trousseau's sign (rapid development of carpopedal spasm on occlusion of blood supply to the upper extremity). Severe hypocalcemia can cause mental status

changes and seizures. Typically, the signs of hypocalcemia are bilaterally symmetrical. Hemifacial involvement is not typically seen; therefore, with a preceding history of upper respiratory infection and facial asymmetry, the most likely diagnosis of this patient is Bell's palsy (lower motor nuclear involvement of the facial nerve). (Choice B) Typical features of hypercalcemia include nausea, vomiting, constipation, polyuria, and pleuritis. Mental status changes can occur with severely high calcium levels. (Choice C) Following parathyroidectomy (adenoma removal or removal of three-and-half parathyroid glands), serum calcium can fall by relative hypoparathyroidism or hungry bone syndrome. Relative hypoparathyroidism results from the suppression of normal parathyroid glands by high calcium levels, which fail to respond to low calcium levels following surgery. The suppression is transient and most patients recover in few days. Hungry bone syndrome is caused by the sudden withdrawal of parathyroid hormone in patients with severe hyperparathyroidism, causing an increased influx of calcium from the circulation into the bone. Preoperative risk factors for hungry bone syndrome are severe hyperparathyroidism, severe bone disease and vitamin D deficiency. All these conditions have a very high bone turnover. Postoperatively, when parathyroid hormone levels fall, the dynamics of bone turnover shifts from net efflux of calcium from bone to net influx of calcium into the bone. In hungry bone syndrome, the serum calcium typically falls to its nadir between 2 to 4 days following surgery. Eight hours following surgery is too early to get symptomatic hungry bone syndrome. Educational Objective: Hypocalcemia following parathyroidectomy is caused by relative hypoparathyroidism or hungry bone syndrome. The signs of hypocalcemia are typically bilateral. Unilateral signs may suggest an alternative diagnosis. Hungry bone syndrome usually develops 2 to 4 days following surgery. *Extremely important question for USMLE step-3 61% of people answered this question correctly;

A 76-year-old white female with a history of hypertension, type-2 diabetes, and multiinfarct dementia was sent to your office because of an erythematous painful lesion under her upper dentures. Her daughter is concerned about cancer, since she smoked and chewed tobacco for several years. She lives in a nursing home. She wears full dentures all the time. She takes aspirin, glyburide and enalapril. She has a 50-pack-year history of smoking, and quit drinking alcohol five years ago. Her vital signs are stable. On examination, an erythematous, tender lesion underneath the upper dentures is noted. The rest of the oral cavity is unremarkable. Which of the following is the most appropriate next step in the management of this patient? A. Prescribe antifungal cream

B. C. D.

Refer the patient to oromaxillary surgeon for resection Obtain deep biopsy of the lesion Advise the patient to remove dentures for one to two weeks

This subscription is licensed to user: roopika only User ID: 123489 Explanation: The most common cause of oral lesions in elderly patients is trauma. People who wear dentures all the time are more prone to develop such lesions. The most appropriate next step is to advise the patient to remove her dentures for one to two weeks, after which reevaluation is necessary. If the lesion does not subside, the patient should have a biopsy to rule out malignancy. Educational Objective: Patients who wear dentures all the time are prone to develop oral lesions. The most appropriate initial step for such patients is to simply advise them to remove their dentures for a couple of weeks. 63% of people answered this question correctly

The following vignette applies to the next 2 items A 14-year-old girl comes to the emergency department with her mother for the evaluation of left knee pain. The girl was playing soccer when she fell and injured her left knee after being tackled by one of the players. She immediately felt a tearing sensation and excruciating pain in the front of her left knee. She is presently having a great deal of pain and difficulty in rising and bearing weight on her left leg. On physical examination, there is diffuse swelling and tenderness on the anterior aspect of her left knee. She has a significant joint effusion. Passive extension and flexion of her left knee are normal; however, she is unable to maintain extension of the left knee against gravity. The anterior and posterior drawer signs are normal. The valgus and varus stress to the left knee does not reveal an increased laxity or instability on the medial or lateral side. McMurray's maneuver on the left knee does not reveal a popping sensation with external rotation and passive extension of the lower leg. Item 1 of 2 Which of the following is the most likely explanation for her findings? B. C. A. Anterior cruciate ligament tear Medial collateral ligament tear Lateral collateral ligament tear

D. E.

Patellar tendon tear Medial meniscal tear

This subscription is licensed to user: roopika only User ID: 123489 Explanation: Soft tissue knee injuries are extremely common in adolescent and young adults who engage in various contact sports. Most patients complain of an immediate, profound pain and difficulty or inability to bear weight on the same leg. It is important to establish the correct anatomic diagnosis of these injuries in order to institute an early and specific intervention. The most common vulnerable structures in the knee include the anterior and posterior cruciate ligaments, medial and lateral collateral ligaments, medial and lateral meniscus, and joint capsule. A detailed history and physical examination (including specific maneuvers) can help in differentiating between the damage to various possible knee structures. The patient in the above vignette has a patellar tendon tear. The patellar tendon is the distal extension of the quadriceps insertion. Its rupture is usually due to traumatic athletic injury. The most common mechanism is a sudden and unusual quadriceps contraction with the foot firmly planted, and with the knee in partial flexion during the injury. Patellar tendon tear or rupture usually occurs at the osseotendinous junction and results in excruciating pain, swelling, and difficulty in bearing weight. Physical examination generally reveals swelling and tenderness in the anterior part of the knee. With a complete tendon rupture, the patients are unable to perform active extension of the leg and are unable to maintain the passively extended knee against gravity. (Choice A) Anterior cruciate ligament tears are one of the most serious forms of knee injuries. Most patients complain of profound pain, inability to ambulate, and a popping sensation or sound at the time of injury. The anterior drawer test is commonly used to evaluate the integrity of the anterior cruciate ligament. The patient is placed in the supine position with the hip flexed at 45 degrees, the knees bent at 90 degrees, and the patient's foot firmly planted on the examination table. With both hands behind the affected knee, the examiner tries to displace the lower leg anteriorly. A difference of 1 cm compared with the opposite side suggests a complete tear of the anterior cruciate ligament. (Choices B and C) The valgus and varus stress tests are used to determine the integrity of the medial and lateral collateral ligaments, respectively. A history of knee injury, local tenderness on the medial or lateral joint lines, and an abnormal opening of the knee as compared to the opposite side on valgus or varus stress is suggestive of ligament injury or disruption. (Choice E) Menisci are the fibrocartilaginous pads located between the femoral condyles and tibial surfaces. The common mechanism of meniscal injury is a twisting force with the foot fixed on the ground. This is commonly seen during football and basketball games. McMurray's maneuver is used to detect the presence of meniscal tears. To detect a

medial meniscal injury, the patient is put in a supine position with the knee in maximum flexion. One hand of the examiner is placed on the posteromedial margin of the involved knee and the other hand supports the foot. The tibia is then externally rotated, and the knee is extended slowly. The test is positive if there is an audible or palpable click or popping sensation during extension of the involved knee. Educational Objective: Patellar tendon rupture presents with excruciating pain, swelling in the anterior part of the knee, and an inability to maintain passive extension of the knee against gravity. 75% of people answered this question correctly;

Item 2 of 2 Which of the following is the most appropriate next step in the management of this patient? B. six weeks C. D. A. Complete bed rest for six weeks Complete bed rest for two weeks with gradual return to baseline activity in Consult orthopedic surgery for early surgical intervention Brace immobilization and ice packs in the local area for four weeks

This subscription is licensed to user: roopika only User ID: 123489 Explanation: Based on the clinical presentation, the patient appears to have a complete disruption of the patellar tendon and dysfunction of the knee extensor mechanism. In all such patients, early surgical intervention and repair of the patellar tendon is the treatment of choice. Complete rupture of the patellar tendon causes significant damage to the knee extensor mechanism. Delayed treatment can lead to quadriceps muscle atrophy, contracture formation, and limited range of motion of the knee, thereby causing early fatigue, pain, and significant disability. Early diagnosis and surgical repair usually lead to excellent recovery of knee function and prevent long-term disability. (Choices A and B) Non-operative treatment (conservative treatment with bed rest and pain control) has a very limited role in the management of patients with acute patellar tendon rupture. It can lead to a significant dysfunction of the knee extensor mechanism and long-term disability. (Choice D) A cast or brace immobilization in full extension is used very occasionally in patients with partial patellar tendon tear. Complete patellar tendon tears are ideally managed by early surgical intervention.

Educational Objective: Early surgical repair of the ruptured patellar tendon is recommended to promote the recovery of normal knee motion and prevent long-term disability. 68% of people answered this question correctly

A 23-year-old Caucasian male is brought to the emergency department after being involved in a major motor vehicle accident. His initial blood pressure is 80/60 mmHg and heart rate is 120/min. After IV fluid resuscitation, his blood pressure and heart rate become 110/80 mmHg and 100/min. His superficial injuries are cleaned and bleeding vessels ligated. One hour after the admission, the patient complains of shortness of breath. Physical examination reveals decreased breath sounds on the right side. The apical impulse is not displaced, and the trachea is strictly midline. His abdomen is soft and non-tender on palpation. You are suspecting pneumothorax. What is the best initial test to confirm the diagnosis? B. C. D. E. A. Upright chest X-ray, PA view Expiratory chest radiograph Standard supine chest X-ray, AP view Lateral decubitus chest X-ray CT scan of the chest

This subscription is licensed to user: roopika only User ID: 123489 Explanation: An upright chest x-ray is the initial test of choice to confirm the diagnosis of pneumothorax. The accumulation of air occurs primarily in the apical and lateral regions when the patient is upright, and is usually seen as a convex white visceral pleural line on x-ray. As little as 50 ml of pleural gas can be visible on upright x-ray. (Choice B) It has been stated that an expiratory x-ray may significantly improve the rate of detection of pneumothorax; however, it has been shown that this statement is incorrect. As an example, one study of 85 patients with pneumothoraces and 93 controls found that inspiratory and expiratory upright chest radiographs have equal sensitivity for pneumothorax detection. Furthermore, considering the limitations of expiratory radiographs, only inspiratory films are recommended as the initial examination of choice for pneumothorax detection. (Choice C) A standard supine chest x-ray is less sensitive than an upright PA chest x-ray; therefore, the former is less appropriate. (Choice D) A lateral decubitus chest x-ray may be used to look for a small pneumothorax, but it is not routinely employed as the initial test of choice.

(Choice E) CT scan is not recommended for routine use, but may be helpful in selected cases (e.g., to distinguish between a large bulla and a pneumothorax). Educational Objective: An upright chest x-ray (PA view) is the initial test of choice to confirm the diagnosis of pneumothorax. 54% of people answered this question correctly; The following vignette applies to the next 2 items A 50-year-old male comes in to the emergency department complaining of unremitting pain in an area of his right buttock near the anus. Although he is uncertain when the discomfort first began, he notes that the pain has increased significantly over the past 24 hours. He finds he is unable to pass stool because of the pain. He has not experienced symptoms like this before. Ten years ago, he was diagnosed with diabetes mellitus. He is currently on insulin. He smokes half a pack of cigarettes per day but would like to quit. He denies drinking alcohol or using recreational drugs. His temperature is 38.1C (100.6F), blood pressure is 128/86 mm Hg, and pulse is 90/min. There is a hot, tender, mass located between the anus and the left ischial tuberosity. The mass fluctuates upon palpation. Item 1 of 2 What is the most appropriate next step in the management of this patient? B. C. D. E. A. Increase the insulin dose and discharge home Prescribe bupropion and discharge home Prescribe oral antibiotics and discharge home Admit to the hospital and start intravenous antibiotics Incise and drain the mass

This subscription is licensed to user: roopika only User ID: 123489 Explanation: Anal abscesses arise when one or more of the several glands that encircle the anus become blocked and the bacteria within grow unchecked. Patients with anal abscesses typically present with severe, constant pain that may be accompanied by fever or malaise. Physical examination commonly reveals erythematous, indurated skin or a fluctuant mass over the perianal or ischiorectal space. Purulent material may be seen if the abscess has begun to drain spontaneously. It is widely agreed that the most important aspect of treatment is prompt incision and drainage of the abscess (Choice E). Perianal and small ischiorectal abscesses are often drained in the office, but larger ischiorectal abscesses typically require surgical intervention.

Increasing the insulin dose (Choice A) or prescribing bupropion (Choice B) are inappropriate because they do not treat the abscess. However, strict control of diabetes is very important in reducing the complications associated with wound healing. Oral antibiotics (Choice C) should also be prescribed to those patients who have diabetes mellitus, immunosuppression, extensive cellulitis, or valvular heart disease. However, antibiotics are an adjunct to and not a substitute for incision and drainage of the abscess. Admission to the hospital for intravenous antibiotics (Choice D) is unnecessary in a patient with localized infection. Educational Objective: Anal abscesses should be treated with prompt incision and drainage. Antibiotics should also be prescribed in those patients who have diabetes mellitus, immunosuppression, extensive cellulitis, or valvular heart disease. 86% of people answered this question correctly; Item 2 of 2 The patient is treated appropriately. To minimize risk of complications, he is advised to promptly schedule a follow-up visit with his primary care physician. Patients with anal abscesses are at greatest risk of developing which of the following conditions? A. B. C. D. E. Soft tissue infection Fistula Fecal impaction Incontinence Hemorrhoids

This subscription is licensed to user: roopika only User ID: 123489 Explanation: While an anal abscess is an infection within one or more of the anal spaces, an anal fistula (Choice B) is a tunneling between the anus or rectum and another epithelial lined space (eg, the skin overlying the drainage site). Fifty percent of patients with anal abscesses will go on to develop a chronic fistula from the involved anal gland to the overlying skin. Patients with fistulas typically present with an anal abscess that persists after incision and drainage, or with a pustule-like lesion in the perianal or ischiorectal area that continually drains. Surgical repair is usually necessary to eliminate the fistula while preserving fecal continence. Soft tissue infection (Choice A) is rarely a concern when the initial mass has been properly incised and drained.

Fecal impaction (Choice C) could be of concern in a patient with an anal abscess who had been unable to pass stool for a long period before presenting for treatment. It does not commonly occur subsequent to treatment of anal abscess, however. Incontinence (Choice D) and hemorrhoids (Choice E) are not commonly associated with anal abscesses. Educational Objective: Fifty percent of patients with anal abscesses will go on to develop a chronic fistula from the involved anal gland to the overlying skin. Such fistulas require surgical repair. 87% of people answered this question correctly

A 35-year-old white male comes to the emergency department because he cut his finger with a rusty knife while doing yard work. His cut was quite deep, and his wife immediately applied a pressure dressing to stop the bleeding. His immunization status is unknown. He last saw his doctor approximately ten years ago. On examination, a deep laceration is noted on the palmar aspect of his right middle finger. Which of the following is the most appropriate next step in the management of this patient? B. C. D. A. Tetanus immune globulins Tetanus antitoxin Tetanus immune globulin and Tetanus antitoxin No immunization

This subscription is licensed to user: roopika only User ID: 123489 Explanation: The above patient sustained a penetrating injury to his hand with a rusty knife. He is at risk for the development of tetanus; therefore, he needs tetanus prophylaxis. As his immunization status is unknown and his wound is not clean, he should be given both tetanus immune globulins and tetanus antitoxin. (Choice A) Tetanus immune globulin is indicated only for contaminated wounds when the immunization status is unknown, or when the patient has received less than three doses of tetanus antitoxin. (Choice B) The situations in which only tetanus antitoxin is given are the following: 1. when the patient has a clean wound but unknown immunization status, or if he has received less than three doses of tetanus antitoxin 2. when the patient has a clean wound and has received three or more doses of tetanus antitoxin with the last dose received more than ten years ago 3. when the patient has a contaminated wound and has received three or more doses of tetanus antitoxin with the last dose received more then five years ago

Educational Objective: For those patients with a contaminated wound and unknown immunization status, both tetanus antitoxin and tetanus immune globulins are indicated. 88% of people answered this question correctly A 3-month-old Caucasian boy is brought to your office by his mother because of a bulge in his groin. The mother says that the bulge appears whenever the child cries. On examination, you notice that the bulge appears in the right inguinal area upon straining of the infant, and extends toward the scrotum. His past medical history is significant for bronchiolitis when he was 2 months old. Which of the following is the best statement concerning the patient's condition? B. C. D. E. A. It is most likely a direct hernia. Muscular weakness of the abdominal wall is likely to be present. The condition can resolve with age. Surgical repair should be deferred until 1 year of age. Elective repair is indicated as early as possible.

This subscription is licensed to user: roopika only User ID: 123489 Explanation: An indirect inguinal hernia that is related to the failure of the processus vaginalis to obliterate is the most likely diagnosis of this patient. Elective repair should be performed as soon as possible. (Choices A and B) A direct hernia related to muscular weakness of the abdominal wall is rare in this age group. It is usually seen in the elderly age group. (Choices C and D) The presence of an inguinal hernia is an indication to elective repair. This condition will not resolve with age, and has a high risk of potential complications, including incarceration, if it remains unrepaired during the first months of life. Educational Objective: Inguinal hernias in the pediatric age group should be surgically repaired as early as possible. These will not resolve with age. The risk of potential complications, including incarceration, is particularly high if it remains unrepaired during the first months of life. 47% of people answered this question correctly

A 56-year-old male accountant is brought to the emergency department by paramedics because of a sudden onset of severe crushing chest pain and dizziness. The pain started

while he was in a meeting with some of his clients. He has a history of longstanding hypertension, gastroesophageal reflux disease, hyperlipidemia, and chronic stable angina. He occasionally gets substernal chest pressure with brisk walking and running, and this is completely resolved by sublingual nitrates. He is a lifelong smoker and currently smokes two packs of cigarettes a day. His other medications include aspirin, hydrochlorothiazide, metoprolol and atorvastatin. During this episode, he took four sublingual nitroglycerine tablets in the first 10 minutes, which did not change the character and severity of the pain. His temperature is 36.7C(98F), blood pressure is 160/86 mm Hg, pulse is 80/min, and respirations are 16/min. Oxygen saturation is 94% on room air. An initial EKG and chest x-ray done in the emergency department are normal. Which of the following is the most appropriate statement to tell the patient? B. C. D. E. A. You may be having an acute myocardial infarction. You may have a perforated peptic ulcer. You may have an aortic dissection. You may have a pulmonary embolism. You may have a tension pneumothorax.

This subscription is licensed to user: roopika only User ID: 123489 Explanation: Aortic dissection is an uncommon but potentially life threatening cause of severe and sudden chest pain. It has the same predisposing risk factors as an acute coronary syndrome (unstable angina and myocardial infarction), including age greater than 55 years, hypertension, hyperlipidemia, and chronic smoking. The diagnosis of aortic dissection is usually based on the history and the physical examination findings. Patients with acute aortic dissection typically present with a sudden onset of severe or "tearing" chest pain radiating to the back. The pain is usually not relieved by sublingual nitroglycerine. Other clinical clues favoring the diagnosis of an aortic dissection include the blood pressure difference in the two arms, an early diastolic murmur (reflecting aortic insufficiency), and the presence of mediastinal widening on chest x-ray. (Choice A) The patient certainly has risk factors and a history of coronary artery disease based on his symptoms of exertional angina. A sudden onset of severe chest pain, unrelieved by nitrates, can be due to an acute myocardial infarction; however, it would be unusual to have a normal EKG in a patient with acute MI while he is having active chest pain. (Choice B) Peptic ulcer perforation is an unlikely diagnosis in the absence of any abnormal abdominal exam findings. Furthermore, a chest-x ray/erect abdominal-x ray usually shows air under the diaphragm. (Choice D) Pulmonary embolism usually presents with pleuritic chest pain that is less severe in intensity, and is associated with shortness of breath and hypoxia.

(Choice E) Tension pneumothorax presents as a sudden onset of shortness of breath, hypoxia, absent airflow, and hyperresonance to percussion on the affected side. The patient's chest x-ray and examination findings are not consistent with tension pneumothorax. Educational Objective: Aortic dissection is a catastrophic condition, and should be suspected from the initial history and physical examination findings. 32% of people answered this question correctly

A 43-year-old Caucasian man comes to the emergency department complaining of moderate right upper quadrant pain and fever for the last 24 hours. He has no other medical problems. He does not use tobacco, alcohol, or drugs. His family history is not significant. He takes no medications. His temperature is 38.3 C (101.0 F), blood pressure is 120/70 mmHg, pulse is 96/min and respirations are 18/min. There is mild jaundice of his eyes and oral conjunctiva. The abdomen is soft, non-distended, and moderately tender on palpation of the right upper quadrant. Bowel sounds are mildly increased. The patient laboratory tests reveal: LFTTotal bilirubin 3.6 mg/dL Direct bilirubin 2.8 mg/dL Alkaline phosphatase 500 U/L Aspartate aminotransferase 38 U/L Alanine aminotransferase 43 U/L Amylase 70 U/L Which of the following is the most appropriate immediate step in management? B. C. D. E. A. Extracorporeal shock wave lithotripsy Blood cultures and antibiotics Open cholecystectomy Laparoscopic cholecystectomy Diet, analgesics and oral bile acids

This subscription is licensed to user: roopika only User ID: 123489 Explanation: The patient has acute cholangitis. This condition is characterized by Charcot's triad: right upper quadrant pain, fever and jaundice. Charcot's triad is present in 50-75% of the cases of cholangitis. Elevated levels of direct bilirubin and alkaline phosphatase, without a rise in the aminotransferases, confirm the presence of an extrahepatic obstruction. Treatment of cholangitis is begun with adequate hydration, strict vital signs monitoring, and

immediate antibiotic therapy. A blood culture should be taken before starting antibiotics. The preferred empiric antibiotics are ampicillin + gentamicin or monotherapy with imipenem or levofloxacin. In 80% of the cases, the cholangitis is controlled within 24 hours. An elective endoscopic retrograde cholangiopancreatography (ERCP) is then scheduled. If the patient does not improve, an urgent biliary decompression is necessary. Decompression is achieved through biliary drainage, and is usually done through ERCP due to its lower morbidity rate of 10% (compared with 50% morbidity rate via surgical drainage). (Choices C and D) Cholecystectomy is performed in the treatment of gallstones and in some cases of acute or chronic cholecystitis, not acute cholangitis. Although this patient might have gallstones (which could be causing common bile duct obstruction), ERCP extraction of the stones should be tried after hydration and antibiotic therapy. An elective (and not immediate) cholecystectomy may eventually be required by this patient to prevent any recurrence. (Choices A and E) Lithotripsy, diet and bile acids are different therapeutic approaches for gallstones, and are not suited for the management of acute cholangitis. Educational Objective: Acute cholangitis is characterized by Charcot's triad: jaundice, fever and right upper quadrant pain. Patients should be admitted into the hospital. Immediate management includes blood cultures followed by antibiotics, hydration, and close monitoring of vital signs. If the patient clinically improves, an elective ERCP can be scheduled. If conservative therapy fails, an urgent biliary decompression (emergent ERCP) is required. A patient in such a condition can be critical. Hypotension and confusion may develop due to sepsis. When these two signs occur with Charcot's triad, the patient is said to have Reynold's pentad, which has a 50% mortality rate. 72% of people answered this question correctly; A 35-year-old man comes to the emergency department after being involved in a snowmobile accident. He complains of severe pain in his right groin and is unable to extend his right knee. He has no other past medical problems. He is an avid athlete and runs five miles every day. On physical examination, he is unable to extend his right knee against resistance; passive movements are within normal limits. There is a sensory loss over the anterior and medial thigh, medial shin, and the arch of the right foot. The knee jerk is markedly decreased on the right side. Which of the following nerve injuries can most likely explain the physical findings? B. C. D. A. Femoral nerve injury Sciatic nerve injury Obturator nerve injury Peroneal nerve injury

This subscription is licensed to user: roopika only User ID: 123489

Explanation: This patient is most likely suffering from a traumatic femoral nerve injury, which is an uncommon lower extremity nerve injury due to the nerve's location within the pelvis and anterior thigh. On examination, patients with significant femoral neuropathies exhibit weakness involving the quadriceps muscle group, with sparing of leg adduction (which is a function of the obturator nerve). This muscle weakness is exemplified by an inability to extend the knee against resistance. In addition, sensory loss over the anterior aspect and most of the medial aspect of the thigh is typical. This sensory loss extends down the medial shin to the arch of the foot due to dysfunction of the saphenous nerve (branch of the femoral nerve). The knee jerk also decreases in amplitude or becomes absent. (Choice B) A patient with a significant sciatic nerve injury will complain of weakness affecting most of the lower leg musculature, including the hamstrings. Hip flexion, extension, abduction, adduction, and knee extension are usually normal. There is sensory loss involving the lower leg. In contrast to a femoral nerve injury, the medial calf and arch of the foot may be spared secondary to the preserved innervation by the saphenous nerve (a branch of the femoral nerve). Furthemore, sensation is spared above the knee, both anteriorly and posteriorly. The knee jerk is normal, but the ankle jerk is unobtainable. The most common cause of compression or injury to the sciatic nerve in this region is trauma, which includes hip dislocation, fracture, or replacement. Other etiologies include wayward buttock injections, compression by external sources (e.g., prolonged bed rest), and any deep-seated mass in the pelvis (e.g., hematoma). (Choice C) A patient with an obturator nerve injury will present with pain, weakness in leg adduction, and sensory loss over a small area in the medial thigh. Obturator neuropathy is often secondary to pelvic trauma or surgery. (Choice D) A patient with a common peroneal nerve injury will usually present with an acute foot drop accompanied by weakness in foot dorsiflexion and eversion. In addition, the patient may also complain of paresthesias and/or sensory loss over the dorsum of the foot and lateral shin (superficial peroneal nerve territory). The injury is usually located at the knee, on the lateral aspect of the fibular head. Educational Objective: Femoral nerve injury is characterized by the following physical findings: inability to extend the knee, loss of knee jerk reflex, and sensory loss over the anterior and medial aspects of the thigh, medial aspect of shin, and arch of the foot. 67% of people answered this question correctly;

A 22-year-old Caucasian female comes to the office for a visit. She has no significant past medical history. She regularly performs breast self-examinations every month. During her last breast self-examination, she felt a lump on the outer side of her left breast.

The patient started menstruating at the age of sixteen. Her last menstrual period was approximately ten days ago. In the office, breast examination reveals an easily palpable, smooth, cystic, 2 cm mass in the lower and outer quadrant of her left breast. There is no nipple discharge from the left side. Examination of the right breast is normal. The patient is extremely anxious, and reports that her grandmother died of metastatic breast cancer at the age of 55. Which of the following is the most appropriate next step in the management of this patient? B. C. D. A. Reassure the patient that the breast lump is benign. Obtain a diagnostic mammogram. Perform a fine-needle aspiration biopsy. Refer her for an excisional biopsy.

This subscription is licensed to user: roopika only User ID: 123489 Explanation: Breast lumps are a very common patient problem in the outpatient setting. They cause significant anxiety in most patients. A vast majority of breast lumps in women under age 35 are usually due to benign causes, most commonly benign cystic disease or fibroadenoma of the breast; however, any complaint of a breast mass or lump in a female patient should be evaluated thoroughly. All patients should be followed closely until a definite diagnosis is made. A thorough history and physical examination is an important part of a complete evaluation. Benign cysts are most prominent just prior to the beginning of the menstrual cycle, and regress after the menstrual period is over. It is important to examine a young patient in the postmenstrual period to document any regression in the size of the breast lump. Fine-needle aspiration biopsy should be performed in an easily palpable cystic breast mass in an anxious patient to obtain fluid or tissue for a specific diagnosis. Any bloodtinged fluid should be sent for cytologic examination. If the fluid obtained is not bloody, the patient should be reexamined in four to six weeks to look for any regression or recurrence of the lump. Ultrasound of the breast is an alternative option for evaluation of breast masses in patients under age 35. It can differentiate a simple or complex cyst from a solid mass. It can also be used in patients who decline a fine-needle aspiration biopsy. (Choice A) Physical exam findings, although useful, are not specific enough, and do not provide a definite diagnosis of the breast mass. The patient should undergo further diagnostic evaluation to obtain a specific diagnosis, and to alleviate the patient's anxiety. (Choice B) Mammograms are usually not indicated in women less than 35 years of age, as the normal breast tissue is too dense to evaluate the lump. Moreover, mammograms

can miss up to 20% of palpable breast cancers and should be substituted or followed up with another diagnostic modality. (Choice D) Excisional biopsy of the breast mass should not be performed in a young (under age 35) patient without a specific diagnosis. Educational Objective: Fine-needle aspiration biopsy or ultrasound (if fine-needle aspiration biopsy is refused or unavailable) is the diagnostic procedure of choice for easily palpable breast lumps in women younger than age 35. 82% of people answered this question correctly

An 80-year-old Caucasian female is being discharged from the hospital after surgical repair of the right hip fracture. She fractured her right hip after falling down the steps in her home. She has a past history of osteoporosis, hypertension controlled with thiazide diuretics and metoprolol, diabetes mellitus treated with metformin, and hypercholesterolemia controlled with simvastatin. She suffered from an acute myocardial infarction six years ago, but she has not had any complications or recurrence of symptoms. She has a 50-year-old daughter who lives near her house. You advise the patient to consider an alternate living arrangement; however, she clearly expresses her desire to live by herself in her own home. She does not want to live in her daughter's house or in a nursing home. Which of the following is the most appropriate next step? A. for her B. C. D. E. Ask her to live with her daughter for a few weeks Ask the patient to prepare advanced directives Get home assessment by a nurse Ask the daughter to convince her mother to stay with her Inform her that nursing home is the only available option

This subscription is licensed to user: roopika only User ID: 123489 Explanation: Falls are a common cause of morbidity and mortality in the elderly population. These are associated with injuries and fractures, significant decline in functional status, and subsequent greater chance of nursing home placement. Falls in the elderly population occur as a result of a mixture of intrinsic and extrinsic factors. The intrinsic factors include any acute illness or an age-related decline in balance, vision, ambulation, proprioception, cognitive impairment, musculoskeletal or cardiovascular function. Some of the extrinsic stressors that can lead to an increased risk of falls include the use of multiple medications, psychotropic medications, and an unsafe home environment.

Prevention of recurrent falls and the associated complications is important in patients with a history of prior falls. Some interventions that have been proven to be useful in the prevention of falls include muscle strength and balance training, withdrawal of psychotropic medications, and home hazard evaluation and modification by a trained professional. Therefore, the patient in the above vignette should have a total assessment and modification of her home environment to reduce the risk of subsequent falls. (Choices A, B & E) The patient appears competent and does not wish to live in the nursing home or with her daughter. She should not be forced by the physician or her daughter to move to the nursing home or her daughter's house. Educational Objective: Some interventions that have been proven to be useful in the prevention of falls in elderly patients include muscle strength and balance training, withdrawal of psychotropic medications, and home hazard evaluation and modification by a trained professional. 88% of people answered this question correctly

The following vignette applies to the next 2 items A 22-year-old college student comes to see you in the office for the evaluation of right lower leg pain for the last five days. The pain gets worse with exercise and weightbearing, and improves with rest. She is an avid runner, and has been preparing for an upcoming marathon event for the last two months. She says that her shoe has been bothering her a lot recently, especially on the lateral side of the right foot. She denies any other past medical history or history of recent trauma. On physical examination, she has point tenderness with localized swelling over her right lower tibial border. You suspect a distal tibial stress fracture. A plain anteroposterior x-ray film of the right lower leg does not reveal any break in the cortical surface of the bone. Item 1 of 2 Which of the following is the most appropriate test to confirm the diagnosis? B. C. D. E. A. Obtain a PA/lateral film of the right lower leg. Obtain x-rays of both lower extremities for comparison. Repeat the plain x-rays of the right lower extremity after one week. Obtain an ultrasound of her right lower extremity. Obtain a magnetic resonance imaging of her right lower extremity.

This subscription is licensed to user: roopika only User ID: 123489 Explanation:

Excessive running is associated with a high incidence of lower extremity injuries, usually to the feet or lower legs. Stress fractures most commonly occur on the lower one-third of the tibia, fibula, lateral metatarsals, and sometimes the tarsal bones of the foot. These are usually seen in athletes and military recruits with a history of excessive running or training. Patients often complain of pain that occurs with exercise or running, and improves with rest. Physical examination reveals localized tenderness and swelling over the fracture site. It is important to distinguish a stress fracture from the other common tendon and ligament injuries in order to provide specific therapy. Plain radiographs are often unremarkable in the first two to four weeks after the injury or fracture. Magnetic resonance imaging or triple phase bone scans with technetium are highly sensitive and accurate imaging modalities used to confirm the diagnosis of stress fracture. (Choice A, B) Obtaining PA/lateral x-ray films or x-ray of the other extremity for comparison does not change the sensitivity of plain x-rays in diagnosing stress fractures. (Choice C) Plain radiographs are often normal in the first two to four weeks after the injury. Repeating the x-ray after one week can still give false negative results. (Choice D) An ultrasound of the right lower extremity is useful in localizing traumatic hematoma or deep venous thrombosis. It is not useful in diagnosing stress fractures. Educational Objective: Magnetic resonance imaging or triple phase bone scans with technetium are highly accurate in the diagnosis of early stress fractures. 43% of people answered this question correctly

Item 2 of 2 The diagnostic study performed on the right lower extremity reveals an undisplaced distal tibial stress fracture. Which of the following is the most appropriate next step in the management of this patient? B. C. D. A. Start her on calcium and vitamin D supplementation. Conservative management with nonweightbearing and pain control. Advise her to change her shoes while running. Orthopedic referral for open reduction and internal fixation.

This subscription is licensed to user: roopika only User ID: 123489 Explanation: Most of the stress fractures of the lower extremities can be managed with conservative treatment. This includes complete cessation of aggravating activities for at least four to

six weeks, with a gradual return to activity. Other general measures include pain control with nonsteroidal antiinflammatory drugs and support with crutches or a brace while walking. (Choice A) There is currently no evidence that calcium and vitamin D supplementation plays a role in the prevention of stress fractures. There is no history of osteoporosis in this patient. Moreover, regular exercise or running is helpful in reducing the risk of osteoporosis. (Choice C) Advising the patient on the proper shoe size and fit is important in preventing future injuries to the lower extremities; however, she should also be advised to discontinue her running activities for at least four to six weeks. (Choice D) Surgical correction with open reduction and internal fixation is not required in majority of stress fractures. Adequate rest with pain control and a gradual return to activity are usually sufficient in mild cases. Educational Objective: Stress fractures can be managed conservatively by adequate rest for at least four to six weeks, pain control, and a gradual return to activity. 92% of people answered this question correctly;

A 24-year-old Colombian-American woman is brought to the emergency room because of severe abdominal pain. She is 30 weeks pregnant, and has no other medical problems. The pain started 24 hours ago in the epigastrium, but the pain is now felt over the lower abdomen. She also feels nauseated, and has not eaten in the last 12 hours. Her temperature is 38.2 C (100.8 F), blood pressure is 100/70 mmHg, pulse is 100/min and respirations are 22/min. Examination reveals a gravid uterus and increased fetal movements. There is severe tenderness of the right lower quadrant (RLQ) on palpation, which is also elicited through leg elevation and left lower quadrant pressure. The laboratory tests show the following:Hb 12.86 g/dL Ht 39% Platelet count 260,000/cmm Leukocyte count 16,000/cmm Segmented neutrophils 75% Bands 9% Lymphocytes 16% Which of the following complications is more likely to occur if the patient is not adequately treated? B. A. Bacterial endocarditis Bacterial meningitis

C. D. E.

Pancreatitis Pylephlebitis Tubo-ovarian abscess

This subscription is licensed to user: roopika only User ID: 123489 Explanation: This patient has the characteristic signs and symptoms of acute appendicitis. McBurney's sign (tenderness at McBurney's point, the site two-thirds between the umbilicus and right anterior superior iliac spine), Rovsing's sign (transmission of pain from the left to the right lower quadrant), and iliopsoas sign (RLQ pain on passive extension of the right hip) are all positive. There is a left shift in the white blood cell count (true for 95% of diagnosed cases). In a third trimester pregnant woman, there is a minimal risk of premature delivery; however, if rupture of the appendix occurs, peritonitis can lead to fetal death, abscess formation and pylephlebitis (infectious thrombosis of the portal veins). These complications are rare but feasible, especially if therapy is not offered within the first 24 hours of the initiation of symptoms. (Choice C) Pancreatitis is a common complication of biliary tract obstruction, not appendicitis. (Choices A and B) Bacterial endocarditis or meningitis can occur in extreme cases of sepsis, but their association with appendicitis is very uncommon. (Choice E) Appendicitis will not cause a tubo-ovarian abscess. Educational Objective: Around 14% of pregnant women in the second trimester can have premature delivery when affected by appendicitis, while around one third of those in the first trimester may experience abortion. During the third trimester, the main complication is appendix perforation with peritonitis and subsequent pylephlebitis. 39% of people answered this question correctly

A 28-year-old Caucasian male presents to your office with enlargement of his right breast. A rubbery disk of tissue is palpated directly beneath his areolar area. You give a diagnosis of gynecomastia, and start the work-up to determine the cause of this condition. Which of the following causes of gynecomastia will increase the risk of breast cancer in this patient the most ? B. C. D. A. Liver disease Exogenous estrogens Hyperthyroidism Klinefelter's syndrome

E.

Secondary hypogonadism

This subscription is licensed to user: roopika only User ID: 123489 Explanation: Klinefelter's syndrome is a chromosomal abnormality caused by an additional X chromosome (47, XXY). It is characterized by hypogonadism, low testosterone levels, and gynecomastia due to an increased testosterone-to-estrogen ratio. Klinefelter's syndrome is the strongest known risk factor for male breast cancer. It carries a 50-fold increase in the risk of breast cancer compared to men with a normal genotype. (Choices A, C, and E) Endogenous hormone abnormalities caused by liver disease, malnutrition, hyperthyroidism, or secondary hypogonadism have an uncertain relationship with male breast cancer. (Choice B) Exogenous hormone administration does not seem to increase the risk of breast cancer significantly. Educational Objective: Klinefelter's syndrome is the strongest known risk factor for male breast cancer. It carries a 50-fold increase in the risk of breast cancer compared to men with a normal genotype. 48% of people answered this question correctly;

A 50-year-old Caucasian man arrives to the emergency department complaining of severe abdominal pain and an inability to void urine since the day before. He has no significant medical history and denies the use of any medications. There is no history of recent trauma or fever. He looks restless and is in moderate distress. The physical examination reveals moderate tenderness on palpation of the suprapubic area, as well as an enlarged, smooth prostate found through rectal exam. Laboratory evaluation showed a creatinine of 2.9 mg/dL, with a BUN of 57 mg/dL, a hematocrit of 51%, and hemoglobin of 13.8 mg/dL. What should be the next step in the management of this patient? B. C. D. E. A. Suprapubic catheterization after bladder emptying Immediate urology and renal service consultation Immediate suprapubic catheterization Immediate urethral catheterization Immediate PSA determination

This subscription is licensed to user: roopika only User ID: 123489 Explanation:

This patient is developing post-obstructive, acute renal failure, probably secondary to benign prostatic hypertrophy. The most urgent procedure that should be done is bladder decompression to alleviate pain and to avoid further renal deterioration. This can be readily accomplished through gentle urethral catheterization with a Foley catheter. (Choices C) Suprapubic catheterization by the urologist is only needed if there is evidence of urethral or pelvic trauma, or if urethral catheterization is unsuccessful. (Choice A) Suprapubic catheterization should never be done on an empty bladder because of the risk of bowel perforation. (Choice B) No renal or urologic consultation needs to be done before bladder decompression is attempted. The patient's renal function is expected to improve dramatically after the obstruction is removed using catheterization. (Choice E) PSA level determination is useless in this setting since the PSA level will be invariably increased due to the urinary retention, the catheterization procedure, as well as any vigorous rectal examination. Educational Objective: Any case of acute urinary obstruction in which there is no evidence of pelvic or urethral trauma, or no history of urethral strictures, must be managed immediately through urethral catheterization in order to decompress the bladder, alleviate the pain, and avoid further renal compromise. Suprapubic catheterization should never be done on an empty bladder, and should preferrably be performed by the urologist when the aforementioned conditions are present, or if Foley catheter placement is difficult. 73% of people answered this question correctly

A 58-year-old, obese, Caucasian male presents to the emergency department immediately after a motor vehicle accident in which he was a restrained passenger. His medical history is significant for diabetes mellitus type 2, hypertension, gastroesophageal reflux disease, erectile dysfunction, and gout. His current medications include metformin, rosiglitazone, lisinopril, and omeprazole. He drinks 1-2 beers daily and denies the use of tobacco or recreational drugs. His temperature is 36.9 C (98.4 F), blood pressure is 134/88 mmHg, pulse is 86/min, and respirations are 15/min. Normal S1 and S2 heart sounds are present, with no murmurs or rubs. The lung fields are clear. The abdomen is obese, nontender and nondistended; bowel sounds are present. Brown stool in the rectal vault is guaiac negative. The rectal sphincter tone is normal. There is no evidence of clubbing, cyanosis, or edema in the extremities. Dorsalis pedis and posterior tibial pulses are present and equal bilaterally. On neurologic exam, the patient is awake, alert, and oriented. Cranial nerves II-XII are intact. Motor strength is 5/5 in all extremities. Finger to nose cerebellar exam is well done. Reflexes are 1+ at the ankles and 2+ at the knees, biceps, and triceps. The cremasteric reflex is absent. Sensation is intact throughout, except for a bilateral symmetric reduction in perception of vibration, pain, and

temperature in both feet and hands. Plantar flexion and dorsiflexion are normal. Ambulation is normal, and the Romberg sign is negative. What is the most likely etiology of this individual's absent cremasteric reflex? B. C. D. E. A. Cauda equina syndrome Injury to the spinal cord at L1-L2 Injury to the spinal cord at L5-S2 Injury to the spinal cord at S2-S4 Diabetic neuropathy

This subscription is licensed to user: roopika only User ID: 123489 Explanation: The loss of the cremasteric reflex in this patient is secondary to his diabetes mellitus. The presence of distal symmetric polyneuropathy (in the classic "stocking-glove" pattern) and history of erectile dysfunction are suggestive of significant diabetic neuropathy. Other possible presentations of neurogenically induced erectile dysfunction in the diabetic male include diminished testicular sensation, bladder dysfunction, and inability to masturbate. (Choice A) The symptoms of cauda equina syndrome include severe lower back pain, urinary or bowel incontinence, motor weakness or sensory loss in the legs bilaterally, and saddle anesthesia. (Choice B) The cremasteric reflex corresponds to the L1-L2 level of the spinal cord, a region also responsible for hip flexion and adduction. The patient's normal ambulation and motor strength indicate that an injury to the spinal cord at L1-L2 is unlikely. (Choice C) The normal dorsiflexion and plantar flexion in this patient signify functionality of the L5-S2 region of the spinal cord. (Choice D) The normal anal sphincter tone in this patient signifies functionality of the S2-S4 region of the spinal cord. Educational Objective: The cremasteric reflex is regulated at the L1-L2 level of the spinal cord. This reflex can be diminished or lost secondary to diabetic autonomic neuropathy. 58% of people answered this question correctly;

A 25-year-old male is brought to the emergency room after a fight in a bar. He was attacked with a knife, and in an attempt to save himself, he grabbed the knife in his right hand. He has a bleeding wound in his right hand. On examination, a deep lacerated wound is found on the palmar surface of the right hand, extending from the index finger at the metacarpophalangeal (MCP) joint through the proximal phalanges of the middle

and ring fingers to the middle phalanx of the little finger. He is unable to flex the distal phalanges of these fingers. Which of the following structures is most likely to be injured in his hand? A. B. C. D. Nerves Arteries Veins Tendons

This subscription is licensed to user: roopika only User ID: 123489 Explanation: Digital arteries, nerves, and veins run on the sides, while the flexor tendons run on the anterior surface of the phalanges. Attempting to grab a knife with a hand is likely to injure the tendons than arteries, veins, or nerves due to their relative, vulnerable, anatomic location. Educational Objective: In digital injuries, tendons are more likely to be injured than arteries, veins, or nerves due to their relative, vulnerable, anatomic location. 89% of people answered this question correctly;

An active 46-year-old nurse injured her left knee while transferring a patient. She felt a "twinge", but there was no immediate deformity or subsequent edema, and she was able to continue walking normally. Within two weeks, she developed intense pain around the left knee and could neither walk, nor put any weight on the left leg. Placing clothing or sheets on the left knee produced abnormal discomfort and insomnia. She consulted a physician who obtained a negative knee x-ray and MRI, prescribed vicodin and naproxen, and said, "You are over 40, you're going to have aches and pains." Over the next two months, the patient noted swelling around her left knee, an increased temperature, and more sweating of her left leg compared to the right. Even on analgesics, exacerbation of her knee pain by flexion and extension caused her to limp. After three more months, the skin over her left knee has a mottled, bluish discoloration. Which of the following measures taken during the first three months after this patient's injury might best have improved her present prognosis? A. Immobilization with an above knee cast B. Intravenous antibiotics C. Arthroscopic removal of synovial plica D. A prescription for oral prednisone beginning with a full dose, then tapered over 2 - 3 weeks E. A prescription for oral phenoxybenzamine

This subscription is licensed to user: roopika only User ID: 123489 Explanation: This patient meets the International Association for the Study of Pain Diagnostic Criteria for Complex Regional Pain Syndrome (CRPS) Type I, also known as Reflex Sympathetic Dystrophy (RSD). These are: 1. The presence of an initiating noxious event or a cause of immobilization. 2. Continuing pain, allodynia (pain elicited by a normally non-noxious stimulus, particularly if repetitive or prolonged) or hyperalgesia (pain disproportionate to the inciting event) 3. Evidence at some time of edema, changes in skin blood flow, or abnormal sudomotor activity in the painful region. 4. Absence of other conditions that could account for the degree of pain and dysfunction. At least one half of the patients meeting these diagnostic criteria have sympathetically maintained pain (SMP), which is defined as pain that is maintained by sympathetic efferent nerve activity or by circulating catecholamines. Normally, the sympathetic reflex after injury results in vasoconstriction to prevent excessive blood loss or swelling. Ordinarily, this reflex shuts off after an appropriate period. In the patient who develops CRPS Type I with SMP, the normal sympathetic reflex does not shut off, but continues unabated in a pathologic positive feedback mechanism. The intense vasoconstriction causes localized ischemia, which in turn causes pain that further initiates the sympathetic "pain reflex," etc. Pharmacologic sympathetic blockade or sympathectomy may interrupt this vicious cycle and reduce long-term periarticular skin and muscle atrophy, flexion contractures, and osteoporosis. There is evidence that such treatment for CRPS- I with SMP is most or only effective during the first three months after onset. In CRPS-I of the knee, the most favorable prognostic indicators are early diagnosis, early interruption of the abnormal sympathetic activity (e.g., via alpha blockers, chemical or surgical sympathectomy), and early institution of aggressive physical therapy). (Choice A) This patient has no evidence of a fracture or soft tissue injury, that might require immobilization via an above knee cast after reduction or repair. In fact, immobilization can promote CRPS Type I and is contraindicated in this patient. (Choice B) This would be a reasonable answer if the patient had a pyogenic arthritis of the left knee. Unlike the presentation in this case, acute pyogenic arthritis usually presents with a tense joint effusion, and the patient generally cannot tolerate weightbearing or joint motion. The presence of allodynia and abnormal unilateral sweating around the patient's knee joint also make this diagnosis less likely. (Choice C) Plica syndrome of the knee is pain, crepitus, snapping, popping, or effusion related to patellofemoral joint motion. The clinical picture mimics a torn medial meniscus or a maltracking patella, and is due to a prominent medial plica (synovial fold) which impinges against the medial patellofemoral articular surface and may become inflamed. It

is not associated with the allodynia or sudomotor and vasomotor changes seen in this patient. (Choice D) The efficacy of corticosteroids in CRPS Type I is controversial. A trial of steroid therapy might be reasonable for a patient with long-standing pain and who has failed to respond to sympathetic blockade. Educational Objective: Pharmacologic sympathetic blockade or sympathectomy in Complex Regional Pain Syndrome Type I (reflex sympathetic dystrophy) is most effective if initiated during the first three months after the onset of symptoms. 7% of people answered this question correctly;

A 45-year-old male was involved in an argument over a drug deal. He ended up being shot in the head and abdomen. He was immediately intubated and taken to the operating room for an exploratory laparotomy. Postoperatively, he remained on the ventilator and was started on total parenteral nutrition. Seven days later, he developed a sudden onset of glucose intolerance. What does this most likely indicate? B. C. D. E. A. Liver dysfunction Sepsis Hypophosphatemia Bowel atrophy Carbon dioxide retention

This subscription is licensed to user: roopika only User ID: 123489 Explanation: Numerous complications can occur from intravenous nutrition in critically ill patients. The most dangerous metabolic complication, hyperglycemia, is most commonly caused by too rapid initiation of the infusion. This complication is best prevented by initiation of the infusion at 40-60 ml per hour, and slowly increasing the rate at 20 cc/hour every 24 hours, while monitoring the patient's glucose levels. Patients with normal glucose tolerance may manifest glycosuria for the first 48 hours of parenteral nutrition. Before initiating insulin, one must verify that the glucose level is high, and that glycosuria is not secondary to a reduced renal threshold for glucose. The most common cause of sudden hyperglycemia is sepsis, and hyperglycemia may antedate other signs of sepsis within 24 hours. The sudden appearance of hyperglycemia should initiate a thorough search for the source of infection. The infection may be due to line sepsis, pneumonia, wound infection, or another process. Blood cultures and a thorough examination of the patient are required.

(Choice A) A fatty liver can be seen when glucose infusions exceed the daily caloric requirements. This is because of the production of fatty acids from excess glucose and the impaired ability to mobilize the fat for energy needs. Fat accumulation eventually leads to abnormal liver enzyme elevations in the serum, particularly alkaline phosphatase. The long-term consequences of fatty liver are not clear, but at present, there seems to be no serious sequelae. (Choice C) Hypophosphatemia is reported to occur in 30% of patients receiving intravenous nutrition. The mechanism is enhanced phosphate uptake into cells associated with enhanced glucose uptake. The consequences of hypophosphatemia include respiratory weakness, hemolysis, and impaired oxygen release from hemoglobin. These may be particularly prominent when serum phosphate falls below 1.0 mg/dL. Phosphate supplementation in the diet will prevent these complications. (Choice D) Bowel rest leads to degenerative changes in the small bowel mucosa only after a few days, and intravenous nutrition does not prevent these atrophic changes. Bowel atrophy during intravenous nutrition is now considered a major risk, because disruption of the normal mucosal barrier might allow intestinal microbes to enter the systemic circulation. (Choice E) Carbon dioxide retention can develop when excess glucose is given to patients with severe lung disease. Glucose metabolism produces a larger quantity of CO2 for each liter of O2 consumed than the other two nutrient substrates. When the ability to eliminate CO2 via alveolar ventilation is impaired, this enhanced CO2 production of carbon produces hypercapnia, and the impaired ability to wean from mechanical ventilation. Educational Objective: The sudden onset of hyperglycemia in a patient receiving total parenteral nutrition should lead to a suspicion of sepsis. 32% of people answered this question correctly

A 65-year-old white female presents to the emergency department with a sudden onset of severe periumbilical pain, nausea and vomiting. She denies any fever, diarrhea, or constipation. She denies any urinary complaints. Her last menstrual period was fourteen years ago. Her past medical history is significant for coronary artery disease, a recent acute anterior myocardial infarction, hypertension, and hyperlipidemia. She had an appendectomy at the age of eighteen. She has a history of 20 pack years of smoking. She is an occasional alcohol drinker. Her temperature is 36.7C(98F), blood pressure is 120/76 mm Hg, pulse is 120/min, irregular, and respirations are 16/min. The abdomen is soft, non-distended and non-tender, with normal bowel sounds. There is no rebound tenderness or rigidity. Pelvic examination is within normal limits. EKG shows atrial fibrillation. CBC shows a WBC count of 14,000/microL. Serum sodium is 140 mEq/L, serum K is 4.0 mEq/L, blood glucose is 110mg/dL, BUN is 20 mg/dL, and serum

creatinine is 0.7 mg/dL. The serum lipase is normal, while the serum amylase is slightly elevated. Urinalysis is normal. Liver function panel is also normal. Fecal occult blood test is negative. Plain abdominal x-ray fails to show any abnormalities. Which of the following is the most likely diagnosis of this patient? B. C. D. E. A. Intestinal obstruction Acute mesenteric ischemia Acute colonic ischemia Acute intermittent porphyria Acute diverticulitis

This subscription is licensed to user: roopika only User ID: 123489 Explanation: This patient is most likely suffering from acute mesenteric ischemia secondary to embolic phenomenon caused by atrial fibrillation. This patient also had a recent anterior myocardial infarction. Embolization usually throws clots into the superior mesenteric artery, resulting in mesenteric ischemia. Patients usually present with a sudden onset of periumbilical abdominal pain. The abdominal examination is initially normal, and the pain felt by the patient is usually out of proportion to the initial examination findings. (Choice A) Patients with intestinal obstruction usually have a history of absolute constipation. Plain abdominal radiographs will show air fluid levels. (Choice C) Patients with acute colonic ischemia usually have a more lateralized abdominal pain with tenderness, along with a history of bloody diarrhea or hematochezia. This usually results from hypovolemic states or transient ischemia to the bowel. This is not because of the obstruction of flow to the colon. (Choice D) Acute intermittent porphyria is a rare disease. Patients will usually have recurrent episodes of abdominal pain that is precipitated by several factors. The symptoms are usually manifested much earlier (usually after puberty), and occur in the presence of other manifestations such as neuropathy, hyponatremia, etc. (Choice E) Acute diverticulitis usually causes a more localized pain (LLQ). The physical exam would show tenderness in the LLQ. Patients may also present with diarrhea. Educational Objective: A sudden onset of severe periumbilical pain that is out of proportion to the physical examination findings, and occurs in a setting with risk factors such as atrial fibrillation, congestive heart failure, and peripheral vascular disease, should always raise the suspicion for acute mesenteric ischemia. 88% of people answered this question correctly;

The following vignette applies to the next 2 items A 40-year-old Caucasian female is brought to the emergency department (ED) by the paramedics after she was involved in a motor vehicle accident. She was the driver of the vehicle and was wearing a seatbelt. Her car was hit by another car coming from the opposite side, and then went off the road and hit a sideline tree. When the medics arrived at the scene of the accident, she was alert, oriented and complained of mild chest pain and shortness of breath. In the ED, she tells you that she has had a dry cough for the past week, but denies any associated fever, chills, difficulty breathing or expectoration with the cough. She does not smoke or drink alcohol. Her temperature is 36.7 C (98F), blood pressure is 126/82 mmHg, respirations are 22/min, and heart rate is 106/min. Physical examination reveals an area of tenderness over the left chest wall where the seatbelt was placed. Her lung examination reveals decreased breath sounds over the left side. An initial EKG reveals sinus tachycardia. The chest radiograph shows a bilateral homogenous opacification of the upper zones of the lung, more pronounced on the left side. Item 1 of 2 What is the most likely diagnosis based on the patient's clinical presentation? B. C. D. E. A. Pulmonary edema Pneumonia Pulmonary contusion Pulmonary infarction Pleural effusion

This subscription is licensed to user: roopika only User ID: 123489 Explanation: Based on the clinical presentation, this patient most likely suffered from a pulmonary contusion during the accident. It is the most common lung parenchymal injury in patients with blunt chest trauma from any cause. The forces associated with blunt trauma are transmitted to the lung parenchyma and results in contusion with hemorrhage into the lung tissue. The clinical presentation depends on the extent of lung injury. The patients usually present with varying degrees of dyspnea, tachypnea, hypoxemia, and hemoptysis. Physical examination reveals decreased breath sounds over the affected area of the lungs. The diagnosis of pulmonary contusion may be missed initially, as it may take several hours for clinical and radiographic abnormalities to develop after the initial trauma. Chest radiography generally reveals homogenous opacification of the lung fields that do not confirm to a specific anatomic segment of the lung. Treatment is mainly supportive and the resolution of symptoms generally occurs in three to five days in mild cases.

(Choice A) Patients with pulmonary contusion are prone to develop pulmonary edema with overly aggressive fluid resuscitation; however, the patient's clinical presentation is not consistent with pulmonary edema at this point. (Choices B, D & E) The clinical and radiographic presentation in this patient is not consistent with a diagnosis of pneumonia, pulmonary infarction or pleural effusion. Educational Objective: Pulmonary contusion is the most common lung parenchymal injury seen in patients with blunt chest trauma. Chest radiography generally reveals homogenous opacification of the lung fields that do not confirm to a specific anatomic segment of the lung. 87% of people answered this question correctly

Item 2 of 2 Which of the following is the most appropriate management for this patient? B. C. D. E. antibiotics A. Admit her to the hospital and start intravenous antibiotics Admit her to the hospital and monitor for 24 to 48 hours Obtain a V/Q scan Admit her to the hospital and start diuretics and fluid restriction Admit her to the hospital, perform a thoracentesis and start her on

This subscription is licensed to user: roopika only User ID: 123489 Explanation: The full clinical and radiographic features of pulmonary contusion may take several hours to develop after the initial blunt injury. Since the patients are at risk for late clinical deterioration, they should be admitted to the hospital after the initial injury and monitored for 24 to 48 hours for any signs of clinical deterioration. The symptoms of mild pulmonary contusion usually resolve within three to five days. In the initial phase, management is generally conservative with aggressive pulmonary toilet, supplemental oxygen, pain control and careful fluid management to prevent worsening edema in the injured lung. In patients with significant lung parenchymal damage, endotracheal intubation and mechanical ventilation may be necessary to provide respiratory support until the resolution of parenchymal injury. (Choice C) The patient should be admitted to the hospital for continuous monitoring; however, there is no indication to start the patient on intravenous antibiotics or obtain a V/Q scan.

(Choice D) Diuretic and fluids restriction have no role in the management of patients with pulmonary contusion. (Choice A) Thoracentesis is indicated only in patients with traumatic hemothorax or pneumothorax causing respiratory compromise. It has no role in the management of patients with pulmonary contusion. Educational Objective: All patients with significant blunt chest trauma and pulmonary contusion should be admitted to the hospital and monitored for signs of clinical deterioration. 68% of people answered this question correctly A 54-year-old Indian female comes to your office because of dysphagia. She emigrated from India two years ago. She is the mother of a computer engineer, and lived her whole life in an iodine-deficient area in India. She denies any past medical problems, and is currently on no medications. For the past five months, she has been having some difficulty in swallowing large chunks of food, which seems to be progressively getting worse. She has no difficulty in breathing or swallowing liquids. Her weight is constant. She denies fever, cough, chest pain, abdominal pain, heat intolerance, sweating, or tremors. She is a vegetarian, and does not smoke or drink. She has never undergone any mammography or Pap smear screening. Her last menstrual period was four years ago. She has minimal hot flashes, and was never on hormone replacement therapy. Her family history is significant for a father with type-2 diabetes mellitus. Examination of her neck reveals a short neck with a very low lying thyroid; its lower margin could not be felt. The upper one-third of the sternum is dull on percussion. Her face becomes plethoric after raising her arms for one minute. There is no lymphadenopathy. The rest of the physical examination is normal. TSH is 0.78 U/mL (normal 0.35-5.0 U/ml), free T4 is 1.3 U/ml (normal 0.8-1.8 U/dl), and antithyroid peroxidase antibody is negative. Chest xrays reveal a large upper mediastinal mass that is shifting the trachea to the left side. A CT scan of the chest reveals that the upper retrosternal mass is consistent with a large multinodular retrosternal thyroid gland. A barium swallow shows mild compression of the esophagus by the goiter. What is the best option for the management of this patient? B. C. D. E. A. Surgical removal of the retrosternal goiter External beam radiation Radioactive iodine ablation Antithyroid drugs Levothyroxine

This subscription is licensed to user: roopika only User ID: 123489 Explanation: The patient has a large retrosternal goiter that is causing compressive symptoms. Surgical removal (preferably via cervical approach) is recommended. The goiter is probably due

to iodine deficiency. Such goiters are usually multinodular. The thyroid has enlarged preferentially in the patient's thorax due to her short neck. (Choice D) The patient is euthyroid. Antithyroid drugs are not indicated. Antithyroid drugs (PTU and methimazole) may even lead to a slight enlargement of the thyroid, and are therefore not preferred over surgery in patients with retrosternal goiters and compressive symptoms. (Choice C) Radioactive radioiodine ablation can also result in some enlargement of the thyroid, which can increase the compressive symptoms. (Choice B) External beam radiation has no role in the treatment of retrosternal goiter. (Choice E) Levothyroxine may be used in the treatment of multinodular goiters; however, results are usually unsatisfactory. Full resolution of the goiter is uncommon. Furthermore, longstanding multinodular goiters usually have fibrosis and degenerative changes, and are unlikely to respond to levothyroxine. Levothyroxine can potentially lead to thyrotoxicosis, and increases the risks for bone loss and atrial fibrillation. Educational Objective: Management of retrosternal goiter with compressive symptoms is surgical. 77% of people answered this question correctly

A 16-year-old Caucasian boy with Down syndrome is brought into the emergency department by ambulance after being in a serious motor vehicle accident. He is unconscious, has unstable vital signs, and has sustained multiple lacerations to his face, arms, and abdomen. His adult sister is present, having witnessed the accident. She mentions that their parents are on a cruise and cannot be contacted. What is the next best step in managing this boy's care? B. C. D. E. A. Provide treatment without seeking consent Seek consent from the sister Seek consent from the grandparents Petition the court for consent Withhold treatment until consent cannot be obtained from parents

This subscription is licensed to user: roopika only User ID: 123489 Explanation: Normally, the parents or legal guardian of a minor must provide voluntary, informed consent for treatment and most medical tests or procedures. However, there are exceptions to this rule, including the allowance that is made in case of emergency. If

immediate medical care is necessary to prevent serious harm or death, consent is always assumed. Therefore the emergency department personnel should provide treatment for this boy without seeking consent (Choice A). Consent from the sister (Choice B) or grandparents (Choice C) is unnecessary and not of legal benefit, since they are not the legal guardians of the boy. Petitioning the court for consent (Choice D) would take far too long in an emergency setting. Moreover, the court is asked to intervene primarily when the parents are refusing treatment recommended by physicians. In this case, the parents have not refused treatment. They are simply unavailable to provide explicit consent. Withholding treatment until consent cannot be obtained from parents (Choice E) may lead to the death of the boy. Since an exception to the informed consent rule is provided in case of emergency, treatment should proceed. Educational Objective: Normally, the parents or legal guardian of a minor must provide voluntary, informed consent for treatment and most medical tests or procedures. However, if immediate medical care is necessary to prevent serious harm or death, consent is always assumed. 70% of people answered this question correctly;

TEST 3 A 60-year-old Caucasian female presents to the emergency department and complains of a severe, sudden onset headache accompanied by nausea. She has had mild diplopia for the past three months. Her past medical history is significant for a long history of hypertension and diabetes mellitus type-2. Her blood pressure is 160/90 mmHg and heart rate is 80/min. Physical examination reveals right-sided ptosis, mild anisocoria, and nuchal rigidity. Which of the following is the most likely diagnosis? B. aneurysm C. aneurysm D. E. A. Trigeminal neuralgia Subarachnoid hemorrhage due to posterior communicating artery Subarachnoid hemorrhage due to posterior inferior cerebellar artery Brainstem glioma Lacunar stroke

This subscription is licensed to user: roopika only User ID: 123489 Explanation:

The clinical scenario described is suggestive of a subarachnoid hemorrhage. The constellation of sudden onset headache, nausea, and nuchal rigidity is characteristic. The patient demonstrates the signs of right cranial nerve III palsy (ptosis and anisocoria), a typical finding in patients with a posterior communicating artery aneurysm, which accounts for about of cerebral aneurysms. It is also characteristic for cavernouscarotid aneurysms, but these are uncommon. (Choice C) Posterior inferior cerebellar artery aneurysms frequently result in ataxia and bulbar dysfunction. (Choice D) Brainstem glioma presents with signs and symptoms of progressive brainstem involvement as ataxia, cranial nerve palsies, motor, and sensory abnormalities. (Choice E) Lacunar stroke manifests as relatively limited neurologic dysfunction (e.g., 'pure motor stroke'), but headache and nuchal rigidity are not present. Educational Objective: Cranial nerve III palsy is a typical finding in patients with a posterior communicating artery aneurysm. 59% of people answered this question correctly A 52-year-old Caucasian male comes to your office for a full body check-up. His vital signs, respiratory and cardiac examinations are all normal. His chest x-ray is normal, but his abdominal x-ray shows a curvilinear calcification in the right hypochondrium. This corresponds to the location and shape of the gallbladder, along with multiple gallstones. These findings are confirmed with a CT scan. He has no complaints of abdominal pain or intolerance to fatty foods. What would be the next best intervention for this patient? B. C. D. E. A. Do a biopsy of the gallbladder Prescribe ursodeoxycholic acid Do nothing at this time, as the patient is having asymptomatic gallstones Check the serum calcium levels along with a parathyroid screen Schedule an elective cholecystectomy

This subscription is licensed to user: roopika only User ID: 123489 Explanation: This patient has asymptomatic gallstones and porcelain gallbladder. Porcelain gallbladder is a condition characterized by calcium salt deposits in the wall of a chronically inflamed gallbladder. The calcifications can be thin or faintly visible, or may be amorphous, patchy, and thick. The gallbladder is generally large, but its size can vary considerably. Most porcelain gallbladders are associated with gallstones. A plain radiograph generally detects these, but computed tomography (CT) has a higher specificity; therefore, a CT scan is performed to confirm the diagnosis. Due to their high risk of gallbladder

carcinoma, all patients with porcelain gallbladder should have an elective cholecystectomy. (Choice A) Biopsy of the gallbladder is not needed. A CT scan has high sensitivity and specificity for the diagnosis of a porcelain gallbladder. (Choice B) Ursodeoxycholic acid is the medical treatment for patients with symptomatic gallstones who are poor surgical candidates. It helps to dissolve the stones, but this will not decrease the risk of carcinoma in patients with porcelain gallbladder. (Choice C) Although asymptomatic gallstones need not be treated, the presence of a porcelain gallbladder is a definite indication for a cholecystectomy, even in patients with asymptomatic gallstones. (Choice D) Calcification in this patient is not due to hypercalcemia but due to chronic inflammation. Educational Objective: All patients with porcelain gallbladder should have a cholecystectomy due to the high risk of gallbladder carcinoma. 41% of people answered this question correctly

A 29-year-old Caucasian male presents to the emergency department complaining of multiple lacerations after being attacked by a dog one hour prior to arrival. He states that he was jogging through an open field when the dog ran over to him and began to bite and scratch at his lower extremities, which caused the man to fall to the ground. According to the dog's owner, the dog has been vaccinated recently against rabies and became very territorial after birthing a litter of puppies last week. Animal Control has retrieved the dog for observation. On physical examination, four prominent bite wounds are visible on the man's left calf, as are numerous lacerations on both the left and right calves. The bite wounds measure approximately 2 cm long, are bleeding freely, and contain some dirt and plant material. The man is uncertain about his childhood immunization history and says, "it has been at least fifteen years since I've gotten a shot." Once his wounds are properly cleaned, how should his condition be managed? A. Administer tetanus toxoid B. Administer tetanus immune globulin C. Administer tetanus toxoid and tetanus immune globulin D. Administer tetanus toxoid and prescribe amoxicillin/clavulanate E. Administer tetanus toxoid and tetanus immune globulin and prescribe amoxicillin/clavulanate This subscription is licensed to user: roopika only User ID: 123489

Explanation: A patient who has sustained an animal bite should be treated with local wound management, rabies prophylaxis, tetanus prophylaxis, and measures to prevent or control bacterial infection. In this patient, post-exposure rabies prophylaxis is not warranted as the dog who attacked him has allegedly been vaccinated and is being held under observation. Tetanus prophylaxis, on the other hand, is necessary, especially since he cannot recall his immunization status, and he attests to not receiving any injections for at least fifteen years. In such a case, prophylaxis includes the administration of active (tetanus toxoid) and passive (tetanus immune globulin) immunization. A second dose of tetanus toxoid must be given in one month, and a third dose in twelve months. With regard to antibiotic therapy, amoxicillin/clavulanate remains the drug of choice for prophylaxis in patients who have sustained dog bites. It should be prescribed for a period of five days, presuming the wound remains uninfected. Administering tetanus toxoid and prescribe amoxicillin/clavulanate (Choice D) would be appropriate if the man knew his childhood immunization status and could confirm he had received the primary tetanus series. Since he is unsure, he must be given tetanus immune globulin as well. Educational Objective: Cat and dog bites should be treated prophylactically with amoxicillin/clavulanate. Tetanus toxoid must be given to individuals with dirty wounds who have received a booster >5 years ago and to individuals with clean wounds who have received a booster >10 years ago. Tetanus immune globulin should be given to any individual with a dirty wound and an unclear or insufficient immunization history. 69% of people answered this question correctly

A 62-year-old non-smoker female comes to your office because of malaise, weight loss, and epigastric discomfort for the past month. Her past medical history is significant only for hypertension. Her physical examination is unremarkable, except for mild midepigastric tenderness. Her laboratory test results are as follows:Total bilirubin 0.9 mg/dL AST 36 IU/L ALT 32 IU/L Alkaline phosphatase 60 IU/L Serum albumin 3.5g/dL PT 10 sec Hemoglobin 12.5 g/dL WBC count 6,000/cmm

A CT scan of the abdomen shows an 8 cm loculated cystic lesion in the head of the pancreas. What is the next best step in the management of this patient? B. C. D. E. A. Serum CEA and CA 19-9 estimation Endoscopic retrograde cholangiopancreatography Follow up of patient with a CT scan every three months Surgical resection of the cystic lesion Percutaneous needle aspiration of the lesion

This subscription is licensed to user: roopika only User ID: 123489 Explanation: The presence of a loculated pancreatic cyst in an elderly patient with complaints of malaise and weight loss is suggestive of a cystic neoplasm of the pancreas. The patient has no past medical illnesses, and this rules out other diagnoses such as past pancreatitis or hepatobiliary disorders. A simple cyst is ruled out by the loculated appearance on CT scan. This loculated cyst in the pancreas is most likely a cystic adenocarcinoma or a mucinous cystic adenoma, and should be treated surgically. (Choice A) Serum CEA is positive in about 40% of patients of pancreatic carcinomas, and CA 19-9 is positive in about 60-70% of patients with pancreatic carcinoma; however, both markers are non-specific and are not required or confirmatory for the diagnosis of pancreatic cancer. (Choice B) ERCP is useful to study the hepatobiliary tract; however, in this case, it will not add any more information, and it cannot be used therapeutically in this condition. (Choice C) Pancreatic cancer is a very malignant cancer and causes death in 98% of afflicted patients; therefore, an interventional approach should be taken if a cure is expected. (Choice E) Percutaneous aspiration would lead to seeding of the cancer cells in the tract and recurrences (very common). Educational Objective: A multilocular pancreatic cyst in a patient without a history of pancreatitis should be considered malignant until proven otherwise. 26% of people answered this question correctly; A 25-year-old Caucasian man comes to see you in the office because of persistent "lumps" in the neck. He recalls an episode of marked fatigue, fever, pharyngitis, and generalized lymphadenopathy about 8-10 weeks ago, when he was diagnosed with infectious mononucleosis. Most of his symptoms except the neck "lumps" resolved completely in two weeks. On physical

examination, he has three anterior cervical lymph nodes, the largest being 2 cm x 2 cm in size. They are mobile and have a firm rubbery consistency. The patient denies any associated symptoms, and the rest of his physical examination is unremarkable. Which of the following is the most appropriate next step in the management of this patient? B. C. D. E. A. Refer to a surgeon for a lymph node biopsy Prescribe a course of oral amoxicillin Refer to an otolaryngologist for fiberoptic examination of the oropharynx Obtain a chest x-ray Continue watchful observation of the patient

This subscription is licensed to user: roopika only User ID: 123489 Explanation: Infectious mononucleosis is a viral illness caused by the Epstein-Barr virus. It classically presents with marked fatigue, fever, tonsillar pharyngitis, and generalized lymphadenopathy. Acute symptoms usually resolve in one to two weeks. The infection is usually seen in adolescents and young adults, although most of them are asymptomatic. Lymphadenopathy is typically symmetric, and usually involves posterior cervical nodes. It peaks in the first week of infection, and subsides over the next 2-3 weeks. Patients with localized lymphadenopathy can be observed for 3-4 weeks. A biopsy should be performed if the abnormal nodes fail to resolve after that time, or earlier, if there are other signs of malignancy. Abnormal and persistent lymphadenopathy is sometimes the first clinical manifestation of an underlying lymphoma. This patient should be referred to a surgeon for a node biopsy. (Choice B) Empiric treatment of unexplained lymphadenopathy is not indicated. (Choice C) Hard cervical nodes, especially in older patients and in smokers, are suggestive of a malignancy. These patients should be referred to an otolaryngologist for fiberoptic examination of the oropharynx to look for a primary malignancy. (Choice D) A chest x-ray is an insensitive study and has a low yield in the absence of any specific clinical features. (Choice E) Patients with localized lymphadenopathy should be observed for three to four weeks. A biopsy should be performed if the abnormal nodes fail to resolve after four weeks. Educational Objective: Persistent and localized abnormal lymph nodes should be biopsied. 41% of people answered this question correctly

A 69-year-old male comes to the emergency department. He is complaining of a severe pain in his abdomen which started 30 minutes ago, and is gradually getting worse. The pain is characterized as 10/10 in intensity, sharp, non-radiating, non-shifting, located mainly in the periumbilical region, and with no aggravating or relieving factors known to the patient. He currently feels nauseated, and has vomited "yellowish-green stuff" once. His past medical history is significant for hypertension and recent MI, for which he underwent a coronary artery angiogram. This revealed a 100% left anterior descending artery occlusion, for which a stent was placed. Vital signs on examination are: blood pressure 110/75 mm Hg, pulse 100/min and irregular, respirations 18/min, and temperature 37.4 C(99.4 F). Only a mild tenderness is noted in the umbilical region. His EKG reveals atrial fibrillation and an old anterior infarct. Plain x-ray and CT scan of the abdomen do not reveal any abnormalities. Labs show: SerumSerum Na 139 mEq/L Serum K 4.1 mEq/L Chloride 105 mEq/L Bicarbonate 14 mEq/L BUN 22 mg/dL Serum creatinine 1.0 mg/dL Blood glucose 118 mg/dL Amylase 250 U/L Lipase 15 U/L CBCHb 15g/dL Platelet count 250,000/cmm Leukocyte count 16,000/cmm What is the most likely diagnosis of this patient? B. C. D. E. A. Small bowel volvulus Acute mesenteric ischemia Acute appendicitis Perforated peptic ulcer Acute pancreatitis

This subscription is licensed to user: roopika only User ID: 123489 Explanation: Acute onset of severe abdominal pain wherein the physical examination findings do not match the severity of pain, along with metabolic acidosis, is generally mesenteric ischemia unless proven otherwise. This condition is frequently overlooked initially. All radiological studies may be initially normal. Laboratory data may show a mild to moderate metabolic acidosis. Leukocytes and elevated Hb and Hct% in the CBC may

occur secondary to hemoconcentration. The ischemia may be due to an occlusion secondary to thrombosis, embolism and/or vasospasm. The most common cause is superior mesenteric artery (SMA) occlusion secondary to an embolism. The SMA is the most common site involved because it runs off at a minimal angle from the aorta and has a wide diameter. The origin of the embolus is commonly from the left atrium, left ventricle or cardiac valves. Risk factors are cardiac arrhythmias, recent MI and abdominal malignancy. The patient may later develop abdominal distention and signs of acute peritonitis. If not treated promptly, consequences may be very serious. Untreated mesenteric ischemia may cause bowel infarction, sepsis and death. Mortality in this case may be as high as 60%. (Choice A) The patient's presentation, abdominal examination findings, and negative xray and CT scan results make the diagnosis of an intestinal obstruction unlikely. (Choice C and D) The negative CT scan and negative x-ray rules out volvulus, acute appendicitis (no signs of inflammation on CT scan) or perforated peptic ulcer (no signs of gas seen under diaphragm). (Choice E) Serum amylase, lactic dehydrogenase (LDH), creatine phosphokinase, or alkaline phosphatase can be elevated in mesenteric ischemia. The normal lipase makes the diagnosis of pancreatitis unlikely. Educational Objective: Acute onset of severe abdominal pain wherein the physical examination findings do not match the severity of pain, along with metabolic acidosis, is generally mesenteric ischemia unless proven otherwise. Laboratory data may show elevated Hb and Hct% secondary to hemoconcentration and leukocytosis. 87% of people answered this question correctly

The following vignette applies to the next 2 items A 27-year-old male comes to the office with a scrotal mass. He first noticed this mass three months ago while taking a bath. At first, he ignored it, but he became concerned as the mass increased in size over the past few weeks. He describes this mass as "many worms in a bag." His scrotum feels heavy, and is sometimes itchy. His vital signs are within normal limits. The physical examination is normal, except for left scrotal enlargement. The mass is warm, nontender, non-fluctuant, and it does feel like a bag of worms. The left testicle is difficult to palpate. He is diagnosed with varicocele and advised to undergo surgical removal. The patient is anxious and afraid of surgery. Item 1 of 2

Which of the following is the most likely complication of untreated varicocele? B. C. D. A. Testicular atrophy Impotence Infection Development of hydrocele

This subscription is licensed to user: roopika only User ID: 123489 Explanation: The testes are located outside the abdomen, and venous drainage involves a complex network of venous channels called the pampiniform plexus. This plexus is responsible for maintaining the appropriate temperature for the scrotal sacs (i.e., approximately 2 degrees lower than body temperature). Seminiferous tubules make up most of the testicular mass, and are very sensitive to an increase in temperature. Patients with varicocele have dilatation of the pampiniform plexus, which leads to a rise in intrascrotal temperature, thereby causing the seminiferous tubules to atrophy. (Choice B) Androgen-secreting Leydig cells are more resistant to an increase in temperature caused by varicoceles; therefore, hypoandrogenism does not commonly occur with this disease. (Choice C) Intrascrotal infection is unlikely in patients with varicocele. (Choice D) Hydrocele is a collection of fluid within the potential space in the tunica vaginalis, and can be congenital (patent processus vaginalis) or acquired. Acquired causes include: epididymitis, orchitis, trauma, testicular torsion and tumor. There is no increased incidence of varicocele in patients with hydrocele. Educational Objective: Venous drainage of the testes involves a complex network of veins called the pampiniform plexus, which are responsible for keeping the temperature of the scrotal sacs below the normal body temperature. Dilatation of the pampiniform plexus (varicocele) results in an increased temperature of the scrotal sacs, thereby causing testicular atrophy. 40% of people answered this question correctly;

Item 2 of 2 Six months later, the patient comes back to your office with a right scrotal mass. The physical findings suggest a diagnosis of right varicocele. Which of the following is the most appropriate intervention?

B. C. D.

A. Surgical removal of varicocele Embolization of right testicle vein CT scan of the abdomen Scrotal ultrasonography

This subscription is licensed to user: roopika only User ID: 123489 Explanation: The testes embryologically develop intra-abdominally. Just before birth, the testes descend down to the scrotal sacs. This explains why the testicular arterial supply and venous drainage come from the abdomen. Varicoceles are more common on the left side because the left spermatic vein enters the left renal vein at a 90-degree angle. The right spermatic vein drains at a more obtuse angle directly into the inferior vena cava, thereby facilitating a more continuous flow. Processes that cause inferior vena caval obstruction (e.g., clot, tumor) should be ruled out in patients who have bilateral varicocele, right varicocele or varicocele that does not disappear in the supine position. This patient has bilateral varicocele; therefore abdominal computed tomography (CT) is warranted to look for causes of inferior venacaval obstruction. (Choices A and B) Surgical removal of the varicocele or embolization is not the most appropriate intervention at this time. (Choice D) Scrotal ultrasonography might show a dilated venous plexus; however, it is not very likely to show a cause of varicocele. Occasionally, testicular masses can cause varicocele by metastasis into the abdominal lymph nodes. Educational Objective: Processes that cause inferior vena caval obstruction (e.g., clot, tumor) should be ruled out in patients who have bilateral varicocele, right varicocele or varicocele that does not disappear in the supine position. 37% of people answered this question correctly

A 24-year-old Caucasian male is brought to the emergency department after being involved in a major motor vehicle accident. He has extensive injury of both lower extremities and blunt chest trauma. His initial blood pressure is 120/80 mmHg and heart rate is 100/min. His lungs are clear on auscultation; breath sounds are heard symmetrically over both lung fields. No murmur or abnormal heart sounds are heard. The abdomen is soft and non-tender on palpation. The chest x-ray is normal. Blunt cardiac trauma is being suspected because superficial injuries are present at the lower sternum

and the left side of anterior chest. What is the best next step to evaluate the possible blunt cardiac injury in this patient? B. C. D. E. A. 12-lead ECG Transesophageal echocardiography Transthoracic echocardiography CT scan of the chest Serum myocardial injury markers

This subscription is licensed to user: roopika only User ID: 123489 Explanation: Blunt cardiac injuries vary in severity, from mild trauma associated only with transient arrhythmias, to rupture of the valve mechanisms, interventricular septum, or myocardium. A patient may therefore be asymptomatic or manifest various symptoms of cardiac injury, ranging from mild chest pain to severe heart failure and cardiovascular collapse. 12-lead ECG can help identify cardiac abnormalities in trauma patients and discover underlying problems that may impact treatment decisions. Furthermore, it is the most important tool that helps identify clinically significant blunt cardiac injuries. Patients with possible blunt cardiac injuries and normal ECG findings require no further treatment or investigation. (Choice C) If the ECG is abnormal, further tests may be required that most probably include transthoracic echocardiography. (Choice E) Serum myocardial injury markers are either non-specific (e.g., CPK) or do not help predict the occurrence of complications (e.g., cardiac troponins); therefore, these are not used routinely in patients with possible blunt cardiac injury. (Choices B and D) Transesophageal echocardiography and CT scan are usually employed when aortal injuries are being suspected. Educational Objective: 12-lead ECG is the most important tool to help identify clinically significant blunt cardiac injuries. 27% of people answered this question correctly;

A 58-year-old female is admitted for an elective left knee replacement. The patient has hypertension, chronic obstructive pulmonary disease, osteoporosis, and GERD. She is currently on ramipril, amlodipine, metformin, multivitamins, calcium carbonate, pantoprazole (Protonix), and raloxifene. She is also on daily salmeterol and ipratropium bromide as metered-dose inhalations. Aside from metformin, which of the following medications should be discontinued before subjecting this patient to surgery?

B. C. D. E.

A. Raloxifene Ramipril Amlodipine Protonix Calcium carbonate

This subscription is licensed to user: roopika only User ID: 123489 Explanation: Raloxifene is a selective estrogen receptor modulator (SERM) which selectively stimulates estrogen receptors on bone cells. It has been shown to improve bone mineral density at both the hip and lumbar spine. Studies have not shown that it reduces the risk for vertebral fractures. Side effects include hot flashes and an increased risk for deep venous thrombosis and pulmonary embolism. Raloxifene should therefore be discontinued at least 72 hours prior to an elective surgical procedure to prevent deep venous thrombosis. It can be restarted after the patient is fully ambulatory. (Choices B, C, D, and E) There is no reason to discontinue amlodipine, ramipril, Protonix, and calcium carbonate. Educational Objective: Raloxifene can predispose a patient to DVT and should be discontinued in immobilized patients. 45% of people answered this question correctly; A 22-year-old male presents to your office with a midline neck swelling. He has had this swelling for the past three months, and is gradually increasing in size. He denies any pain, difficulty in swallowing, or voice change. He also denies fever, cough, palpitations, nausea, and vomiting. His past medical history is significant for a laparoscopic appendectomy at the age of 18 years. He was never exposed to ionizing radiation in the past. He denies use of tobacco, alcohol, or recreational drugs. His family history is unremarkable. He is currently on no medications and denies allergies to medications. On examination, he has a 3 cm slightly fluctuant, midline neck mass in front of the hyoid bone. The mass is nontender and has no erythema of the overlying skin. The swelling moves with protrusion of the tongue. There are no palpable lymph nodes in the neck region. The rest of the examination is unremarkable. Which of the following statements is true regarding this swelling? B. C. resection D. A. It is likely to decrease with levothyroxine therapy It has high chances of becoming malignant Imaging studies to look for normal thyroid is required before surgical It is not likely to get infected

E.

Immediate surgical resection is required

This subscription is licensed to user: roopika only User ID: 123489 Explanation: About one-third of patients with a thyroglossal duct cyst present after the age of 20. They present with a midline neck mass that moves with protrusion of the tongue. The thyroid is formed as an outpouching from the base of the tongue, and then descends to the base of the anterior neck. The thyroglossal duct connects the tongue and the thyroid gland, and a cyst can develop from the epithelial remnants within the duct. Ectopic thyroid tissue is present in a large number of patients within the thyroglossal duct cysts, but sometimes this is the only functional tissue present. Therefore, imaging studies, like a thyroid nuclear scan, ultrasound, or CT is mandatory before subjecting the patient to surgery. A CT scan is thought to be more useful because it not only delineates the normal thyroid gland, but also clarifies the anatomy of the thyroglossal duct cyst in relation to the surrounding structures. Preoperative assessment of thyroid function is also required. A fine needle aspiration biopsy is not generally required unless there is high suspicion of malignancy. (Choice A) Levothyroxine therapy is not indicated in the management of thyroglossal duct cysts. (Choice B) The chances of malignancy in thyroglossal duct cysts are around 2-3%. (Choice D) Thyroglossal duct cysts have high chances of getting infected due to the connection with the oropharynx. (Choice E) Immediate surgical resection without ascertaining the presence of functional thyroid tissue would be incorrect in the management of this patient. Educational Objective: Ectopic thyroid tissue is present in a large number of patients within the thyroglossal duct cysts, and sometimes this is the only functional tissue present. Therefore, imaging studies, like a thyroid nuclear scan, ultrasound, or CT is mandatory before subjecting the patient to surgery. 71% of people answered this question correctly;

A 56-year-old African American male is diagnosed with rectal cancer after a single positive fecal occult blood test performed at a routine check-up. His past medical history is significant for hypertension and diabetes mellitus. His current medications are atenolol, amlodipine and glyburide. He has no known drug allergies. Surgical intervention to treat his rectal cancer is considered, but the patient is seriously concerned about the functional

outcome of the surgery. Which of the following descriptions favors most a sphinctersparing surgical procedure? B. C. D. E. A. Proximal node-positive cancer Big tumor with favorable histology Failed preoperative radiation Fixed non-ulcerated tumor Perirectal fat invasion

This subscription is licensed to user: roopika only User ID: 123489 Explanation: An important determining factor for the feasibility of a sphincter-sparing surgical procedure in patients with rectal cancer is the location of the tumor. Proximal nonmetastatic rectal cancer is usually treated with lower anterior resection that is supplemented with radiation and chemotherapy in patients with node-positive disease. Distal rectal cancers are treated with either local resection (sphincter-sparing operation) or abdomino-perineal resection (extensive radical operation). (Choices B, D and E) Only selected patients with distal cancer can be treated with local resection; these patients should have mobile, non-ulcerated, and relatively small (less than 4 cm) tumors. (Choice C) Patients with a big tumor are sometimes treated with preoperative irradiation and concurrent chemotherapy prior to a planned resection, which may (in some cases) permit a sphincter-preserving local resection. Educational Objective: An important determining factor for the feasibility of a sphincter-sparing surgical procedure in patients with rectal cancer is the location of the tumor. *High yield question* 19% of people answered this question correctly

A 35-year-old Hispanic woman comes to the office. She is 20 weeks pregnant. This is her fourth pregnancy. She has had one miscarriage and two normal deliveries. She has no other medical problems. She does not use tobacco, alcohol or illicit drugs, and takes no medications. An abdominal sonogram reveals normal development of the fetus for the gestational age, but also discloses the presence of five small gallstones. Three weeks after an uncomplicated delivery, the patient returns and asks you to treat her gallstones. She denies any symptoms or discomfort. Which of the following is the most effective strategy?

B. C. D. E.

A. Perform a laparoscopic cholecystectomy Start ursodeoxycholic acid Schedule for extracorporeal lithotripsy Schedule for electrohydraulic lithotripsy Reassurance and observation

This subscription is licensed to user: roopika only User ID: 123489 Explanation: Majority of patients with asymptomatic gallstones will never experience symptoms or may continue to be asymptomatic for prolonged periods of time. The risks of therapy may exceed the benefits in such patients; therefore, no intervention is required. (Choice A) Laparoscopic cholecystectomy is indicated for symptomatic patients. (Choice C) Extracorporeal shock wave lithotripsy is only effective in patients with three or less documented cholesterol stones. The procedure is not very effective and can be painful for most patients. (Choice D) Electrohydraulic lithotripsy is an expensive, time-consuming and cumbersome procedure that is indicated for high-risk patients with non-cholesterol gallstones. The procedure involves endoscopic cannulation of the gallbladder, and the patient remains with the cannula for two weeks before a laser is applied to destroy the stones and wash them out. (Choice B) Ursodeoxycholic acid is indicated for patients with mild symptoms and small cholesterol stones. The drug slowly dissolves gallstones over one to three years, but provides symptomatic relief approximately three months after the start of therapy. It is effective in 50% of the patients (who must have a functional gallbladder); however, it causes diarrhea in some of these patients. Educational Objective: Multiple therapies are available for individuals with gallstones. Asymptomatic patients require no further intervention. Patients with mild symptoms and small cholesterol stones may respond to ursodeoxycholic acid. Extracorporeal lithotripsy may be used for those with bigger cholesterol stones. The most effective therapy for symptomatic gallstones is cholecystectomy; however, high-risk patients, especially those with non-cholesterol stones, may be treated with endoscopic electrohydraulic lithotripsy. 72% of people answered this question correctly;

A 62 year-old Caucasian man is brought to the emergency department (ED) with complaints of nausea, vomiting, crampy lower abdominal pain and distension over the past two days. His last bowel movement was four days ago. His wife tells you that he has

been unable to eat anything for the past 24 hours due to repeated episodes of vomiting. His past medical history includes hypertension, hyperlipidemia, and peptic ulcer disease. His current medications include thiazide diuretics, atorvastatin, and pantoprazole. He had an appendectomy at the age of 36 years, and surgical repair of a perforated peptic ulcer 8 years ago. His temperature is 36.7C (98F), blood pressure is 124/62 mmHg, respiratory rate is 22/min and heart rate is 96/min. His mucous membranes are dry. Abdominal examination reveals a distended abdomen with increased bowel sounds, a tympanic note throughout the abdomen, and tenderness on deep palpation over the lower abdomen. The rest of the examination is unremarkable. His initial blood work reveals marked electrolyte abnormalities. An obstruction series done in the ED reveals multiple air fluid levels in the small intestine consistent with a diagnosis of small bowel obstruction. There is some air seen in the distal colon. What is the most appropriate next step in the management of this patient? B. C. D. E. A. Consult surgery for laparoscopic intervention Stabilize the patient and perform a colonoscopy Stabilize the patient and admit him for observation Insert a rectal tube Consult surgery for emergent laparotomy

This subscription is licensed to user: roopika only User ID: 123489 Explanation: The patient in the vignette has a clinical presentation consistent with a partial small bowel obstruction. The presence of air in the distal colon makes the diagnosis of complete obstruction less likely. Partial small bowel obstruction should be initially managed with observation and supportive treatment. If the patient fails to improve in the next 12 to 24 hours, early surgical intervention is recommended. (Choices A and E) Patients with partial small bowel obstruction should be managed by conservative therapy initially before proceeding to invasive interventions; however, patients with signs of impending strangulation (incarcerated hernias) or mesenteric ischemia should undergo urgent surgical intervention to prevent further deterioration of the clinical status. (Choice B) Colonoscopy has no role in the management of patients with small bowel obstruction. (Choice D) A rectal tube is not indicated in patients with small bowel obstruction. Educational Objective: All patients with partial small bowel obstruction should be admitted and managed initially with conservative therapy (e.g., intravenous hydration, nasogastric suctioning and correction of electrolyte abnormalities). 43% of people answered this question correctly;

A 30-year-old African-American woman is brought to the emergency department after a gunshot wound to her pelvis. She is screaming in severe pain. She is diaphoretic and anxious. There is very little external bleeding. Which of the following is the earliest sign of hypovolemia? B. C. D. E. A. Increased heart rate Decreased arterial blood pressure Dry mucous membranes Decreased skin turgor Decreased urine output

This subscription is licensed to user: roopika only User ID: 123489 Explanation: Tachycardia is the immediate physiologic response to volume depletion, and is the earliest sign of hypovolemia. (Choice B) In patients with hypovolemia, the arterial blood pressure is initially normal; postural hypertension will then develop, followed by persistently low blood pressure. (Choices C and D) Dry mucous membranes and decreased skin turgor are generally the features of interstitial fluid depletion; however, these signs do not develop rapidly. (Choice E) Decreased urine output is another sign of hypovolemia resulting from salt and water retention; however, it is not the earliest sign. Educational Objective: Tachycardia is the earliest sign of hypovolemia. 78% of people answered this question correctly; A 70-year-old African-American male is brought to the emergency department with a low-trauma hip fracture. His past medical history is significant for long-term hypertension and myocardial infarction experienced two years ago. His current medications include metoprolol, enalapril, hydrochlorothiazide, and aspirin. He smokes two packs of cigarettes daily, and consumes alcohol occasionally. His mother had a vertebral fracture "when she was old." His serum calcium, phosphate, and alkaline phosphatase levels are normal. An x-ray of the hip shows osteopenia and a hip fracture. Which of the following is the single most important risk factor for this patient's condition? B. C. A. Race Medications Age

D. E. F.

Smoking Alcohol consumption Family history

This subscription is licensed to user: roopika only User ID: 123489 Explanation: While less common in men than in women, osteoporosis is associated with significant morbidity. It is estimated that a 60-year-old male has a 25% chance of suffering an osteoporotic fracture. Bone mineral density depends on peak bone mass and the rate of bone mass loss. The peak bone mass is higher in men than in women; therefore, the former present with osteoporotic fractures approximately ten years later than women. The rate of bone mineral content loss after 30 years of age is approximately 0.5 - 1% per year. Age is the most important single risk factor for osteoporosis and osteoporotic bone fracture. (Choices F, D and E) Other contributing factors such as family history, smoking, and alcohol consumption are also important. (Choice C) The medications which predispose a patient to have osteoporosis include: glucocorticoids, anti-androgens, and some anticonvulsants. (Choice B) Caucasian males are more susceptible to osteoporosis than AfricanAmericans. Educational Objective: Age is the most important single risk factor for osteoporosis and osteoporotic bone fracture. 43% of people answered this question correctly;

A 43-year-old Caucasian woman comes to the office because of increased osteoarticular pain during intercourse. She complains of great difficulty during sexual intercourse due to limited movement of hip and leg abduction, as well as severe lumbar and leg pain. She is a 325-pound obese woman with a body mass index (BMI) of 42 Kg/m2. Her past medical history is significant for severe knee and hip osteoarthritis, obstructive sleep apnea, hypertriglyceridemia, and hypercholesterolemia. Her daily medications include orlistat, ibuprofen and atorvastatin. Physical examination does not reveal new abnormalities. Which of the following is the most effective strategy to manage this patient's symptoms? A. Recommend the use of soft pads under the patient's back while having sexual intercourse.

B. intercourse. C. D. E.

Recommend high dose NSAIDS or other analgesics one hour prior to Start her on a strict low-calorie diet. Refer her to an Orthopedics specialist. Refer her for gastric bypass surgery.

This subscription is licensed to user: roopika only User ID: 123489 Explanation: The National Institute of Health (NIH) recommends surgery for those individuals with a body mass index (BMI ) of more than 40 Kg/m2. Other indications are for obese patients with: (1) coexisting serious medical problems, or (2) a markedly decreased quality of life. This patient fulfills all the requirements for a gastric bypass surgery (or gastric banding). She has a BMI of 42 Kg/m2, has a decreased quality of life, and has coexisting serious medical problems (osteoarthritis, sleep apnea, hypertriglyceridemia and hypercholesterolemia). Benefits of surgery include better diabetes control, decreased triglyceride levels, increased HDL cholesterol, hypertension prevention, sleep improvement, and depression improvement. Mortality is about 1-20%, especially in those with multiple comorbidities. Such patients will require post-surgical ICU management. (Choices A and B) Soft pads and analgesics may offer only partial symptomatic relief, but will not solve the patient's problem. (Choice C) A strict diet may not be very effective, considering that the patient is already trying orlistat (an obesity drug which acts by reducing fat absorption) without any significant improvement. (Choice D) An orthopedics referral for further evaluation may be important since hip and knee replacement may eventually be necessary; however, this does not address the underlying cause (severe obesity) of her current problems. Educational Objective: The National Institute of Health (NIH) recommends that severe obesity (BMI of more than 40 Kg/m2) is an indication for surgery. Other indications are: (1) a significant decrease in the quality of life, and (2) serious medical problems such as obstructive sleep apnea, movement limitations, or brittle diabetes. 37% of people answered this question correctly;

A healthy, 44-year-old, Middle-eastern, male taxi driver had a motor vehicle accident. He was a restrained driver. You come to help him, along with the paramedics, and find him unresponsive with spontaneous respirations. His blood pressure is 100/60 mmHg, pulse is

112/min, and respirations are 12/min. Examination shows clear lung fields and normal first and second heart sounds. He could not be extricated from the car. Suddenly, he starts to cough and spit some blood. Which of the following is the most appropriate course of action? B. C. D. E. A. Intubate immediately Suction the mouth Do cricothyrotomy Do the jaw lift maneuver Stabilize the spine first

This subscription is licensed to user: roopika only User ID: 123489 Explanation: There is a considerable risk of neck trauma during most motor vehicle accidents. The first thing to always do in such trauma cases is to secure the airway. Whenever the airway needs to be protected, it is advisable to use the jaw lift maneuver to avoid further complications if there is any neck injury. (Choice B) After securing the airway, the paramedics can proceed to suction the mouth to prevent any possible aspiration. (Choices A and C) The patient has spontaneous respirations. There is no indication for immediate intubation or cricothyrotomy at this point. (Choice E) Stabilization of the spine is also done after securing the airway. Educational Objective: Whenever there is an accident with possible neck trauma, the airway must be secured through the jaw lift maneuver to avoid further strain in the neck area. Cricothyrotomy may be necessary if there is complete obstruction of the airway. If the patient is unconscious, completely unresponsive, unable to protect the airway, not breathing or coughing, he or she will need to be intubated. 13% of people answered this question correctly

A 24-year-old baseball pitcher arrives to the emergency department with excruciating left shoulder pain that occurred after falling on an outstretched arm during a game. He has no significant past medical history. There is no history of alcohol, tobacco, or illicit drug use. His temperature is 36.7 C (98 F), blood pressure is 120/66 mmHg, pulse is 100/min, and respirations are 18/min. A thorough evaluation reveals a compound fracture of the left clavicle. What is the most appropriate treatment to be offered to the patient? A. Placement of a figure-of-eight brace

B. C. D. E.

Closed reduction of the fracture Open reduction and internal fixation of the fracture Absolute rest and strong pain medication Sling placement

This subscription is licensed to user: roopika only User ID: 123489 Explanation: Certain fractures of the shoulder need to be treated in the operating room. The common ones are: compound fractures, distal comminuted fractures, multiple trauma, and severe displacement that jeopardizes the integrity of the skin. In the majority of cases, only pain medication, rest, and a sling or a figure-of-eight brace are needed (Choices A, B, D, and E); however, the presence of the compound fracture in this patient requires open reduction and internal fixation (ORIF) of the left clavicle. Educational Objective: Most shoulder fractures are treated with a sling or a figure-of-eight brace placement, along with rest and pain control; however, some lesions, such as compound, distal comminuted, or severely displaced fractures, need surgical intervention. 39% of people answered this question correctly;

You might also like

pFad - Phonifier reborn

Pfad - The Proxy pFad of © 2024 Garber Painting. All rights reserved.

Note: This service is not intended for secure transactions such as banking, social media, email, or purchasing. Use at your own risk. We assume no liability whatsoever for broken pages.


Alternative Proxies:

Alternative Proxy

pFad Proxy

pFad v3 Proxy

pFad v4 Proxy